SlideShare una empresa de Scribd logo
1 de 44
Descargar para leer sin conexión
CICLO DE REVISION EN MEDICINA
Examen Simulacro
Tema A y B
Academia de Preparación Médica
Estudios M&C SAC.
Jr. Manuel Corpancho Nro. 313
Urb. Santa Beatriz, Lima.
www.estudiosmyc.com
Sábado, 4 de Agosto del 2012
1. Hombre de 48 años que acude a Urgencias por dolor
abdominal y vómitos. Los datos analíticos iniciales son:
GOT: 80 U/I. GTP 54 U/I. Leucocitos 21.800/mm3.
Amilasa 4.500 U/I. La TAC abdominal demuestra
colección peripancreática. Se inicia tratamiento con
fluidoterapia y analgésicos. Señale cuál de los
siguientes fármacos añadiría al tratamiento, en primer
lugar, para mejorar el pronóstico de este caso:
A) Metilprednisolona.
B) Antiinflamatorios no esteroideos.
C) Omeprazol.
D) Imipenem.
E) Inhibidores de la secreción pancreática.
2. ¿Cuál es el principal factor responsable de la no
cicatrización de una úlcera péptica?:
A) Estrés.
B) Infección por Helicobacter pylori.
C) Consumo de alcohol.
D) No abandono del hábito tabáquico.
E) Determinados hábitos dietéticos.
3. ¿Cuál de las siguientes recomendaciones es FALSA
en relación con el tratamiento de la peritonitis bacteriana
espontánea en un paciente cirrótico?:
A) El diagnóstico se basa en la existencia de más de
250 PMN/mm3 en el líquido ascítico.
B) El tratamiento de elección lo constituyen las
cefalosporinas de tercera generación.
C) La administración de albúmina intravenosa previene
el desarrollo de insuficiencia renal.
D) El tratamiento antibiótico debe mantenerse durante
15 días.
E) Una vez resuelta la infección, debe iniciarse
tratamiento profiláctico con norfloxacino.
4. Paciente de 72 años, que como único tratamiento
toma antidiabéticos orales, presenta anemia ferropénica
crónica, con hemorragias ocultas positivas. ¿Cual es el
método diagnóstico más indicado para localizar la lesión
sangrante?:
A) Tránsito gastroduodenal.
B) Gammagrafía con hematíes marcados.
C) Tránsito intestinal.
D) Colonoscopia total.
E) Panendoscopia oral.
5. ¿Cuál de las siguientes definiciones corresponde con
mayor precisión al concepto de Metaanálisis?:
A) Es una revisión narrativa en la que la bibliografía se
busca de manera no estructurada.
B) Es una revisión narrativa en la que la bibliografía se
busca de manera estructurada.
C) Es una revisión narrativa en la que se presentan
tablas estructuradas de los resultados de los distintos
estudios incluidos.
D) Es una revisión en la que se combinan
estadísticamente los resultados de los estudios
incluidos.
Examen Simulacro :: Ciclo de Revisión en Medicina :: Sábado 4 de Agosto del 2012 :: www.estudiosmyc.com
E) Es una revisión en las que se presentan tablas
estructuradas de los resultados de los distintos estudios
incluidos.
6. Le encargan el diseño de un ensayo clínico en el que
es muy importante que un factor pronóstico se distribuya
por igual en los dos grupos de tratamiento. El método de
aleatorización que usted utilizaría es:
A) Aleatorización simple.
B) Aleatorización por bloques.
C) Aleatorización estratificada.
D) Aleatorización centralizada.
E) Aleatorización ciega (ocultación de la secuencia de
aleatorización).
7. En una revista biomédica se publica un estudio, en el
que los autores notifican el resultado en coste/años de
vida ganados. ¿De qué tipo de análisis de evaluación
económica se trata?:
A) Coste de la enfermedad.
B) Coste-efectividad.
C) Coste-beneficio.
D) Coste-consecuencia.
E) Coste-utilidad.
8. La mejor forma de verificar una hipótesis en
epidemiología es a través de:
A) Un estudio descriptivo.
B) Un estudio experimental.
C) Un estudio caso-control.
D) Un estudio de cohortes.
E) Un estudio transversal.
9. Todas estas situaciones, EXCEPTO una, aumentan la
probabilidad de detectar un cáncer colorrectal, señálela:
A) Enfermedad inflamatoria intestinal de larga evolución.
B) Endocarditis por Streptococcus bovis.
C) Tabaquismo de más de 35 años de duración.
D) Ureterosigmoidostomía hace 20 años, para corregir
una malformación vesical.
E) Ingesta crónica de aspirina o antiinflamatorios no
esteroideos.
10. ¿Cuál de las siguientes características NO es propia
de un ensayo clínico controlado?:
A) Intervención experimental.
B) Tratamiento asignado según el criterio del
investigador.
C) Participación voluntaria del sujeto.
D) Asignación aleatoria a las distintas posibilidades de
tratamiento en estudio.
E) Enmascaramiento de la medicación.
11. La vacuna antineumocócica está recomendada para
todas, MENOS una de las siguientes situaciones
clínicas, señálela:
A) Alcoholismo crónico.
B) Contactos familiares de un paciente con neumonía
neumocócica
C) Fístula crónica de líquido cefalorraquídeo tras un
traumatismo craneal.
D) Infección VIH avanzada.
E) Insuficiencia cardíaca crónica.
12. ¿Cuál de la siguientes vacunas no se incluye en las
recomendaciones actuales de un paciente
esplenectomizado?:
A) Vacuna neumocócica.
B) Vacuna meningocócica.
C) Vacuna frente a Haemophilus influenzae tipo B.
D) Vacuna frente al virus de la gripe.
E) Vacuna frente al virus hepatitis A.
13. ¿En qué patología pensaría en primer lugar en un
paciente de 65 años, que presenta disminución lenta,
progresiva e indolora de su agudeza visual sin signos de
inflamación ocular?:
A) Error de refracción.
B) Distrofia corneal.
C) Papilitis.
D) Glaucoma agudo.
E) Degeneración macular senil.
14. Un paciente diabético tratado mediante
fotocoagulación focal con láser de Argón tres años
antes, presenta una pérdida brusca e importante de
visión, sin dolor ni alteraciones en la superficie ocular.
La causa más probable de esta disminución de agudeza
visual es:
A) Hemorragia vítrea.
B) Edema corneal.
C) Glaucoma crónico simple.
D) Catarata nuclear.
E) Atrofia óptica.
15. Una de los siguientes signos o síntomas, NO es
habitual en una uveítis anterior aguda:
A) Midriasis.
B) Dolor.
C) Inyección ciliar.
D) Sinequias posteriores.
E) Fotofobia.
16. ¿Cuál es el tratamiento de la obstrucción intestinal
no quirúrgica en la fase terminal del cáncer?:
A) Sonda nasogástrica, aspiración continua,
sueroterapia intravenosa.
B) Administración de vitamina grupo B, laxante y enema
de limpieza.
C) Alimentación parenteral total y esteroides i.v.
D) Morfina, buscapina y haloperidol por vía subcutánea.
E) Dieta absoluta y sueros por vía subcutánea.
17. ¿Cuál de los siguientes opioides NO es adecuado
para el tratamiento del dolor crónico de etiología
cancerosa?:
A) Metadona.
B) Meperidina.
C) Morfina.
Examen Simulacro :: Ciclo de Revisión en Medicina :: Sábado 4 de Agosto del 2012 :: www.estudiosmyc.com
D) Fentanilo.
E) Tramadol.
18. La localización ideal para hacer una traqueotomía
es:
A) La membrana crico-tiroidea.
B) La membrana mio-tiroidea.
C) Primer anillo traqueal.
D) Segundo o tercer anillo traqueal.
E) Cuarto o quinto anillo traqueal.
19. ¿Cuál de estas afirmaciones es FALSA en relación
con la otitis media secretoria?:
A) Los niños con paladar hendido son más susceptibles
a padecerla.
B) En casos unilaterales en adultos, es obligada la
inspección del cavum.
C) Su período de mayor incidencia en climas templados,
es de junio a septiembre, coincidiendo con la época de
baños.
D) El 66% de los cultivos de las muestras obtenidas por
miringocentesis presentan bacterias.
E) La otoscopia puede mostrar niveles líquidos o
burbujas de aire tras el tímpano íntegro.
20. Una mujer de 68 años, sin aparentes factores de
riesgo cardiovascular, ingresa en la Unidad Coronaria
del Hospital por un cuadro ¿agudo de cardiopatía
isquémica. En la analítica realizada a su llegada se
objetiva una anemia (Hb 8gr/dl) previamente no
conocida. En este caso, la actitud más adecuada con
respecto a la anemia es:
A) Actitud expectante, ya que sólo se debe transfundir la
anemia sintomática.
B) Transfundir hematíes.
C) Transfundir sangre total.
D) Instaurar tratamiento con eritropoyetina.
E) Administrar hierro intravenoso.
21. Los cuerpos de Howell-Jolly, son inclusiones
eritrocitarias de fragmentos nucleares y se observan en:
A) Asplenia.
B) Mielofibrosis.
C) Leucemia linfática crónica.
D) Déficit de G6PD.
E) Intoxicación por plomo.
22. ¿Cuál de los siguientes hallazgos es el más
característico de la leucemia mieloide crónica?:
A) El cromosoma Filadelfia.
B) El esplenomegalia palpable.
C) La disminución de la fosfatasa alcalina granulocitaria.
D) El aumento del ácido úrico sérico.
E) El reordenamiento del gen bcr/abl.
23. Joven de 23 años, con historia de conducta sexual
de riesgo, consulta por fiebre, malestar general y dolor
en glande. La exploración física muestra múltiples
tatuajes, "piercings" y la presencia de tres lesiones
vesiculosas en glande y adenopatías inguinales
bilaterales. ¿Cuál es el diagnóstico más probable?:
A) Herpes genital.
B) Secundarismo luético.
C) Primoinfección VIH.
D) Infección por citomegalovirus.
E) Condilomas acuminados.
24. En cuanto a la neumonía por Neumococo en
paciente infectados por el virus de la inmunodeficiencia
humana (VIH) es FALSO que:
A) Es una infección frecuentemente bacteriémica.
B) Tiene mayor incidencia que en la población general.
C) Junto a Haemophilus influenzae es la causa más
común de neumonía en pacientes con SIDA.
D) Se recomienda vacuna neumocócica en aquellos
pacientes con CD4 < 100 cel/L.
E) Esta neumonía puede ser vista en pacientes con
sistema inmune relativamente intacto.
25. La tuberculosis asociada a la infección por VIH se
caracteriza por:
A) Presentación subclínica de la enfermedad.
B) Aparición característica en los estadios de
inmunodepresión más severa (>50 CD4/mm3).
C) Elevada frecuencia de afectación extrapulmonar y
diseminada.
D) Escaso rendimiento de los métodos microbiológicos
de diagnóstico.
E) Mala respuesta al tratamiento antituberculoso.
26. NO es propio del shock tóxico estafilocócico:
A) Fiebre elevada.
B) Lesiones cutáneas.
C) Metástasis sépticas.
D) Fracaso renal.
E) Rabdomiólisis.
27. ¿Cuál de las siguientes permite diferenciar la
pielonefritis aguda de la cistitis aguda?:
A) Leucocituria.
B) Hematuria.
C) Bacteriuria.
D) Antecedentes de infección urinaria.
E) Fiebre de más de 38.5ºC.
28. Un paciente de 22 años, que acude para evaluación
de una lesión genital ulcerada, presenta un VDRL
positivo a título de 8 diluciones con un FTA-Abs
negativo. La interpretación más adecuada de estos
resultados es:
A) Falso positivo de las pruebas no treponémicas.
B) Falso negativo de las pruebas treponémicas.
C) Sífilis curada (pendiente la positivización del FTA-
Abs).
D) Sífilis curada (pendiente la negativización del VDRL).
E) Sífilis de larga evolución.
Examen Simulacro :: Ciclo de Revisión en Medicina :: Sábado 4 de Agosto del 2012 :: www.estudiosmyc.com
29. Señale cuál de las siguientes asociaciones de
helmintos y su clínica característica es INCORRECTA:
A) Ascaris lumbricoides - Síndrome de Löeffler.
B) Giardia lamblia – Mala absorción intestinal.
C) Ancylostoma duodenale - Anemia megaloblástica.
D) Strongyloides stercolaris - Síndrome de
hiperinfestación en inmunodeprimidos.
E) Taenia solium - Convulsiones generalizadas.
30. Paciente de 30 años, seropositivo VIH conocido
desde 5 años antes, con antecedentes de neumonía por
P. jivovecci, que consulta por cefalea desde 10 días
antes. La exploración física muestra como datos más
relevantes mínima rigidez de nuca y temperatura de
37,5ºC, fondo de ojo normal, TAC: ligera atrofia cortical.
La punción lumbar da salida a líquido claro con 40
células mononucleares, proteínas: 90 mgrs%, glucosa:
30 mg% (glucemia: 90 mg%). Señalar, de entre las
siguientes, la causa más probable:
A) Herpesvirus tipo 8.
B) Listeria.
C) Criptococo.
D) CMV.
E) VIH.
31. En relación con Streptococcus pyogenes y la
faringoamigdalitis, ¿cuál de las siguientes afirmaciones
NO es correcta?:
A) En tratamiento de la faringoamigdalitis estreptocócica
se efectúa con una sola inyección i.m. de 1,200,000 UU
de penicilina benzatina.
B) El tratamiento antibiótico de la faringoamigdalitis
estreptocócica se efectúa con 250,000 UU/6h oral de
penicilina V durante 10 días.
C) El tratamiento antibiótico de la faringoamigdalitis
estreptocócica se efectúa con una sola inyección i.m. de
1,200,000 UU de penicilina procaína.
D) El tratamiento antibiótico de la faringoamigdalitis
estreptocócica se efectúa con amoxicilina oral 500
mg/8h durante 10 días.
E) El tratamiento antibiótico recomendado de la
faringoamigdalitis estreptocócica en los casos de alergia
a la penicilina es un macrólido oral durante 10 días.
32. A lo largo de los últimos 10 años se ha producido un
cambio muy importante en la etiología de la Endocarditis
Infecciosa del adulto. El microorganismo más frecuente
en la actualidad, es:
A) Microorganismos del grupo HACEK.
B) Staphylococcus aureus.
C) Estafilococos coagulasa negativos.
D) Bacilos gram negativos.
E) Streptococcus viridans.
33. Un hombre de 74 años con un infarto agudo de
miocardio es tratado con estreptoquinasa. Seis horas
después desarrolla un cuadro de hipotensión arterial
severa y obnubilación. ¿Cuál de las siguientes
complicaciones es MENOS probable que sea la causa?:
A) Infarto de ventrículo derecho.
B) Tromboembolismo pulmonar.
C) Rotura del músculo papilar.
D) Rotura de la pared libre ventricular.
E) Hemorragia cerebral.
34. En el tratamiento de la hipertensión arterial, la
ventaja de los bloqueadores de los receptores de la
Angiotensina II con respecto a los inhibidores del
enzima conversor de la Angiotensina es que:
A) Son más potentes.
B) Producen menos tos.
C) No producen hiperpotasemia.
D) Se puede dar en embarazadas.
E) Se pueden dar en sujetos con estenosis de la arteria
renal bilateral.
35. El tratamiento más eficaz para prevenir recurrencias
en el aleteo o flutter auricular común es:
A) Buen control de la hipertensión arterial que con
frecuencia padecen estos enfermos.
B) Digoxina asociada a un fármaco que disminuya la
conducción en el nodo A-V (anticálcicos o
betabloqueantes).
C) Ablación con catéter y radiofrecuencia del istmo cavo
tricúspide.
D) Inserción de un marcapasos con capacidad
antitaquicardia.
E) Amiodarona.
36. Un paciente obnubilado con una presión arterial de
80/40 mmHg, un gasto cardíaco de 3 l/min, una presión
de enclavamiento pulmonar de 14 mmHg y una presión
auricular derecha de 14 mmHg, puede estar sufriendo
cualquiera de las siguientes situaciones patológicas con
la excepción de una:
A) Taponamiento cardíaco.
B) Deshidratación.
C) Infarto de ventrículo derecho.
D) Tromboembolismo pulmonar.
E) Constricción pericárdica.
37. La campana del fonendoscopio es más adecuada
que la membrana para:
A) Distinguir un soplo diastólico de uno sistólico.
B) Oír mejor el chasquido de apertura en pacientes con
estenosis mitral.
C) Valorar la presencia de un tercer y/o cuarto tono.
D) Detectar el click mesosistólico del prolapso mitral.
E) Auscultar a los niños pequeños y bebés, en los que la
membrana no capta bien la tonalidad de los ruidos
cardíacos.
38. ¿Cuál de las siguientes respuestas sobre el Delirium
Tremens es la correcta?:
A) Se trata del estado peculiar de intoxicación producido
por el consumo de alcohol.
B) Se caracteriza por la aparición de conductas
impulsivas tras el consumo de una pequeña cantidad de
alcohol.
Examen Simulacro :: Ciclo de Revisión en Medicina :: Sábado 4 de Agosto del 2012 :: www.estudiosmyc.com
C) Es un cuadro clínico con elevado índice de
complicaciones psiquiátricas y con pocas
complicaciones orgánicas.
D) Aparece con mayor frecuencia entre los hombres
alcohólicos a partir de los 50 años.
E) Su tratamiento de elección son las benzodiacepinas.
39. ¿Cuáles son los trastornos perceptivos, de entre los
que se enumeran, más característicos de las
esquizofrenias?:
A) Ilusiones hipnagógicas.
B) Alucinaciones visuales zoomórficas.
C) Alucinaciones o pseudoalucinaciones auditivas.
D) Paraeidolias.
E) Alucinosis auditivas.
40. Una paciente de 24 años acude a la Urgencia
llevada por un familiar por vómitos recidivantes. Parece
bulimia nerviosa. ¿Cuál de las pruebas de laboratorio es
más útil para evaluar la gravedad de los vómitos?:
A) Nivel de Hemoglobina.
B) Nivel plasmático de Amilasa.
C) Nivel sérico de Socio.
D) Nivel plasmático de Calcio.
E) Nivel plasmático de Creatinina.
41. Acude a urgencias un paciente de 37 años con gran
postración, fiebre elevada (38,5°C) e intensa cefalea.
Presenta en miembros inferiores, tronco y raíz de
miembros superiores un exantema tenue eritemato-
violáceo, maculoso, escasamente confluente, con
afectación de palmas y plantas. Al interrogar a
familiares, reconocen haber estado hace siete días en
una excursión. Reexplorado el paciente, en cara
posterior de la piera derecha aparece una lesión
necrótica-costrosa, de 0,5 cm. de diámetro, rodeada por
un halo violáceo, edematoso de 0,3-0,4 cm. El
tratamiento más indicado sería:
A) Doxiclina v.o. (200 mgr. cada 12 horas un día ó 100
mgr. cada 12 horas por 5 días).
B) Cloxacilina 1 gr. i.v. cada 6 horas durante 10 días.
C) Ceftriaxona 1 gr. i.v. cada 12 horas durante 10 días.
D) TMP-SMX 500 mgr. i.v. cada 8 horas por 10 días.
E) Eritromicina 250 mgr. i.v. cada 6 horas por 7 días.
42. Un paciente de 27 años ex-ADVP desde hace tres
años, con muguet oral y antecedentes de neumonía por
neumocistis carinii, presenta en el dedo índice, en
superficie dorsal de la 2.ª falange, una lesión única,
úlcero necrótica, con crecimiento serpinginoso de
aproximadamente 4-5 cms. de diámetro, bordes
geográficos y algunas áreas costrosas, hemorrágicas.
Es extremadamente dolorosa. Su 1.ª sospecha clínica
es:
A) Linfoma B cutáneo.
B) Ulcera secundaria al tratamiento.
C) Infección por herpes simple.
D) Chancro luético.
E) Picadura sobreinfectada.
43. Un paciente de 67 años, comienza a presentar
lesiones eritematoedematosas en brazos y abdomen.
Ocasionalmente se observan ampollas salpicando el
área afectada. No hay afectación de mucosas. Conserva
el estado general. Se realizó biopsia de piel donde se
observó las imagen de una vesícula subepidérmica con
eosinófilos. En inmunofluorescencia directa se observó
una imagen de depósito lineal en UDE de IgG. Tras usar
la técnica de separación con sal, la banda de depósito
aparece tanto en el suelo, como en el techo de la
ampolla. El diagnóstico más probable será:
A) Enfermedad de During-Brocq.
B) Fogo Selvagen.
C) Enfermedad IgA lineal.
D) Epidermólisis ampollosa adquirida.
E) Penfigoide ampolloso.
44. Un paciente de 44 presenta lesiones pápulo-
erosivas muy pruriginosas en glúteo y rodillas, que
comenzaron a salir hace meses, cursando en brotes que
curan espontáneamente en una semana. Se ha
realizado biopsia de piel observándose ampolla
subepidérmica en tinción de hematoxilina-eosina y un
depósito de IgA linea en UDE. La biopsia de
vellosidades intestinales en normal. Los anticuerpos
antigliadina IgA son negativos. Los anticuerpos
antigliadina IgG y los anticuerpos antiendomisio son
positivos. Presenta además anticuerpos antitiroideos. El
tratamiento adecuado será:
A) Corticoides orales porque es una enfermedad IgA
lineal.
B) Corticoides orales porque es una Dermatitis
Herpetiforme con biopsia intestinal negativa.
C) Sulfonas más dieta sin gluten, porque es una
Dermatitis Herpetiforme.
D) Sulfonas sin dieta, porque es una Dermtitis
Herpetiforme con biopsia intestinal negativa.
E) Sulfonas más inmunosupresores.
45. Un varón de 6 años presenta de manera brusca un
brote de lesiones eritematoescamosas redondeadas, de
pqueño tamaño, distribuídas por todo el tegumento. La
semana anterior acudió a Urgencias por un cuadro
catarral, con T°= 37,5°C y amigdalitis. En el cultivo
faríngeo se aisló un estreptococo. La patología cutánea
más probable corresponde a:
A) Pitiriasis rosada.
B) Eczema numular.
C) Psoriasis gutata.
D) Herpes circinado.
E) Vasculitis séptica.
46. Un joven de 26 años, con antecedentes familiares
de psoriasis, presenta un brote de lesiones en placas
eritematosas, con centro amarillento y halo periférico
descamativo, ovaladas, bien delimitadas, en cara
anterior y posterior del tronco. La 1.ª lesión había
aparecido hacía 1 semana, era de mayor tamaño y se
localizaba en el tercio superior de la espalda. El
Examen Simulacro :: Ciclo de Revisión en Medicina :: Sábado 4 de Agosto del 2012 :: www.estudiosmyc.com
paciente conservó un buen estado general en todo
momento. Probablemente el cuadro corresponde a:
A) Psoriasis en pequeñas placas.
B) Psoriasis tipo Von Zumbusch.
C) Pitiriasis rosada de Gibert.
D) Roséola sifilítica.
E) Eczema diseminado.
47. Un varón de 16 años comienza a presentar
vesículas de contenido claro agrupadas, de localización
peribucal, que a lo largo de 1 semana se van rompiendo
formándose erosiones y costras. Transcurrido este
período, en el dorso de las manos aparecen 2 lesiones
ampollosas con vesículas dispuestas periféricamente y
alguna pápula eritematosa con centro más oscuro. Se
trata de:
A) Inicio de una varicela.
B) Aparición de lesiones en manos por contagio directo
desde la zona peribucal.
C) Impétigo estafilocócico.
D) Eritema multiforme minor.
E) Síndrome de Stevens-Johnson.
48. Una mujer de 65 años viene presentando desde
hace 2 años brotes de lesiones nodulares < de 2 cm.,
localizadas de forma bilateral en MMII, acompañados de
livedo reticularis, febrícula y artralgias. La histología
corresponde a una vasculitis leucocitoclástica.
Probablemente se trata de:
A) Síndrome de Sweet.
B) Eritema elevatum diutinum.
C) Panarteritis nodosa cutánea.
D) Vasculitis urticarial.
E) Enfermedad de Kawasaki.
49. Un paciente de 63 años de edad presenta una
eritrodermia de varios meses de evolución con
adenopatías generalizadas y más de 10% de células
con núcleo cerebriforme en sangre periférica. Tiene
además intenso prurito y edemas pretibiales. Su
diagnóstico sería:
A) Eczema seborreico.
B) Exantema medicamentoso crónico.
C) Eritrodermia psoriasica.
D) Síndrome de Sezary.
E) Parapsoriasis en grandes placas.
50. Un niño sin alteraciones cutáneas al nacimiento
comienza a presentar hacia los 3 meses de edad
escamas grandes y negruzcas en tronco y
extremidades, afectando pliegues axilares y poplíteos.
Tiene un hermano mayor igualmente afecto. Es
probable que se trate de:
A) Una ictiosis ligada a X.
B) Una ictiosis laminar.
C) Una ictiosis vulgar.
D) Enfermedad de Darier.
E) Ninguna de las anteriores.
51. Una mujer acude a nuestra consulta poque tras
haber estado esa mañana expuesta al sol presenta una
quemadura solar exagerada localizada en cara
(respetando región peiorbitaria, retroauricular y
submentoniana), dorso de manos y en ambas piernas
hasta la altura de las rodillas. Estaba en tratamiento con
un diurético tiacidico.
A) Se trata de una reacción fototóxica.
B) Es una reacción fotoalérgica por mecanismo tipo IV.
C) Debe eliminarse el agente causante, poner
tratamiento sintomático y evitar la radiación lumínica
hasta que remita el cuadro.
D) Es una erupción polimorfa lumínica.
E) Son correctas A y C.
52. Una joven de 16 años de edad presenta varias
máculas de color blanco lechoso de varios centímetros
de diámetro, de distribución simétrica sobre codos,
rodillas, manos y zona peribucal. En la biopsia cutánea
hay ausencia de malanocitos. Tras fotoquimioterapia
sistémica con psoralenos han pigmentado parcialmente
las lesiones. El diagnóstico de esta paciente es:
A) Esclerosis tuberosa.
B) Lepra.
C) Hipomelanosis guttata idiopática.
D) Vitíligo.
E) Hipomelanosis de Ito.
53. Niño de 10 años de edad que acude a la consulta
por amigdalitis pultácea con adenopatias cervicales. Se
pone tratamiento con penicilina a dosis correctas. A las
72 horas acude de nuevo al no experimentar mejoria.
Debemos pensar en:
A) Posible Mononucleosis infecciosa.
B) Posible amigdalitis viral de etiología diferente al
Estreptococo beta hemolítico del grupo A.
C) Linfoma.
D) V. Parainfluenzae.
E) Rubéola.
54. La presencia de vómitos en la infancia es uno de los
motivos más frecuentes de consulta. Pueden ser
debidos a enfermedades digestivas y extradigestivas.
¿Cuál de las siguientes patologías produce menos
frecuentemente vómitos en la etapa de la lactancia?
A) Reflujo gastroesofágico.
B) Estenosis hipertrófica de píloro.
C) Apendicitis.
D) Invaginación intestinal.
E) Gastroenteritis.
55. Ante un niño que de forma brusca presenta
hipotensión, vómitos y colapso cardiovascular que no
responde a la administración de drogas vasoactivas o
catecolaminas y en la analítica realizada presenta
hiponatremia e hiperpotasemia, se debe considerar
como probable diagnóstico:
A) Insuficiencia cardíaca.
B) Insuficiencia suprarrenal.
C) Diabetes juvenil.
D) Diabetes insípida.
E) Intoxicación por monóxido de carbono.
Examen Simulacro :: Ciclo de Revisión en Medicina :: Sábado 4 de Agosto del 2012 :: www.estudiosmyc.com
56. Un varón de 17 años se encuentra mareado, con
vómitos y en el transcurso de unos minutos se halla
tumbado en el suelo en coma, con una exploración
neurológica normal. La causa más probable sería:
A) Ingestión de barbitúricos.
B) Hemorragia subaracnoidea.
C) Coma etílico.
D) Status convulsivo.
E) Tumor cerebral.
57. Un varón de 4 años tiene lesiones purpúricas
palpables, simétricas, de 3 días de evolución en las
extremidades inferiores. Los estudios hematológicos
revelan:
Hemoglobina: 10 g/dl; recuento leucocitario
16.500/mm3; recuento plaquetario 240.000/mm3 y VSG
de 45 mm/hora. La etiología más probable es:
A) Maltrato infantil.
B) Púrpura de Schonlein Henoch.
C) Enfermedad de Kawasaki.
D) Meningococemia.
E) Enfermedad de Von Willebrand.
58. Durante la evaluación previa al ingreso en el colegio
de un niño de 5 años, se detecta retraso en el habla.
Como antecedentes se recogen episodios reiterados de
cuadros catarrales sin control médico. ¿Cuál de las
siguientes es la casusa más probabnle de esta
situación?
A) Trastorno de déficit de atención con hiperactividad.
B) Hipoacusia de conducción.
C) Disfunción de la Trompa de Eustaquio.
D) Retraso mental.
E) Hipoacusia neurosensorial.
59. Niña de 7 años sin antecedentes de interés que
acude a urgencias por dolor abdominal generalizado y
vómitos desde doce horas antes. Deposición normal. No
antecedentes quirúrgicos previos. A la exploración
presenta abdomen muy distendido y dolor en zona
periumbilical con aumento de ruidos intestinales,
Blumberg (-). En la Radiografía de abdomen en
bipedestación se observa obstrucción a nivel de
intestino delgado. El diagnóstico más probable sería:
A) Invaginación intestinal.
B) Brida intestinal.
C) Malrotación intestinal.
D) Divertículo de Meckel.
E) Estenosis ileal congénita.
60. Niño de 5 meses que llevan a la consulta por tos y
secreción nasal desde hace 24 horas. Toma mal los
biberones por presentar fatiga. Se observa febrícula y
retraccion intercostal, con zonas de hipoventilación y
estertores en la auscultacion respiratoria. Existen otros
familiares con cuadro catarral. Se le pone tratamiento
sintomático y se aconseja la revisión a las 24 horas. En
la nueva visita el niño ha empeorado, con importante
insuficiencia respiratoria, sibilancias, tos continua y
fiebre de 38°C. ¿Cuál de los siguientes cuadros cree
que presenta el niño?:
A) Catarro habitual descendente con evolución a asma.
B) Aspiración de cuerpo extraño.
C) Bronquiolitis.
D) Epiglotitis.
E) Laringitis.
61. Un niño nacido a término de 2.100 gr. de peso
presenta irritabilidad y temblores amplios a las 36 horas
de vida. Se alimenta mal y tiene diarrea y obstrucción
nasal. ¿Cuál es el diagnostico más probable?:
A) Hipocalcemia.
B) Hipomagnesemia.
C) Déficit de piridoxina.
D) Sindrome de abstinencia por adición materna a
heroína.
E) Hipoglucemia.
62. Lactante de dos meses que presenta llanto agudo,
en crisis, desde hace 20 días, síntomas motores y heces
normales para su edad. ¿El diagnóstico más probable
será?:
A) Gastroenteritis aguda.
B) Intolerancia a la lactosa.
C) Cólico del lactante.
D) Otitis media aguda.
E) Hernia inguinal.
63. Lactante de tres meses sin antecedentes previos de
interés, que en el examen de salud se le detecta un
deterioro en las adquisiciones sicomotoras, el resto de la
exploración por aparatos es normal. En la anamnesis
presenta sacudidas musculares breves de cabeza y
extremidades superiores. ¿Cuál sería el diagnóstico
más probable?:
A) Cólico del lactante.
B) Hemorragia cerebral.
C) Síndrome de West.
D) Síndrome de Lenaux-Gastaut.
E) Fenilcetonuria.
64. Ante un lactante de 1,5 meses de edad que presenta
ictericia debemos pensar en todos los siguientes
cuadros, excepto en:
A) Infección urinaria.
B) Lactancia materna.
C) Atresia congénita de vías biliares.
D) Ictericia fisiológica.
E) Hipotiroidismo congénito.
65. Un paciente acude por cuadro de dolor intenso
epigástrico, de aparición brusca, acompañado de
vómitos en los que sólo consigue arrojar saliva, gran
distensión abdominal alta e imposibilidad para pasar
más allá del esófago distal con una sonda nasogástrica,
¿cuál sería la principal sospecha diagnóstica?:
A) Perforación gástrica.
B) Estenosis pilórica aguda.
C) Vólvulo gástrico agudo.
Examen Simulacro :: Ciclo de Revisión en Medicina :: Sábado 4 de Agosto del 2012 :: www.estudiosmyc.com
D) Síndrome de Boerhaave.
E) Tricobezoar.
66. Un paciente de 60 años refiere dolor en epigastrio
desde hace unas tres semanas y en menor medida
desde meses antes, que se alivia con los alimentos y
reaparece 2 horas después de las comidas, con
irradiación a hipocondrio derecho. En la gastroscopia
muestra una úlcera de 1 cm. en curvadura menor, con
bordes netos bien definidos, ausencia de islotes de
tejido dentro del nicho ulceroso, con mucosa de aspecto
normal. Ante lo cual Ud.:
A) Decide no practicar biopsia de la úlcera por tener
características de benignidad.
B) Sólo practicará biopsia del fondo del nicho.
C) Practicaría biopsias múltiples independientemente de
las características del nicho ulceroso.
D) Algunos de los signos descritos son sugestivos de
malignidad, por lo que practicaría biopsias múltiples.
E) Por no tener características de malignidad claras, no
practicaría biopsias.
67. Paciente de 84 años que presenta cuadro de diarrea
mucosa con decaimiento generalizado y pérdida de
peso. Analíticamente el paciente presenta anemia con
hipopotasemia, hiponatremia e hipocloremia; al tacto
rectal se palpa una masa homogénea de consistencia
blanda, recubierta de moco, no dolorosa. Este paciente
presentará con gran probabilidad:
A) Hemorroides.
B) Enteropatía pierde proteínas.
C) Leiomioma de recto.
D) Adenoma velloso de recto.
E) Hamartoma rectal.
68. Mujer de 42 años que acude al hospital con historia
de 10 años de disfagia, primero para líquidos y
posteriormente para sólidos, y que en la actualidad
presenta regurgitaciones de carácter principalmente
nocturno. ¿Cuál sería la prueba diagnóstica esencial?:
A) Rx de tórax.
B) Tránsito digestivo.
C) Endoscopia alta.
D) Manometría esofágica.
E) TAC torácica.
69. Un varón de 60 años sin antecedentes personales ni
familiares de interés presenta sangre roja mezclada con
las heces. El médico realiza una inspección perianal y
un tacto rectal encontrando hemorroides internas grado
III. La actitud más correcta sería:
A) Medidas higiénicas más pomada anti hemorroidal.
B) Solicitar endoscopia alta.
C) Solicitar un enema opaco.
D) Solicitar rectoscopia y enema opaco.
E) Solicitar colonoscopia total.
70. Paciente de 43 años en tratamiento por colitis
ulcerosa con corticoides y sulfasalazina; acude a
urgencias por malestar general, fiebre, distensión
abdominal, náuseas y vómitos. Presenta gran distensión
abdominal, intenso dolor a la palpación y signos de
irritación peritoneal. En la analítica presenta marcada
leucocitosis con desviación izquierda. En la RX simple
de abdomen se observa gran dilatación del colon. El
paso siguiente es:
A) Intervención quirúrgica.
B) Rectocolonoscopia.
C) Inmunosupresores.
D) Sonda nasogástrica, sueros y corticoides iv.
E) TAC abdominal.
71. Una mujer de 52 años diagnosticada de úlcera
gástrica de 2 cm. con biopsia negativa por malignidad, a
las 8 semanas de tratamiento con ranitidina 150
mg cada 12 horas, se somete a control endoscópico
encontrándose una úlcera de 0,5 cm, con nueva biopsia
negativa y totalmente asintomática. ¿Cuál es la
conducta preferida?:
A) Cambiar el medicamento a fenotidina.
B) Añadir curalfato a la ranitidina.
C) Remitir por cirugía.
D) Continuar con ranitidina 8 semanas más.
E) Suspender toda medicación.
72. Una mujer de 76 años presenta vómito en posos de
café. En la endoscopia se encontró un pólipo en cuerpo
gástrico. No se identificó ningún otro origen de la
hemorragia. El hematócrito es del 28%. ¿Cuál es la
mejor opción terapéutica?:
A) Biopsia con pinzas por estudio histológico, si es
benigna ninún tratamiento adicional.
B) Biopsia con pinzas, si es benigna, antagonistas H2.
C) Polipectomía endoscópica con asas.
D) Resección quirúrgica.
E) Actitud expectante por si se repitiese la hemorragia.
73. Un paciente de 45 años con anemia ferropénica y
colonoscopia normal se sometió a endoscopia alta
observándose un duodeno con mucosa testoreada sin
lesiones sugerentes de hemorragia. La biopsia mostró
atrofia total de vellosidades, lo cual sugiere:
A) Glardiasis.
B) Mucosa normal.
C) Celíaca.
D) Enfermedad de Crohn.
E) Linfoma intestinal.
74. Una mujer de 65 años con artritis reumatoide
deformante grave se presenta en el hospital por dolor
periumbilical de inicio nocturno, con aumento rápido de
intensidad. La exploración abdominal era casi normal
con hemocultivo positivo, recuento de leucocitos de
20.000 mm3 con desviación a la izquierda y VSG > 100
mm/h. ¿Cuál es el diagnóstico más probable?:
A) Colitis ulcerosa.
B) Enfermedad de Crohn.
C) Isquemia intestinal.
D) Parasitosis intestinal.
E) Angiodisplasia de colon.
Examen Simulacro :: Ciclo de Revisión en Medicina :: Sábado 4 de Agosto del 2012 :: www.estudiosmyc.com
75. Varón de 78 años con debilidad, pérdida de peso,
diarrea, artritis y fiebre, durante el último año. En el
examen físico: pérdida de masa muscular,
linfadenopatía y tumefacción en rodilla izquierda. En
pruebas de laboratorio: anemia ferropénica y hemault
positivo. Con Rx abdomen y enema opaco banales. El
diagnóstico diferencial debe incluir todos los siguientes,
excepto:
A) SIDA.
B) Enfermedad de Crohn.
C) Vasculitis reumatoide.
D) Enfermedad de Whipple.
E) Singellosis.
76. Se hace colonoscopia en un enfermo, donde
aparece un pólipo de 8 mm en colon sigmoide que se
extirpa sin observar otras lesiones. ¿Cuál de los
siguientes considera el intervalo de vigilancia en este
paciente?:
A) Seis meses.
B) Un año.
C) Dos años.
D) Tres años.
E) Cinco años.
77. Niña de 6 años remitida a consulta por hemorragia
vginal, que presenta un desarrollo mamario, en etapa III
de Tanner, estatura alta, con una edad ósea de 9 años
(Rx. de mano y muñeca izda.), valores basales de
gonadotropinas y estradiol elevados para la prepubertad
con ovarios aumentados para la edad y con múltiples
quistes de diámetro igual o mayor de 14 mm. Examen
neurológico clínico-radiológico normal, sin pigmentación
cutánea en mancha de café con leche y estudio
hormonal tiroideo y suprarrenal normal. El tratamiento
de elección es:
A) Laparocopia diagnóstico-terapéutica ya que su
etiología es un tumor ovárico.
B) Agonistas de GnRH, ya que estamos ante una PPV,
y los agonistas de GnRH son los únicos que retrasan el
desarrollo sexual y la maduración esquelética.
C) Danazol.
D) Acetato de medroxiprogesterona ya que producen un
retraso en el desarrollo sexual y muy buenos resultados
en el control del crecimiento.
E) No precisa tratamiento, aunque sí una vigilancia
anual.
78. Una paciente de 30 años, que consulta por
amenorrea secundaria, presenta concentraciones
plasmáticas basales de FSH 2 MUI/ml., LH 1,5, MUI/ml.,
prolactina 9 ngr./ml. Tras la administración de
gestágenos 10 mgr./día durante 5 días no se observa
sangrado vaginal. En cambio, tras la administración de
estrógenos conjugados durante 21 días y en los 5
últimos días gestágenos aparece una menstruación. De
las siguientes causas de amenorrea ¿cuál es la que
corresponde con el cuadro clínico?:
A) Síndrome de ovario poliquístico.
B) Fallo ovárico autoinmune.
C) Prolactinoma hipofisario.
D) Síndrome de Asherman.
E) Tumor cerebral.
79. Una mujer de 54 años solicita tratamiento hormonal
sustitutivo por síntomas neurovegetativos y
manifestaciones genitourinarias importantes. Refiere
que la >FUR fue hace un año y medio, pero que hace 2
meses ha empezado a sangar muy abundantemente.
Presenta exploración física y mamografía normal. ¿Cuál
es la actitud más apropiada en esta paciente?:
A) Tratamiento con gestágenos.
B) Tratamiento con estrógenos.
C) Tratamiento con terapia combinada.
D) Ecografía transvaginal para valorar línea media y si
existe alguna duda de patología endometrial realizar
histología endometrial.
E) Citología (triple toma).
80. Una mujer de 45 años con el antecedente de un
proceso gripal hace dos semanas por el que fue tratado
con amoxicilina presenta ahora un cuadro de prurito
vulvar y leucorrea. En la exploración se aprecia
enrojecimiento y edemas de la vulva y del introito y
secreción vaginal blanca grumosa de aspecto caseoso.
En la mucosa vaginal aparecen unas placas
blanquecinas irregulares que se desprenden con
facilidad y dejan ulceraciones rojas superficiales. ¿La
etiología probable es?:
A) Candidiasis o moniliasis.
B) Herpes genital.
C) Tricomaniasis.
D) Gardnerella vaginalis.
E) Clamidias.
81. Mujer de 40 años que consulta por cuadro de
poliartritis simétrica de grandes y pequeñas
articulaciones de 15 días de evolución. En la analítica
destaca un factor reumatoide positivo siendo el resto del
estudio inmunológico negativo. ¿Qué diagnóstico
realizaría?:
A) Artritis reumatoide.
B) Artritis paraneoplásica.
C) Artritis no filiada.
D) Enfermedad de Still del adulto.
E) Poliartritis vírica.
82. Mujer de 65 años que ingresa por cuadro de cefalea,
fiebre y dolor con limitación de ambos hombros y ambas
caderas de 2 meses de evolución. El resto de
anamnesis y exploración física no aporta datos
relevantes. La analítica pone de manifiesto una gran
elevación de los reactantes de fase aguda (VSG y
PCR). La conducta a seguir sería:
A) Diagnosticar a la paciente de polimialgia reumática e
iniciar tratamiento.
B) Realizar biopsia de la arteria temporal para descartar
la existencia de arteritis pues el tratamiento difiere.
Examen Simulacro :: Ciclo de Revisión en Medicina :: Sábado 4 de Agosto del 2012 :: www.estudiosmyc.com
C) Iniciar tratamiento con prednisona 1 mg./kg./día para
“curarse en salud” ante la posible existencia de una
arteritis.
D) Realizar artrocentesis de un hombro o una cadera
para descartar primero la existencia de una artritis
séptica.
E) Iniciar tratamiento con 20 mg./día de prednisona y si
no mejorara en 15 días subir a 1 mg./kg./día.
83. Varón de 25 años afecto clínica y radiológicamente
de una sacroileítis unilateral de 2 meses de evolución.
La conducta a seguir es la siguiente:
A) Solicitar analítica con HLA y dar tratamiento con
AINE ante la sospecha diagnóstica de
espondiloartropatía.
B) Además de lo anterior añadir salazopirina al
tratamiento pues probablemente padecerá una
espondilitis anquilosante.
C) Además de lo anterior realizar una buena anamnesis
y exploración física para descartar la existencia de
psoriasis.
D) Realizar anamnesis y exploración física, solicitar
analítica con HLA así como ppD y serologías a brucella.
E) Lo primero es realizar una artrocentesis de la
articulación afecta para descartar proceso infeccioso
crónico.
84. Mujer de 45 años que acude a la consulta por “dolor
óseo generalizado” desde hace varios años. La
conducta a seguir es:
A) Diagnosticarla de fibromialgia y tratarla con
analgésicos y antidepresivos.
B) Sospechar la posible existencia de proceso
metastásico y realizar un completo estudio de búsqueda
del tumor primario.
C) Realizar una correcta anamnesis, exploración física y
solicitar analítica completa con hormonas tiroideas y
CPK.
D) Además de lo explicado en el apartado 3., solicitar
una gammagrafía ósea para asegurarse de la existencia
de proceso inflamatorio articular.
E) Realizar una anamnesis y exploración física
correctas. Solicitar un estudio analítico con VSG y PCR.
Cuando exista sospecha clínica solicitar determinación
de hormonas tiroideas, anticuerpos anti-ANA y creatin-
P-quinasas.
85. Varón de 45 años que acude a la consulta por
presentar dolor, tumefacción y edema a nivel de la mano
izquierda. Entre sus antecedentes patológicos
únicamente destaca la luxación del hombro izquierdo 1
mes antes. La conducta a seguir es la siguiente:
A) Realizar artrocentesis de la muñeca para descartar
artritis séptica.
B) Solicitar analítica y radiografías ante la sospecha de
artritis de muñeca.
C) Solicitar analítica, radiografías y gammagrafía ósea
con tecnecio ante la sospecha de DSR.
D) Solicitar analítica con inmunología y radiografías ante
la sospecha de inicio de proceso inflamatorio crónico.
E) Solicitar analítica, radiografías y gammagrafía con
tecnecio y galio ante la sospecha de DSR.
86. Mujer de 20 años que acude por dolor y tumefacción
de ambos tobillos junto lesiones cutáneas
eritematovioláceas dolorosas en ambas EEII de 10 días
de evolución. El diagnóstico más probable es:
A) Artritis reactiva.
B) Vasculitis.
C) Sarcoidosis.
D) Enfermedad inflamatoria intestinal.
E) Artropatía no filiada.
87. Una paciente de 55 años sin antecedentes
patológicos de interés consulta por dorsalgia de inicio
brusco y ritmo mecánico de dolor. Aporta Rx simple de
columna en la que se aprecian aplastamientos
vertebrales múltiples. La exploración física es normal
salvo por la presencia de debilidad muscular proximal.
Ante la sospecha clínica de osteomalacia, cuál de las
siguientes exploraciones es más rentable:
A) VSG y hemograma.
B) Determinación de calcio, fosforo y fosfatasas
alcalinas.
C) Densitometría ósea.
D) Gammagrafía ósea.
E) Resonancia magnética.
88. Una paciente de 23 años con antecedentes de ulcus
duodenal presenta una poliartritis simétrica con afección
predominante de manos. En la exploración física, aparte
de la poliartritis, presenta aftas orales. Se practica
analítica general que muestra como únicas alteraciones
VSG: 40 mm/1.ª hora, leucocitos: 3.000/mm3 (1.200
linfocitos) y ANA + 1/320 patrón homogéneo. ¿Cuál
sería su diagnóstico?:
A) Artritis reumatoide.
B) Lupus eritematoso sistémico.
C) Gota poliarticular.
D) Condrocalcinosis.
E) Artritis reactiva.
89. ¿Cuál es el tratamiento indicado en la paciente de la
pregunta anterior?:
A) Antiinflamatorios no esteroideos.
B) Antipalúdicos.
C) Glucocorticoides orales.
D) Pulsos de metilprednisolona.
E) Inmunosupresores.
90. Un paciente de 45 años consulta porque en una
analítica de rutina se ha detectado una uricemia de 9
mg./dl. No existen antecedentes de artritis ni cálculos
urinarios. ¿Cuál es la actitud correcta?:
A) Iniciar tratamiento con uricosúricos.
B) Iniciar tratamiento con uricosúricos y colchicina.
C) Iniciar tratamiento con alopurinol.
D) Iniciar tratamiento con alopurinol y colchicina.
E) No precisa tratamiento.
Examen Simulacro :: Ciclo de Revisión en Medicina :: Sábado 4 de Agosto del 2012 :: www.estudiosmyc.com
91. Una paciente de 15 años es remitida por sospecha
de fiebre reumática: tras un cuadro gripal con
artromialgias generalizadas se determinaron las
antiestreptolisinas (ASLO), que son positivas. ¿Cuál de
las siguientes afirmaciones es verdadera?:
A) Ante el cuadro clínico de la paciente y las ASLO +, el
diagnóstico de fiebre reumática es seguro.
B) Con las ASLO +, bastaría para diagnosticar la fiebre
reumática.
C) La fiebre reumática es un tipo de artritis séptica.
D) Con estos datos, no se puede realizar el diagnóstico
de fiebre reumática.
E) Debemos iniciar rápidamente tratamiento con
penicilina.
92. Una paciente de 40 años presenta una gonartosis
con afección predominante del compartimento interno
de la rodilla e impotencia funcional severa. Señale la
respuesta verdadera:
A) El tratamiento es reposo absoluto y esperar a que
sea más mayor para colocarle una prótesis.
B) Una osteotomía podría estar indicada.
C) Nunca se debe tratar con antiinflamatorios.
D) No debe realizar fisioterapia ya que podría dañarse
más la articulación.
E) El uso de un bastón está contraindicado.
93. Un paciente presenta latencia del sueño de menos
de 10 minutos con dificultades para despertarse y
episodios de sueño de 18-20 horas, asociado al
despertarse con hiperfagia, hipersexualidad, aumento
de peso, irritabilidad, depresión, comportamiento
impulsivo, disfunción vegetativa y alteraciones
neurológicas. Las siestas diurnas duran varias horas.
Estos episodios duran varias semanas intercalándose
varios meses sin somnolencia. En uno de los episodios
de somnolencia el paciente tuvo un grave accidente. El
diagnóstico más probable es:
A) Epilepsia.
B) Narcolepsia.
C) Simulación o trastorno conversivo-histeria.
D) Síndrome de Klein-Levin.
E) Apnea del sueño.
94. Una chica de 17 años acude al servicio de urgencias
traída por su madre por un desmayo. Cuando la vemos
está irritable y dice querer irse de alta porque no le pasa
nada. Va con ropas holgadas y algo más abrigada que
el resto de pacientes, está delgada y parece más joven
de lo que le corresponde por su edad. Dice comer
normal y niega usar laxantes o diuréticos, su madre dice
que no come con ellos porque es muy activa y está todo
el día fuera. Rompió con el novio hace un año. Dice que
está harta de tener a su madre siempre pendiente de
ella. El dato que más nos ayudaría a diferenciar una
anorexia nerviosa de otros trastornos sería:
A) Presencia de amenorrea.
B) Peso inferior al 15%.
C) Que la paciente diga que tiene miedo a ganar peso o
que diga que se ve gorda.
D) Que diga que tiene frío.
E) Uñas y pelo frágil.
95. Una adolescente de apariencia física normal acude
por irritabilidad y tristeza. Comenta que decidió comer
menos en las comidas porque estaba algo gorda.
Haciendo esto consigue adelgazar a temporadas pero
luego se vuelve a engordar. Ahora está en período de
transición, sigue restringiendo las comidas pero se pasa
todo el día picoteando y en ocasiones come más de lo
que quisiera de forma descontrolada. En esos
descontroles efectivamente la paciente come una gran
cantidad de alimentos especialmente chocolate, galletas
y dulces, aunque en otras ocasiones son salados. Tras
estos atracones se siente muy culpable e irritada y
vomita para evitar engordarse. El diagnóstico más
probable es:
A) Síndrome de Klein-Levin.
B) Depresión bipolar.
C) Bulimia.
D) Esquizofrenia.
E) Anorexia nerviosa tipo compulsivo/purgativo.
96. Una paciente de 20 años se presenta en la guardia
quejándose al internista de que tiene ataques de
corazón y sensación de ahogo, sudoración y sensación
de mareo que se inicia bruscamente mientras estaba
leyendo relajadamente en su casa, le duró unos
minutos, creyó morir y sentía que lo que le ocurría no
era real, temiendo perder el control o estar volviéndose
loca. Se le realizan pruebas ECG y auscultación,
detectándose frecuencia cardíaca alta y signos
sugerentes de prolapso mitral. Aun así dada la gran
ansiedad de la paciente se llama al psiquiatra ya que
insiste en que se le hagan más pruebas y en que no se
quiere quedar sola en casa nunca más, ¿cuál es el
posible diagnóstico?:
A) Prolapso mitral.
B) Agorafobia.
C) Angor.
D) Trastorno de angustia.
E) A y D.
97. Una paciente de 55 años acude a consultas por
cansancio especialmente por la mañana, falta de
concentración desde hace 1 mes. Dice estar por las
mañanas despierta antes de hora sin poder dormir. Ha
perdido apetito si bien come como siempre pero
forzándose, por lo que no ha perdido peso. Pierde el hilo
de las conversaciones y está irritable. Ha dejado de
hacer las cosas que le gustaba hacer y no sale de casa
más que lo justo. Viene acompañada por su hermana y
nos ruega que no le digamos nada a su marido ya que
desde que no hace las cosas de la casa tan bien como
antes no quiere darle más disgustos. ¿Cuál es el
diagnóstico más probable?:
A) Ansiedad y depresión.
B) Demencia.
C) Depresión mayor.
D) Trastorno de personalidad.
Examen Simulacro :: Ciclo de Revisión en Medicina :: Sábado 4 de Agosto del 2012 :: www.estudiosmyc.com
E) Neurosis.
98. El paciente, que es traído por un familiar, dice no
saber porqué está aquí y parece irritado y con
agresividad contenida. El familiar nos hace gestos con
los ojos haciendo ver que el paciente no está muy bien.
Entrevistando a ambos por separado el paciente se
muestra contenido y dice estar bien, mejor que nunca.
El familiar dice que últimamente ha comprado cosas
innecesarias y ha hecho algunos regalos. Dice que
quiere iniciar la carrera de psicología, se ha apuntado a
un gimnasio y quiere proponer a su jefe algunos
cambios en la empresa si bien no tiene
responsabilidades directivas. Está muy hablador y
agudo en las conversaciones pero irritable, dominante y
con explosiones de mal genio. Debe dormir 4 horas pero
no está cansado al día siguiente, no hay problemas de
apetito. No ha habido problemas en su trabajo, pero
pasa mucho tiempo en la calle. ¿Cuál es el diagnóstico
más correcto?:
A) Esquizofrenia.
B) Manía.
C) Depresión ansiosa.
D) Trastorno esquizoafectivo.
E) Trastorno de personalidad.
99. Paciente traído por la policía por haberlo encontrado
andando descalzo por la carretera. El paciente tiene 28
años, es de otra ciudad y por la documentación que
lleva parece que pueda ser estudiante. Parece
deshidratado, completamente desaseado y con olor a
orín. Pupilas normales. La piel está tostada sin
quemaduras en las zonas de exposición al sol. Está
ausente, como absorto de manera que al hablarle más
fuerte se sobresalta y nos atiende momentáneamente
sonriéndonos de forma inapropiada hasta que ladea la
cabeza y la gira sin motivo. Murmura algo y mira al
techo, permaneciendo con los brazos extendidos. ¿Cuál
es el diagnóstico más probable?:
A) Intoxicación por LSD.
B) Esquizofrenia.
C) Demencia.
D) Cuadro maníaco.
E) Histeria.
100. Acude la madre de un presunto paciente de 18
años porque su hijo no sale casi nada de casa, parece
evitarles. Se hace su propia comida a partir de
conservas generalmente y come aparte. Nunca usa
conservas o alimentos ya empezados, ni acaba los que
no usa por completo. Alguna vez lo ha visto en el pasillo
haciendo posturas extrañas o dando golpes de kárate.
Fue aficionado a las lecturas de ovnis y ciencia ficción.
Fue regular estudiante. No trabaja ni estudia. Cierra
todas las persianas y se queja de que los vecinos están
demasiado pendientes de ellos. Por las noches a veces
no duerme. Dice que puede oír al vecino del tercero
(ellos viven en el primero) hablar mal de ellos y que
siempre está comentando lo que él va haciendo. ¿Cuál
es el diagnóstico más probable?:
A) Esquizofrenia.
B) Depresión psicótica.
C) Manía.
D) Demencia.
E) Klein-Levin.
TEMA B
1. Paciente de 64 años ingresado para cirugía diferida
que al día siguiente del ingreso presenta cuadro de
hiperactividad vegetativa, temblor, sobresaltos,
hiperactividad e hiperalerta pero con confusión,
alucinaciones y convulsiones. ¿Cuál es la causa más
frecuente?:
A) Abstinencia a alcohol.
B) Sobredosis de neurolépticos.
C) Sobredosis de benzodiacepinas.
D) Sobredosis de anticolinérgicos.
E) Ingestión de barbitúricos.
2. Mujer de 25 años que acude por intento autolítico con
benzodiacepinas. Es el quinto intento en 7 años, en esta
ocasión tras una pelea familiar. Su madre dice que es
muy voluble y caprichosa que siempre consigue salirse
con la suya mediante grandes crisis de nervios y peleas.
La madre dice estar harta ya que siempre está
regañándola mientras que su marido la perdona todo y
no la castiga lo suficiente. No tiene novio fijo ya que
rompe con ellos en seguida, es mala estudiante y no es
capaz de dedicarse a nada fijo ya que se aburre en
seguida, dice sentirse en ocasiones muy vacía y en
otras muy inspirada en hacer el bien pero nada le dura
mucho, quisiera poder tener una relación con un hombre
romántico. Sale mucho por la noche y en ocasiones no
llega hasta el día siguiente por la tarde, llegando ebria
en ocasiones. Ha perdido a sus amigas, que se
quejaban entre otras cosas de que las estaba llamando
continuamente. Le han diagnosticado previamente de
hepatitis. El diagnóstico de esta paciente es:
A) Esquizofrenia.
B) Manía.
C) Trastorno de personalidad límite o inestable.
D) Trastorno de personalidad por dependencia.
E) Trastorno esquizoide.
3. El paciente acude a nuestra consulta porque quisiera
dejar de hacer algunas de las cosas que actualmente
hace ya que le quitan mucho tiempo. El paciente tarda
mucho tiempo en vestirse ya que ha de pensar la ropa
que le dará suerte para esa mañana. Teme que si no
coge la adecuada algo podría salir mal. No puede evitar
sumar los números que ve en las matrículas de los
coches, así consigue neutralizar los pensamientos que
le vienen sobre la posibilidad de que el coche tenga un
accidente. El sabe que éstos son tonterías suyas pero
son cosas que no puede quitarse de la cabeza. Es muy
ordenado en algunos aspectos pero se le acumula el
Examen Simulacro :: Ciclo de Revisión en Medicina :: Sábado 4 de Agosto del 2012 :: www.estudiosmyc.com
trabajo y no consigue hacer las cosas que tiene
preparadas en una lista para ese día. Le gusta corregir
sus escritos hasta que estén impecables, lo mismo le
ocurre con su trabajo, es muy perfeccionista, exigente
consigo mismo y con los demás, es incapaz de delegar
trabajo. El diagnóstico del paciente es:
A) Esquizofrenia.
B) Trastorno de personalidad dependiente.
C) Trastorno obsesivo-compulsivo.
D) Fobia.
E) Depresión.
4. Paciente que acude por no poder salir de casa sola.
Dice que le da miedo salir por sitios solitarios por si le
ocurre algo y que en los sitios con mucha gente se
agobia y se pone muy nerviosa teniendo crisis de
ansiedad con palpitaciones, se imagina así misma
necesitada de ayuda y sin poder salir. Si se encuentra
acompañada se encuentra mejor ya que esa persona
podría ayudarla si ocurriera algo. Desde hace unos
meses tiene unas crisis parecidas pero más cortas
estando relajada, incluso acompañada, sin aviso, la
primera estando en la calle distraída viendo un
escaparate. Cuando le dan en casa es capaz de salir a
la calle para coger aire o bien abre las ventanas. Desde
la primera crisis cogió algo de miedo a estar sola hasta
llegar a la situación actual. Se ha comprado un teléfono
móvil para poder pedir ayuda si fuera necesario. Si entra
en un cine siempre está pendiente de sentarse cerca de
alguna salida. El cuadro clínico de la paciente es:
A) Agorafobia.
B) Hipocondría.
C) Trastorno facticio.
D) Trastorno obsesivo.
E) Agorafobia con crisis de pánico.
5. Ante un paciente con dolor epigástrico irradiado en
hemicinturón, náuseas, distensión abdominal, descenso
de los ruidos hidroaéreos y ascenso del ST en el
electrocardiograma, ¿cuál sería su actitud?:
A) Llamaría de inmediato a la Unidad de Cuidados
Intensivos para tratar el infarto agudo de miocardio.
B) Repetiría el electrocardiograma a las 8 horas para
confirmar el diagnóstico.
C) Lo diagnosticaría de pancreatitis aguda.
D) Su diagnóstico sería aneurisma disecante de aorta.
E) El paciente tiene una pericarditis aguda.
6. Ingresa en el Servicio de Urgencias un paciente con
dolor epigástrico irradiado en hemicinturón y amilasa
elevada, que pierde de forma brusca visión ¿qué
pensaría que ha ocurrido?:
A) Retinopatía de Purtscher.
B) Accidente cerebro vascular agudo de la región
occipital posterior.
C) Desprendimiento de retina.
D) Simulación.
E) Glaucoma agudo.
7. Ante una paciente que ingresa en el Servicio de
Urgencias con dolor abdominal, elevación de la amilasa
y la glucosa en suero y con un pH de 7,1 ¿en qué
pensaría primero?:
A) Pancreatitis aguda.
B) Rotura de embarazo ectópico.
C) Quiste de ovario.
D) Cetoacidosis diabética.
E) Ulcera de estómago.
8. Un paciente de 18 años consulta por ictericia sin
fiebre y coluria sin prurito. En las pruebas
complementarias presenta: Ac. anti VHA IgG positivos,
Ac. antiHBs positivos, Ac. anti HBc IgG positivos,
ecografía hepática normal, hay predominio de la
bilirrubina directa con monoconjugados, la
colecistografía oral es normal y al dar al paciente
fenobarbital se observa disminución de las cifras de
bilirrubina total. ¿Cuál sería su diagnóstico de
sospecha?:
A) Coledocolitiasis.
B) Síndrome de Gilbert.
C) Síndrome de Dubin-Johnson.
D) Hepatitis aguda por VHA, con hepatitis aguda por
VHB curada.
E) Síndrome de Rotor.
9. Ante un paciente con cirrosis alcohólica en estadío C-
10 de Child-Pugh que ingresa por descompensación
hidrópica, y que se encuentra hipotenso, taquicárdico,
oligúrico, con un sodio sérico de 125 mEq/l. y de 5
mEq/l. en orina, con sedimento normal, creatinina sérica
3 mg./dl. y un aclaramiento de creatinina de 40 ml., ¿en
qué pensaría usted?:
A) Deficiente tratamiento diurético.
B) Insuficiencia renal prerrenal secundaria a tercer
espacio.
C) Síndrome hepatorrenal.
D) Glomerulonefritis mesangial IgA.
E) Insuficiencia renal prerrenal secundaria a
deshidratación.
10. Paciente varón de 28 años, VIH(+), en tratamiento
con DDI (Dideoxinosina), que acude al hospital por dolor
intenso en epigastrio irradiado hacia la espalda, que
mejora al flexionar el tronco, acompañado de náuseas y
vómitos. ¿Cuál de los siguientes diagnósticos es el más
probable?:
A) Pericarditis.
B) Obstrucción intestinal.
C) Apendicitis.
D) Pancreatitis aguda.
E) Endocarditis.
11. Tras una semana de ingreso por una pancreatitis
aguda, a pesar del tratamiento médico sintomático
persiste la fiebre, leucocitosis e hiperamilasemia. Se
aprecia a la palpación una masa abdominal localizada
en hipocondrio derecho. ¿Cuál es el diagnóstico más
probable?:
Examen Simulacro :: Ciclo de Revisión en Medicina :: Sábado 4 de Agosto del 2012 :: www.estudiosmyc.com
A) Carcinoma de páncreas.
B) Flemón pancreático.
C) Pancreatitis crónica.
D) Plastón secundario a perforación duodenal.
E) Quiste hidatídico.
12. El principal diagnóstico de presunción ante una
enferma que acude por un cuadro de diarrea acuosa,
dos úlceras duodenales resistentes al tratamiento
médico e hipercalcemia es:
A) Insulinoma.
B) Gastrinoma.
C) Adenocarcinoma de páncreas.
D) Somatostatinoma.
E) Vipoma.
13. Varón de 60 años que acude por pérdida de peso de
10 kilos, dolor sordo en piso abdominal superior de 3
meses de evolución, acompañado de ictericia
mucocutánea y deposición de color blanco desde hace 5
días. A la exploración destaca masa palpable en
hipocondrio derecho. ¿Qué patología se sospecharía en
primer lugar?:
A) Adenocarcinoma pancreático.
B) Tumor gástrico.
C) Coledocolitiasis.
D) Colecistitis.
E) Hepatitis aguda.
14. Varón de 45 años, etilismo crónico, con dolor
abdominal en el hipocondrio izquierdo, de 3 meses de
evolución, que empeora con la ingesta y se acompaña
de deposiciones diarreicas pastosas muy mal olientes.
En las exploraciones complementarias destaca:
glucemia de 280, amilasemia en los límites de la
normalidad y en la placa de abdomen múltiples
calcificaciones a nivel de L2:
A) Pancreatitis aguda.
B) Cólico biliar.
C) Pancreatitis crónica.
D) Hepatitis aguda.
E) Ulcus gástrico
15. Paciente de 60 años con fibrilación auricular en
tratamiento con amiodarona desde hace meses,
colelitiasis diagnosticada ecográficamente e
insuficiencia renal crónica moderada. Consulta por
cuadro de anorexia, astenia, náuseas, vómitos e ictericia
mucocutánea. En los datos de laboratorio destaca un
leve aumento de las transaminasas y una creatinina de
2,2. Se realiza una biopsia hepática, observándose al
microscopio electrónico cuerpos lisosómicos lamelares
cargados de fosfolípidos. Qué proceso patológico le
sugieren estos datos:
A) Hepatitis viral aguda.
B) Cólico biliar.
C) Pancreatitis aguda.
D) Hepatitis tóxica de origen medicamentoso.
E) Hepatitis viral crónica.
16. Un varón de 13 años que había presentado un
cuadro febril de vías respiratorias altas de una semana
de evolución acude a urgencias por vómitos súbitos e
incoercibles. La semana previa había consumido ácido
acetilsalicílico para la sintomatología respiratoria y tres
semanas antes estuvo en contacto con un paciente con
hepatitis aguda por virus B. Dos días después del
ingreso el paciente se encuentra estuporoso con
convulsiones sin signos neurológicos de focalidad y
dolor en hipocondrio derecho con hepatomegalia. ¿Cuál
sería su diagnóstico?:
A) Cuadro convulsivo en relación con la fiebre.
B) Hepatitis fulminante vírica.
C) Síndrome de Reye.
D) Reagudización del cuadro gripal.
E) Intoxicación por ácido acetilsalicílico.
17. Un hombre de 21 años recibió un golpe en el escroto
dos horas antes de ser examinado en urgencias. Su
escroto está tenso, hinhado, y equimótico. No se
pueden palpar los testículos. El paso siguiente es:
A) Hacer uretrografía retrógrada.
B) Hacerle un Doppler de flujo color.
C) Realizar una ecografía del escroto.
D) Tratarle con hielo, reposo y suspensorio.
E) Hacer exploración quirúrgica del escroto.
18. Un hombre de 64 años presenta una hinchazón
indolora del testículo derecho de tres meses de
duración. Los resultados de los análisis de orina son
normales y la ecografía testicular muestra un aumento
del tamaño de dicho teste.
El diagnóstico más probable es:
A) Linfoma testicular.
B) Leucemia linfática crónica.
C) Seminoma espermatocítico.
D) Teratocarcinoma.
E) Carcinoma de células embrionarias.
19. Una mujer sana presenta de forma aguda
polaquiuria y disuria. En el sedimento urinario se
observan más de 5 leucocitos por campo y el
urinocultivo revela
1.000 colonias de E. coli por ml. El diagnóstico más
probable es:
A) Uretritis por clamydia.
B) Síndrome uretral.
C) Bacteriuria por E. coli.
D) Cistitis intersticial.
E) Cistitis quística.
20. Un hombre de 74 años con cáncer de próstata
metastásico conocido presenta dolor agudo de cadera
derecha. Hace dos años se le practicó una orquiectomía
pero no ha recibido ningún otro tratamiento. Su estado
general es bueno. Una gammagrafía ósea muestra
metástasis difusas y una Rx simple revela una osteólisis
en el acetábulo derecho. La siguiente medida a aplicar
es:
A) Flutamida.
Examen Simulacro :: Ciclo de Revisión en Medicina :: Sábado 4 de Agosto del 2012 :: www.estudiosmyc.com
B) Análogos de la LH-RH.
C) Radioterapia localizada.
D) Fosfato de estramustina.
E) Ketoconazol.
21. Un recién nacido presenta hematuria, proteinuria y
creatinina elevada. Las presiones de la arteria umbilical
están significativamente altas y el paciente desarrolla
una insuficiencia cardíaca congestiva. Una gammagrafía
renal revela una ausencia de función en el riñón
izquierdo. El diagnóstico más probable es:
A) Trombosis de la vena renal.
B) Necrosis cortical renal.
C) Hemorragia adrenal.
D) Rotura de un nefroma mesoblástico.
E) Trombosis de la arteria renal.
22. Un hombre de 35 años presenta un cólico renal
izquierdo. En la Rx de abdomen se observa un cálculo
de 3 mm. de diámetro alojado en uréter medio. La
necesidad de tratamiento quirúrgico depende de:
A) La duración de los síntomas del paciente.
B) Del número de episodios de cólicos previos.
C) Del número de intervenciones quirúrgicas previas.
D) De la presencia de fiebre e infección urinaria.
E) De una anormalidad metabólica subyacente.
23. A un paciente se le somete a una intervención de
bypass en el intestino delgado y presenta una litiasis
urinaria. ¿Cuál será la composición más probable de la
litiasis?:
A) Acido úrico.
B) Urato amónico.
C) Fosfato cálcico.
D) Oxalato cálcico.
E) Estruvita.
24. El factor pronóstico más importante en los niños que
presentan un tumor de Wilms intracava es:
A) La histología.
B) El volumen del tumor.
C) La extensión atrial del tumor.
D) La afectación de ganglios linfáticos.
E) La diseminación del tumor durante la intervención
quirúrgica.
25. Un muchacho de 7 años presenta de forma súbita
dolor escrotal derecho de 4 horas de duración. Se
sospecha de una torsión testicular. ¿Cuál de las
siguientes observaciones es la más probable?:
A) La ausencia del reflejo cremastérico.
B) El aumento de la captación por parte del teste
derecho después de la exploración radioisotópica.
C) La presencia del reflejo cremastérico.
D) Una transiluminación correcta del compartimento
escrotal derecho.
E) La estetoscopia con Doppler será simétrica en ambos
compartimentos escrotales.
26. En un paciente asmático, ¿cuál de los siguientes
fármacos está contraindicado en el tratamiento de la
incontinencia?:
A) La efedrina.
B) Sudafed.
C) El propranolol.
D) La fenilefrina.
E) Las anfetaminas.
27. Ante un paciente de 60 años con un cáncer de
próstata localizado, mal diferenciado y sin metástasis,
qué tratamiento le recomendaría para intentar aumentar
su supervivencia:
A) Prostatectomía radical.
B) Análogos de la LH-RH.
C) Flutamida.
D) Estramustina.
E) Orquiectomía.
28. Ante un paciente de 55 años que presenta una
tumoración vesical que infiltra la muscular y que ocupa
la mitad de la vejiga, ¿cuál sería el tratamiento que
emplearía con intención curativa?:
A) Cistectomía radical y derivación urinaria.
B) Resección transuretral vesical.
C) Quimioterapia intravesical.
D) Quimioterapia sistémica.
E) Cistectomía parcial.
29. Paciente mujer de 75 años de edad, que es traída a
la consulta por cambio progresivo de conducta en los
últimos meses. Previamente habían observado fallo de
memoria reciente sin poder precisar el momento de
inicio. Destacaba a la exploración lenguaje pobre y fallos
de juicio. ¿Cuál es el diagnóstico más probable en esta
paciente?:
A) Depresión.
B) Síndrome confusional agudo.
C) Demencia tipo Alzheimer.
D) Demencia multiinfarto.
E) Trastorno de la personalidad.
30. Varón de 87 años, con amaurosis bilateral, que
ingresa por neumonía basal derecha en la Unidad de
Agudos del hospital. La noche del ingreso presenta
cuadro de agitación psicomotriz, con alteración del nivel
de conciencia y desorientación temporoespacial. ¿Cuál
es el diagnóstico más probable?:
A) Demencia.
B) Depresión delirante.
C) ACVA.
D) Síndrome confusional agudo.
E) Ninguno.
31. Paciente de 85 años de edad con antecedentes de
insuficiencia cardíaca en tratamiento con diuréticos, e
insomnio que trataba con lorazepam. Presenta nicturia
2-3 veces. Sufre caída al levantarse bruscamente
durante la noche para ir al retrete. ¿Qué factores
pueden haber contribuido e la caída?:
Examen Simulacro :: Ciclo de Revisión en Medicina :: Sábado 4 de Agosto del 2012 :: www.estudiosmyc.com
A) Fármacos.
B) Hipotensión postural.
C) Factores ambientales.
D) Ninguno.
E) Todos ellos.
32. Paciente de 75 años de edad, varón, que vive solo.
Es encontrado la mañana del día 15 de enero caído en
el suelo. A su llegada al hospital se evidencia afasia
motora y hemiplejía derecha diagnosticándose de
ACVA. ¿Cuál de las siguientes patologías debe ser
descartada en la valoración inicial?:
A) Rabdomiólisis.
B) Hipertiroidismo.
C) Depresión.
D) Hipotermia.
E) A y D.
33. Paciente mujer de 85 años que ingresa por fractura
pertrocantérea de cadera derecha colocándose tracción.
Se evidencia neumonía basal derecha que obliga a
retrasar la intervención. La paciente sufre de
incontinencia urinaria que se maneja con catéter. A los
cinco días se observa enrojecimiento en región sacra
que no palidece con la presión, diagnosticándose de
úlcera por presión grado I. ¿Cuál es el principal factor de
riesgo para esta complicación?:
A) Incontinencia urinaria.
B) Hipoxemia.
C) Edad avanzada.
D) Inmovilidad.
E) Todos.
34. Paciente de 70 años que sufre caída al suelo
golpeándose en la cabeza. Al cabo de unas semanas
sufre trastornos de conducta, pérdida de memoria y
posteriormente alteración del nivel de conciencia. ¿Qué
diagnóstico debe descartarse en primer lugar?:
A) Hipotiroidismo.
B) Demencia tipo Alzheimer.
C) Pseudodemencia.
D) Hematoma subdural.
E) Tumor cerebral.
35. Mujer de 75 años que consulta por incontinencia
urinaria, en la que predomina la urgencia-miccional y
que no sigue ningún tratamiento farmacológico habitual.
A través de la exploración física no se objetiva patología
orgánica, siendo el residuo postmiccional normal. El
estudio analítico es normal. El tratamiento médico de
elección sería:
A) Sondaje vesical intermitente.
B) Calcioantagonistas.
C) Cirugía.
D) Antocolinérgicos.
E) Colector externo.
36. Durante los últimos 3 meses un varón de 80 años
presenta una rápida progresión de una demencia
acompañada de signos extrapiramidales y mioclonías.
El diagnóstico de sospecha inicial sería:
A) Demencia senil tipo Alzheimer.
B) Enfermedad de Creutzfeldt-Jakob.
C) Enfermedad de Huntington.
D) Hidrocefalia a presión normal.
E) Enfermedad de Parkinson.
37. ¿Cuál de las lesiones cutáneas siguientes se asocia
más frecuentemente con neoplasia oculta en un
paciente anciano?:
A) Penfigoide bulloso.
B) Dermatomiositis.
C) Eritema multiforme.
D) Herpes zoster.
E) Pénfigo vulgar.
38. Anciano de 70 años con un melanoma maligno de
reciente diagnóstico y sin otra patología médica. Su
pronóstico vital estará más estrechamente relacionado
con una de las siguientes características:
A) Ausencia de regresión.
B) Tipo histológico de la lesión.
C) Grado de invasión.
D) Presencia de ulceración.
E) Lugar de la lesión.
39. Una mujer de 75 años con historia de diabetes
mellitus no insulindependiente y epilepsía secundaria a
enfermedad cerebro-vascular padece una inflamación
con retracción gingival. Se encuentra en tratamiento con
glipizida y fenobarbital. La causa más probable de su
proceso gingival será:
A) Caries dental.
B) Edentulismo.
C) Tratamiento con fenobarbital.
D) Déficit de cinc.
E) Pobre higiene oral.
40. Un varón de 76 años con historia de diabetes
mellitus de larga evolución y con datos clínicos de
polineuropatía periférica, empieza a tomar amitriptilina
por prescripción médica para las parestesias en
miembros inferiores. De forma rogresiva nota
disminución del volumen de diuresis y ocasionalmente
escapes involuntarios de orina. En la revisión médica
siguiente se objetiva deterioro del estado general con
insuficiencia renal. ¿Cuál es la causa más lógica de su
deterioro clínico?:
A) Nefrotoxicidad por amitriptilina.
B) Pielonefritis aguda.
C) Infección urinaria de vías bajas.
D) Retención urinaria con fracaso renal secundario.
E) Glomeruloesclerosis diabética.
Examen Simulacro :: Ciclo de Revisión en Medicina :: Sábado 4 de Agosto del 2012 :: www.estudiosmyc.com
41. Varón de 45 años con otalgia derecha y sensación
de taponamiento auditivo, sin otorrea. A los dos días
presenta aumento del dolor, que se hace retroauricular,
y fiebre en agujas. El Rinne es negativo en oído derecho
y el Weber lo lateraliza a la derecha. El diagnóstico más
probable es:
A) Colesteatoma antral invasivo.
B) Petrositis.
C) Tromboflebitis del seno lateral.
D) Otitis externa maligna.
E) Carcinoma de oído derecho.
42. Mujer de 22 años que presenta parálisis facial
periférica derecha. A la exploración presenta otoscopia
normal y lengua geográfica fisurada. La paciente
comenta haber tenido otro episodio anteriormente,
aquella vez asociado a edema de labio inferior. El
diagnóstico más probable es:
A) Síndrome de Guillain-Barré.
B) Síndrome de Heerfordt-Waaldenström.
C) Déficit de C1-inhibidor.
D) Síndrome de Melkerson-Rosenthal.
E) Parálisis facial de Bell.
43. Paciente que presenta hipoacusia neurosensorial
izquierda de larga evolución con caída en agudo, y
acúfeno persistente. La actitud correcta en este caso
sería:
A) No hacer nada, pues se trata de un traumatismo
acústico crónico.
B) Potenciales evocados para descartar neurinoma del
VIII.
C) Instaurar terapia vasodilatadora endovenosa, pues se
trata de una hipoacusia súbita.
D) Intervenir el posible colesteatoma.
E) Administrar sedantes vestibulares ante la posibilidad
de vértigo de Ménière.
44. Paciente de 65 años con rinorrea unilateral
purulenta, dolor hemifacial y epistaxis ocasionales. En
Rx se observan lesiones osteolíticas en maxilar.
Probablemente se trate de:
A) Cuerpo extraño nasal.
B) Ocena.
C) Granuloma sangrante de tabique.
D) Carcinoma de fosa nasal.
E) Coriza.
45. Un varón de 60 años consulta por tumoración
indolora en raíz nasal, que desplaza la órbita
produciéndole diplopia. El cuadro se debe
probablemente a:
A) Mucocele etmoidal.
B) Quiste de retención en seno frontal.
C) Enfermedad de Woakes.
D) Papiloma invertido en techo de fosa nasal.
E) Plasmocitoma solitario en seno frontal.
46. Una mujer de 45 años acude a la urgencia con un
síndrome de disnea y estridor importantes, con tiraje
supraclavicular. A la exploración presenta taquicardia y
leve cianosis, un tiroides agrandado y, en la
laringoscopia, parálisis de ambas cuerdas vocales en
posición paramediana. La actitud que debe seguirse es:
A) Bolo de corticoides endovenosos, ante la posibilidad
de carcinoma subglótico.
B) Tiroidectomía de urgencia, pues probablemente el
tiroides agrandado comprime la tráquea.
C) Intubación y observación.
D) Traqueostomía de urgencia, pues se trata de una
parálisis recurrencial bilateral secundaria a patología
tiroidea.
E) Administración endovenosa de espasmolíticos.
47. Un niño de 3 años es traído a urgencias con un
cuadro de fiebre y tos irritativa "perruna", a lo que se
asocia disfonía y cierto grado de disnea. El cuadro es
compatible con todas menos:
A) Cuerpo extraño en vías aéreas.
B) Laringotraqueítis aguda.
C) Epiglotitis aguda.
D) Adenoamigdalitis aguda obstructiva.
E) Edema alérgico.
48. Paciente mujer de 25 años que consulta por pérdida
de audición de comienzo insidioso. Su madre era sorda
y ganó audición tras operarse del oído. Otoscopia
normal. Rinne negativo oído izdo., positivo oído dcho.
Weber a la izda. Audiometría: hipoacusia transmisiva
izquierda. Reflejo estapedial abolido en oído izdo. y
presente en derecho. Timpanometría normal. El
diagnóstico probable es:
A) Otosclerosis oído izquierdo.
B) Colesteatoma izquierdo.
C) Fijación de cadena osicular derecha.
D) Timpanosclerosis izquierda.
E) Luxación de cadena osicular izquierda.
49. Un varón de 30 años acude a urgencias por vértigo
periférico intenso con nistagmus a la derecha y acúfeno
en oído izquierdo. Recuerda haber tenido déficit auditivo
izquierdo previo, que le desapareció. Audiometría:
hipoacusia neurosensorial izda. leve. Reflejo estapedial:
derecho en 70 dB; izquierdo en 50 dB. Diagnóstico
probable:
A) Neuronitis vestibular.
B) Neurinoma del VIII par con reclutamiento positivo.
C) Tumor de tronco cerebral.
D) Otitis media secretora con fístula perilinfática.
E) Síndrome de Ménière.
50. Un niños de 2 años presenta rinorrea purulenta de
larga evolución por fosa nasal derecha, con mala
ventilación nasal. Son diagnósticos posibles todos
menos:
A) Tuberculosis nasal.
B) Cuerpo extraño intranasal.
C) Sinusitis maxilar.
D) Coriza común.
E) Rinitis alérgica sobreinfectada.
Examen Simulacro :: Ciclo de Revisión en Medicina :: Sábado 4 de Agosto del 2012 :: www.estudiosmyc.com
51. Paciente de 30 años que acude a urgencias
presentando una ulceración amigdalar unilateral. Son
diagnósticos probables todos menos:
A) Amigdalitis tifoidea de Duguet.
B) Angina de Ludwig.
C) Carcinoma escamoso de amígdala.
D) Angina de Plaut-Vincent.
E) Agranulocitosis.
52. Un niños de 12 años acude a urgencias por
obstrucción nasal bilateral crónica y otitis seromucosa
bilateral. Recientemente ha tenido epistaxis importante
por ambas fosas nasales. El diagnóstico más probable
es:
A) Hipertrofia adenoidea.
B) Ototubaritis asociada a rinitis alérgica.
C) Poliposis nasal bilateral.
D) Adenocarcinoma de etmoides.
E) Angiofibroma nasofaríngeo.
53. Mujer de 75 años con historia de cefalea presenta
pérdida súbita de visión unilateral con edema de papila
ipsilateral. ¿Qué medida tomaríamos en primer lugar?:
A) TAC.
B) VSG.
C) Ingreso hospitalario paratratamiento antibiótico iv.
D) Observación.
E) Radiografía centrada en el agujero óptico.
54. Mujer de 20 años con ojo rojo bilateral, acompañado
de quemosis, folículos conjuntivales tarsales,
adenopatía preauricular, sin pérdida de visión. La
etiología más frecuente será:
A) Adenovirus.
B) S. aureus.
C) H. influenzae.
D) Queratitis herpética.
E) Parainfluenzae virus.
55. Mujer de 23 años que presenta pérdida brusca de
agudeza visual indolora en ojo derecho, en la
exploración se observa defecto pupilar aferente relativo
en ojo derecho, segmento anterior normal y F.O. normal.
El diagnóstico más probable es:
A) Neuritis óptica retrobulbar.
B) Histeria.
C) Compresión quiasmática por tumor hipofisario.
D) Compresión del globo ocular por tumor orbitario.
E) Glaucoma agudo de ángulo cerrado.
56. Varón de 50 años con ojo rojo y doloroso, midriasis
arreactiva y cámara anterior estrecha. ¿Cuál de las
siguientes respuestas es falsa?:
A) No sería raro que fuera hipermétrope.
B) Puede presentar cefalea con náuseas y vómitos.
C) Evitar tomar la PIO por posible etiología infecciosa.
D) El tratamiento definitivo es láser o cirugía.
E) Puede presentar visión en halos de colores.
57. Ante un varón de 32 años con síndrome febril y
pérdida visual unilateral que presenta en fondo de ojo
lesiones en queso y tomate. ¿Cuál de las siguientes
afirmaciones es falsa?:
A) Sería aconsejable realizar serología HIV.
B) El pronóstico vital del enfermo es malo.
C) No necesita tratamiento por ser con frecuencia una
alteración transitoria.
D) Con bastante probabilidad se deberá a CMV.
E) La afectación suele ser bilateral.
58. Paciente de 70 años que presenta metamorfosias,
micropsia y disminución de la agudeza visual de varias
semanas de evolución, el diagnóstico más probable es:
A) Catarata nuclear.
B) Catarata subcapsular posterior.
C) Degeneración macular senil.
D) Pars planitis.
E) Hemorragia vítrea.
59. Ante un niño de 5 años con endotropía que ha
seguido tratamiento con corrección óptica y colusiones y
cuya agudeza visual aún no es normal. La pauta a
seguir será:
A) Corrección quirúrgica de la endotropía.
B) Continuar las oclusiones sobre ojo con mejor visión.
C) Continuar las oclusiones sobre ojo con peor visión.
D) No ocluir más y pasar a otro tratamiento.
E) Observación.
60. Varón de 70 años que presenta pérdida visual
progresiva unilateral, que precisa cambios sucesivos de
corrección óptica miópica. La causa más probable es:
A) Degeneración macular senil.
B) Glaucoma crónico simple.
C) Vitritis senil.
D) Coriorretinosis senil.
E) Catarata nuclear.
61. Varón de 51 años que sufre súbita pérdida visual
total e indolora en ojo derecho, apreciándose en el F.O.
una retina de color blanquecino con mácula rojo-cereza.
El diagnóstico más probable es:
A) Desprendimiento de retina con afectación macular.
B) Enfermedad de Tay-Sacks.
C) Enfermedad de Newman-Pick.
D) Obstrucción de arteria central de la retina.
E) Obstrucción de vena central de la retina.
62. Varón de 10 años de edad que presenta estrabismo,
disminución de agudeza visual y leucocoria en ojo
derecho, sin malformaciones oculares. ¿Qué patología
debemos descartar como más probable?:
A) Retinoblastoma.
B) Catarata congénita.
C) Vítreo primario hiperplásico persistente.
D) Enfermedad de Coats.
E) Fibroplasia retrolental.
Examen Simulacro :: Ciclo de Revisión en Medicina :: Sábado 4 de Agosto del 2012 :: www.estudiosmyc.com
63. Paciente de 65 años con Diabetes mellitus tipo II de
10 años de evolución con mal control metabólico
presenta disminución de visión brusca unilateral. La
causa más probable será:
A) Isquemia macular.
B) Desprendimiento de retina traccional.
C) Hemorragia vítrea.
D) Edema macular.
E) Obstrucción de arteria central de la retina.
64. Varón de 35 años sano de carácter nervioso
comienza con metamorfopsias y escotoma central
unilateral, disminución de visión moderada y patrón
angiográfico en chimenea. El diagnóstico más probable
será:
A) Distrofia viteliforme de Best.
B) Retinosis pigmentaria.
C) Degeneración macular ligada a la edad.
D) Enfermedad de Coats.
E) Coriorretinopatía central serosa.
65. Una mujer de 35 años, no fumadora, atleta de fondo
y sin antecedentes personales de interés refiere en los
últimos 3 meses una menor respuesta al ejercicio
habitual, con disnea de moderados esfuerzos. Tos no
productiva pero niega fiebre. En una ocasión reciente
presentó la emisión de varios esputos hemoptoicos, que
no se han vuelto a repetir. La Rx de tórax muestra un
patrón reticular fino difuso bilateral y un mínimo derrame
pleural derecho. En las pruebas funcionales llama la
atención el incremento de los volúmenes pulmonares.
De los siguientes, ¿cuál le parece el diagnóstico más
probable?:
A) Lupus eritematoso sistémico.
B) Tuberculosis.
C) Sarcoidosis.
D) Linfangioleiomatosis.
E) Enfermedad de Hamman-Rich.
66. Varón de 50 años, no fumador, que refiere disnea de
moderados esfuerzos de unos 4 meses de evolución
con tos no productiva. Ha recibido tratamiento con
diuréticos de asa (furosemida) en el último mes tras
realizarse una Rx de tórax (que no aporta). Acude a
urgencias por incremento de la disnea y expectoración
de esputos claros. Refiere febrícula de predominio
vespertino y pérdida de unos 6 Kg de peso. Presenta
hipoxemia con hipocapnia (insuficiencia respiratoria
parcial) y la Rx de tórax muestra densidades difusas
bilaterales, confluentes, mal definidas de predominio
parahiliar con un índice cardiotorácico en el límite de la
normalidad. FVC: 65%, FEV1: 70%, FEV1/FVC: 75%.
DLCO: 60%.
A) Insuficiencia cardiaca. Edema agudo de pulmón.
B) Enfermedad de Hamman-Rich.
C) Hemorragia pulmonar.
D) Neumonía por CMV.
E) Carcinoma bronquioalveolar.
67. Varón de 64 años, veterinario de profesión, bebedor
esporádico que presenta fiebre, cefalea y artromialgias
desde hace 7 días. Durante las últimas 48 horas refiere
tos productiva, dolor pleurítico en el costado derecho y
disnea progresiva. Ha sido tratado con eritromicina,
pese a lo que se encuentra febril y desorientado. Se
evidencian estertores crepitantes en la base derecha y
una hepatomegalia a 4 cm del reborde costal. En las
pruebas complementarias efectuadas, destaca Hb de
14.8 mg/dL, leucocitosis (17.000/μL) con desviación
izquierda, GOT (AST): 106, GPT (ALT): 82. En la Rx de
tórax se aprecia un infiltrado intersticial en lóbulo inferior
derecho. Su diagnóstico de presunción debe ser:
A) Neumonía neumocócica.
B) Neumonitis por hipersensibilidad.
C) Fiebre Q.
D) Legionella.
E) Tuberculosis.
68. Varón de 37 años que viene presentando durante
los tres últimos meses astenia, esputos hemoptoicos y
disnea progresiva hasta hacerse de mínimos esfuerzos,
con intolerancia al ejercicio. Salvo una TA de 90/60, los
datos exploratorios son anodinos. Sin embargo, los
estudios complementarios nos sorprenden: pH: 7.37,
PaO2: 62, PaCO2: 37, HCO3: 27. Hb: 9.2 y creatinina
de 2.3. La Rx de tórax muestra infiltrados difusos
parahiliares bilaterales. Ante los hallazgos reseñados,
se añade la petición de un sedimento de orina, que
muestra microhematuria y proteinuria. De las siguientes,
¿qué prueba diagnóstica le parece prioritaria en la
evaluación del enfermo?:
A) Test de difusión (DLCO).
B) Examen citológico (esputo o lavado broncoalveolar).
C) Anticuerpos anti membrana basal glomerular, c-
ANCA.
D) Biopsia pulmonar.
E) Biopsia renal.
69. En el caso anterior, la capacidad de difusión del CO
está aumentada y el título de anticuerpos antimembrana
basal glomerular es de 1:128. ¿Cuál le parece el
diagnóstico más probable?:
A) Granulomatosis de Wegener.
B) Síndrome de Goodpasture.
C) Tuberculosis.
D) Tromboembolismo pulmonar.
E) Granulomatosis de Churg-Strauss.
70. Mujer de 78 años que en el curso de un
postoperatorio por fractura de cadera comienza con un
cuadro brusco de disnea y febrícula. Exploración:
taquipnea a 30 r.p.m., taquicardia a 130 l.p.m., refuerzo
del segundo tono, abolición del murmullo vesicular en
base de pulmón derecho y extremidades sin edemas, no
dolorosas, sin signos flogóticos. Complementarios: GAB:
pH: 7.52, PaO2: 56, PaCO2: 30, HCO3: 25. 13.000
leucocitos con desviación izquierda.
Rx de tórax: pinzamiento del seno costodiafragmático
derecho. ECG: Taquicardia sinusal con bloqueo
Examen Simulacro :: Ciclo de Revisión en Medicina :: Sábado 4 de Agosto del 2012 :: www.estudiosmyc.com
incompleto de rama derecha. Señale la actitud más
adecuada:
A) Diuréticos.
B) Toracocentesis.
C) Antibióticos.
D) Corticoides.
E) Heparina.
71. Mujer de 37 años, fumadora, que consulta por
presentar durante el último mes fiebre, malestar general,
artralgias y una erupción cutánea dolorosa en ambas
piernas. Complementarios. Rx de tórax: adenopatías
hiliares bilaterales, sin afectación del parénquima
pulmonar. Mantoux negativo. Se realizó una FBC con
lavado broncoalveolar (LBA). Líquido del LBA: 22% de
linfocitos con cociente CD4/CD8 de 5.2. El diagnóstico
más probable es:
A) Tuberculosis.
B) Sarcoidosis.
C) Linfoma.
D) Cáncer de pulmón .
E) Asbestosis.
72. Varón de 47 años que acude a urgencias por fiebre,
tos, artralgias y rinorrea purulenta con ulceraciones de la
mucosa nasal de dos semanas de evolución. Inició
tratamiento antibiótico 7 días antes, al ser diagnosticado
por su médico de cabecera de sinisitis (opacificación de
ambos senos maxilares), sin obtener una mejoría
clínica. La Rx de tórax presenta múltiples nódulos
pulmonares bilaterales, algunos de ellos cavitados. En
los análisis efectuados destaca un sedimento de orina
con 8 hematíes por campo con algún cilindro
eritrocitario. La biopsia de la mucosa nasal mostró
inflamación granulomatosa con necrosis. El diagnóstico
más probable es:
A) Granulomatosis de Wegener.
B) Granulomatosis de Churg-Strauss.
C) Cáncer de cavum con metástasis pulmonares.
D) Granulomatosis linfomatoide.
E) Tuberculosis.
73. Un grave problema de las unidades de cuidados
intensivos son las infecciones (neumonías y sepsis) por
gérmenes gramnegativos multirresistentes
(pseudomonas, serratias, citrobacter, morganella,
acinetobacter, etc.). Ya se han identificado previamente
cepas multirresistentes en nuestra UCI. A falta de un
antibiograma, ¿cuál sería el tratamiento empírico de
elección?
A) Ceftazidima, amicamicina y vancomicina.
B) Ceftriaxona y tobramicina.
C) Imipenem o ciprofloxacino.
D) Imipenem y amikamicina.
E) Esperar hasta los resultados del antibiograma.
74. Un varón de 30 años, fumador de 20 cigarrillos/día
desde los 20 a los 25 años y ex fumador desde
entonces, presenta, en un reconocimiento laboral, un
nódulo pulmonar solitario (NPS) de unos 2 cm. de
diametro en la periferia del LSD. La Rx de tórax muestra
un mediastino normal y no permite identificar
calcificaciones en el NPS. El paciente se encuentra
asintomático y niega la posibilidad de recuperar
radiografías anteriores antes de 6 meses (por cambio de
domicilio). ¿Qué actitud mantendría ante este enfermo?
A) Informar de la baja probabilidad de malignidad y Rx
de tórax en 3 meses.
B) Fibrobroncoscopia.
C) Realización de una TAC torácica.
D) Realización preferente de una PAAF con control de
TAC.
E) Insistir en la recuperación de las Rx previas y nueva
cita en la consulta entonces (6 meses).
75. En el caso anterior, la TAC no muestra nuevos datos
(confirma la ausencia de calcificaciones, no adenopatías
ni afectación mediastínica y no existe afectación
pleural). La familia del enfermo ha localizado las Rx de
tórax previas (14 meses antes) en la que se identifica el
mismo NPS con un diámetro de 1,4 cm. ¿Cuál sería la
actitud más adecuada?
A) Actitud expectante y repetir pruebas de imagen en 2
meses.
B) Fibrobroncoscopia con citología en las muestras
obtenidas.
C) Fibrobroncoscopia y biopsia transbronquial (BTB).
D) PAAF con control de TAC.
E) Toracotomía.
76. En caso de que la PAAF obtenga material suficiente
y el resultado sea de malignidad, ¿qué tipo histológico le
parece el más probable?
A) Ca. epidermoide.
B) Adenocarcinoma.
C) Ca células pequeñas (CCP).
D) Ca. bronquioalveolar.
E) Carcinoide.
77. Paciente de 36 años con amenorrea de 10 semanas.
Tiene un antecedente de infertilidad por factor tubárico.
Refiere episodios de dolor cólico hipogástrico desde
hace aproximadamente un mes. El test de embarazo en
orina es positivo.
La prueba diagnóstica que solicita a continuación es:
A) Amniocentesis precoz.
B) Ecografía.
C) Laparoscopia.
D) Triple screening.
E) Biopsia de vellosidades coriónicas.
Examen Simulacro :: Ciclo de Revisión en Medicina :: Sábado 4 de Agosto del 2012 :: www.estudiosmyc.com
78. Gestante de 16 semanas sin antecedentes de
interés que presenta los siguientes resultados en la
analítica de triple screening: alfafetoproteína 0,3 MM
(disminuido), betahcg 1,7 MM (normal), riesgo estimado
de T21 1/43. A continuación se le realiza:
A) Ecografía.
B) Amniocentesis.
C) Biopsia de vellosidades coriónicas.
D) Funiculocentesis.
E) Fetoscopia.
79. Gestante de 26 semanas que consulta por fiebre de
39° C y dolor lumbar unilateral. La analítica de sangre
presenta 16.000 leucocitos y desviación izquierda. El
tratamiento indicado es:
A) Abundante ingesta de líquidos.
B) Analgésicos orales y abundante ingesta de líquidos.
C) Analgésicos endovenosos y forzar diuresis.
D) Antibióticos orales y reposo domiciliario.
E) Antibióticos endovenosos intrahospitalarios.
80. Gestante de 30 semanas con aumento excesivo de
peso (ganancia de 18 kg. hasta la actualidad) a la que
se realiza un test de O’Sullivan que resulta patológico.
La actitud médica ha de ser a continuación:
A) Vigilancia fetal estricta, con registros semanales de la
frecuencia cardíaca fetal.
B) Controles de glucemia capilar (BMtest) en desayuno,
comida y cena.
C) Ecografías seriadas para diagnosticar a la mayor
brevedad posible un hidramnios o macrosomía fetal.
D) Confirmar el diagnóstico mediante una prueba de
tolerancia oral a la glucosa.
E) Tratar con insulina rápida según los resultados del
test de O’Sullivan.
81. Gestante de 9 semanas que consulta por
metrorragia menor que una regla y dolor abdominal.
Todavía no ha acudido a ninguna visita de control por su
tocólogo. En la exploración se obseva un útero de
aproximadamente 8 semanas de gestación, abdomen
blando y depresible y cérvix permeable a un dedo. Poco
después la paciente empieza a sangrar
abundantemente, mucho más que una regla. El
tratamiento indicado es:
A) Ingreso y observación.
B) Laparoscopia.
C) Legrado.
D) Laparotomía.
E) Venoclisis de oxitocina.
82. Primigesta de 27 años, sin antecedentes médicos de
interés, que consulta por metrorragia insidiosa y
recurrente en semana 36 de embarazo. No presenta
dolor abdominal, el útero está relajado y el latido fetal se
escucha vigoroso.
El diagnóstico más probable será:
A) Vasa praevia.
B) Desprendimiento prematuro de placenta.
C) Expulsión del tapón mucoso.
D) Placenta previa.
E) Pérdida de líquido amniótico hemático.
83. Tercigesta isoinmunizada que presenta test de
Coombs indirecto de 1/10 en semana 30 de embarazo.
Se realiza un amniocentesis en que se determina la
madurez fetal (se confirma la presencia de fosfatidil
glicerol en líquido amniótico) y el nomograma de Liley,
que se encuentra en la zona II. La actitud indicada es:
A) Nueva amniocentesis en una semana.
B) Seguimiento ecográfico con la paciente hospitalizada.
C) Administración de corticoides para inducir la madurez
fetal.
D) Extracción fetal.
E) Administración endovenosa materna de IgG contra
Ac anti D.
84. Gestante de 33 semanas que consulta por dinámica
uterina, disminución de movimientos fetales y febrícula.
Al ingreso presenta una analítica de sangre con
leucocitosis moderada y el resto de los parámetros
normales. En la
ecografìa se observa un oligoamnios y un perfil biofísico
fetal de 9. El tratamiento consiste en:
A) Antibióticos endovenosos.
B) Antibióticos intracavitarios.
C) Inducción del parto.
D) Cultivo del líquido amniótico y tratamiento según
antibiograma.
E) Administración de corticoides para favorecer la
madurez fetal e inducción del parto en semana 36.
85. Gestante de embarazo gemelar que presenta
dinámica espontánea en semana 36. Ambos gemelos se
encuentran en presentación cefálica. El parto del
primero se produce sin complicaciones, pero el segundo
se encuentra en posición occipito iliaca derecha
transversa al cabo de 25 minutos desde el nacimiento
del primero, sin progresar desde un III plano de Hodge.
Para finalizar el parto está indicada la realización de:
A) Vacuum.
B) Fórceps.
C) Cesarea.
D) Maniobra de Kristeller.
E) Cualquiera de las anteriores según el estado fetal.
86. Primigesta de 36 años que presenta un aumento de
la tensión diastólica de 30 mmHg respecto a tomas
iniciales, y albuminuria con edemas generalizados.
Actualmente se encuentra en la 34 semana de
embarazo. Súbitamente inicia un cuadro de cefalea y
transtornos visuales. Vd. le inicia tratamiento médico
con:
A) Nifedipina.
B) Dihidralacina.
C) Alfametildopa.
D) Sulfato de magnesio.
E) Labetalol.
Examen Simulacro :: Ciclo de Revisión en Medicina :: Sábado 4 de Agosto del 2012 :: www.estudiosmyc.com
87. En la paciente del caso anterior el estudio fetal
descubre un feto afecto de crecimiento intrauterino
retardado, en el que la relación entre el área cefálica y el
área abdominal es mayor de uno. El feto se encuentra
en situación cefálica.
La conducta obstétrica adecuada es:
A) Expectante, con parto vaginal como via de elección.
B) Controles de bienestar fetal (perfil biofísico)
semanales hasta la semana 40.
C) Valoración de la madurez fetal e inducción del parto,
si el feto es maduro.
D) Cesárea inmediata.
E) Evaluación de la funcionalidad placentaria mediante
ecografía Doppler.
88. Puérpera que acaba de parir mediante parto
eutócico un varón de 3,450 kg. a los 30 minutos no ha
alumbrado, por lo que se realiza una maniobra de Credé
para extraer la placenta. Transcurridos unos minutos
presenta un cuadro de disnea y hemorragia profusa. El
útero está bien contraído, pero el sangrado no cesa. El
diagnóstico más probable es:
A) Ruptura uterina.
B) Desgarro de cérvix.
C) Hipotonía uterina.
D) Retención de restos placentarios.
E) Coagulación intravascular diseminada.
89. Un paciente de 26 años de edad, adicto a drogas
por vía parenteral, consulta por malestar general, fiebre
de 39°con tiritona, escalofríos y dolor y tumefacción en
rodilla derecha. En la exploración llama la atención un
soplo cardíco panfocal que previamente no estaba en la
historia del enfermo y artritis de rodilla derecha. No se
pudo realizar artrocentesis diagnóstica. La cobertura
empírica antibiótica más segura sería:
A) Vancomicina.
B) Vancomicina y Gentamicina.
C) Ciprofloxacina.
D) Eritromicina y cefuloxima.
E) Ceftriaxona.
90. Un joven de 18 años sin antecedentes de interés,
consulta por tos, fiebre y otalgia. En la Rx de tórax
presenta infiltrado intersticial derecho. La exploración
ORL demuestra miringitis ampollosa. Iniciaríamos
tratamiento con:
A) Ceftriaxona.
B) Vancomicina.
C) Norfloxacina.
D) Isoniazida, rifampicina y pirazinamida.
E) Eritromicina.
91. Un paciente al que le detecta una infección urinaria
por Pseudomona inicia tratamiento con ceftacidima.
Tras 2 días de tratamiento el paciente comienza a
encontrarse peor, la fiebre aumenta hasta los 39° y
desarrolla hipotensión. En el hemograma destaca
leucopenia e importante trombopenia que previamente
no estaban. Bioquímicamente presenta hiperglucemia
de 198 g./dl. Probablemente nos encontramos ante:
A) Efecto secundario de la ceftacidima.
B) Evolución natural del proceso.
C) Situación de shock séptico.
D) Probable asociación de una hemopatía.
E) Cetoacidosis diabética.
92. Un varón de 27 años consulta por presenta en la
región balanoprepucial una úlcera de 1 cm. de tamaño
de borde indurado, no doloroso y que secreta serosidad.
Presenta adicionalmente adenopatías inguinales
bilaterales y fiebre. Estaríamos obligados a solicitarle en
el estudio:
A) Aglutinaciones a salmonella.
B) Aglutinaciones a brucella.
C) Serología de VIH.
D) Serología luética.
E) Serología de micoplasma.
93. Una paciente de 19 años ha sido diagnosticada de
mononucleosis infecciosa, confirmada mediante Paul-
Bunnel. Ha sido tratada con paracetamol a dosis de 2,5
g./día. A los cinco días del diagnóstico comienza de
forma brusca con un cuadro de abdomen agudo y
shock. El proceso más probable es:
A) Rotura espontánea del bazo.
B) Hepatitis por virus de Epstein-Barr.
C) Complicación infecciosa intraabdominal.
D) Complicación del tratamiento.
E) Otra cualquier causa de abdomen agudo: p. ej.
apendicitis aguda.
94. Un lactante de 7 meses de edad, con Tetralogía de
Fallot intervenida, desarrolla un cuadro de dificultad
respiratoria, sibilancias y tos. No se termometra fiebre.
Además de las medidas de soporte, el tratamiento de
elección sería:
A) Reintervención quirúrgica de su cardiopatía.
B) Esteroides a dosis plenas.
C) Ribavirina en aerosol.
D) Eritromicina intravenosa.
E) Ceftriaxona intravenosa.
95. Una paciente de 49 años consulta en un Servicio de
Urgencias por dolor, enrojecimiento y tumefacción de la
parte distal de su miembro inferior derecho. Por
sospecha de trombosis venosa profunda se le realiza
flebografía que resulta ser negativa. La actitud
terapéutica a seguir es:
A) Tratamiento antiinflamatorio.
B) Heparina de bajo peso molecular.
C) Cobertura empírica antibiótica con Oxacilina.
D) Reposo del miembro sin más.
E) Repetir flebografía pasadas 48 horas.
Examen Simulacro :: Ciclo de Revisión en Medicina :: Sábado 4 de Agosto del 2012 :: www.estudiosmyc.com
96. Un adicto a drogas por vía parenteral consulta por
tumoración fluctuante y con signos inflamatorios en la
flexura del codo donde ha realizado inyecciones
intravenosas. Se debe realizar:
A) Tratamiento quirúrgico de drenaje.
B) Cobertura con ceftriaxona.
C) Cobertura con vancomicina.
D) Cobertura con Oxacilina.
E) Medidas antiinflamatorias.
97. Una pareja de turistas que han regresado a Lima,
luego de estar en la Selva Central del Perú, comienzan
a presentar deposiciones en cantidad abundante de
características líquidas, similares al agua, en número de
20-25 al día. No se acompaña de dolor abdominal ni
fiebre. El cuadro obliga a una rehidratación y ésta se
consigue con dificultad dada la gran pérdida de agua y
electrólitos. El cuadro clínico más probable es:
A) Gastroenteritis por Salmonella.
B) Disentería bacilar.
C) Disentería amebiana.
D) Cólera.
E) Gastroenteritis viral.
98. Un enfermo leucémico, muy inmunodeprimido,
desarrolla tras un tratamiento antibacteriano de amplio
espectro, cuadro importante de insuficiencia respiratoria
y fiebre. En la Rx. de tórax se aprecia una masa densa,
cubierta por un menisco delgado de aire en el interior de
una cavidad. El diagnóstico más probable es:
A) Tuberculosis.
B) Neumonía bacteriana.
C) Masa tumoral sobreinfectada.
D) Neumonía por cándidas.
E) Aspergiloma.
99. Un niño de 3 años desarrolla un cuadro de infección
respiratoria de vías altas con fiebre elevada.
Posteriormente aparece tos paroxística con gallo
inspiratorio. El tratamiento de elección sería:
A) Ceftriaxona.
B) Cefonicid.
C) Cefalotina.
D) Eritromicina.
E) Amoxicilina-clavulánico.
100. Un paciente de 56 años acude a su médico de
cabecera por presentar en el último mes fiebre diaria,
con una distribución de dos picos, matutino y vespertino.
A la exploración llama la atención una
hepatoesplenomegalia muy importante. No se objetivan
adenopatías a ningún nivel. El hemograma muestra
pancitopenia. La prueba diagnóstica que se debería
realizar sería:
A) Estudio de médula ósea.
B) Ecografía abdominal.
C) Marcadores tumorales.
D) Biopsia hepática.
E) Marcadores de hepatitis.
Examen_Simulacro_Tema_A_y_B_Sabado_4_de.pdf
Examen_Simulacro_Tema_A_y_B_Sabado_4_de.pdf
Examen_Simulacro_Tema_A_y_B_Sabado_4_de.pdf
Examen_Simulacro_Tema_A_y_B_Sabado_4_de.pdf
Examen_Simulacro_Tema_A_y_B_Sabado_4_de.pdf
Examen_Simulacro_Tema_A_y_B_Sabado_4_de.pdf
Examen_Simulacro_Tema_A_y_B_Sabado_4_de.pdf
Examen_Simulacro_Tema_A_y_B_Sabado_4_de.pdf
Examen_Simulacro_Tema_A_y_B_Sabado_4_de.pdf
Examen_Simulacro_Tema_A_y_B_Sabado_4_de.pdf
Examen_Simulacro_Tema_A_y_B_Sabado_4_de.pdf
Examen_Simulacro_Tema_A_y_B_Sabado_4_de.pdf
Examen_Simulacro_Tema_A_y_B_Sabado_4_de.pdf
Examen_Simulacro_Tema_A_y_B_Sabado_4_de.pdf
Examen_Simulacro_Tema_A_y_B_Sabado_4_de.pdf
Examen_Simulacro_Tema_A_y_B_Sabado_4_de.pdf
Examen_Simulacro_Tema_A_y_B_Sabado_4_de.pdf
Examen_Simulacro_Tema_A_y_B_Sabado_4_de.pdf
Examen_Simulacro_Tema_A_y_B_Sabado_4_de.pdf
Examen_Simulacro_Tema_A_y_B_Sabado_4_de.pdf
Examen_Simulacro_Tema_A_y_B_Sabado_4_de.pdf

Más contenido relacionado

Similar a Examen_Simulacro_Tema_A_y_B_Sabado_4_de.pdf

100615 final-examen-parte-a-estudios myc
100615 final-examen-parte-a-estudios myc100615 final-examen-parte-a-estudios myc
100615 final-examen-parte-a-estudios mycehpt
 
I TALLER CORRELACIÓN CLINICA
I TALLER CORRELACIÓN CLINICAI TALLER CORRELACIÓN CLINICA
I TALLER CORRELACIÓN CLINICAfaquintero
 
I Taller Correlación Clínica
I Taller Correlación ClínicaI Taller Correlación Clínica
I Taller Correlación Clínicafaquintero
 
BANCO MEDICINA INTERNA PATOLOGÍA INFECCIOSA - Estudios M y C.pdf
BANCO MEDICINA INTERNA PATOLOGÍA INFECCIOSA - Estudios M y C.pdfBANCO MEDICINA INTERNA PATOLOGÍA INFECCIOSA - Estudios M y C.pdf
BANCO MEDICINA INTERNA PATOLOGÍA INFECCIOSA - Estudios M y C.pdfEdwinRivera106990
 
BANCO MEDICINA INTERNA PATOLOGÍA INFECCIOSA - Estudios M y C.pdf
BANCO MEDICINA INTERNA PATOLOGÍA INFECCIOSA - Estudios M y C.pdfBANCO MEDICINA INTERNA PATOLOGÍA INFECCIOSA - Estudios M y C.pdf
BANCO MEDICINA INTERNA PATOLOGÍA INFECCIOSA - Estudios M y C.pdfSheila Martino T
 
ENAM 2021 - 20 DE MARZO.pdf
ENAM 2021 - 20 DE MARZO.pdfENAM 2021 - 20 DE MARZO.pdf
ENAM 2021 - 20 DE MARZO.pdfEdward leyva
 
30788305 preguntas-medicina-interna
30788305 preguntas-medicina-interna30788305 preguntas-medicina-interna
30788305 preguntas-medicina-internaxixel britos
 
Caso clinico para entregar
Caso clinico para entregarCaso clinico para entregar
Caso clinico para entregarJdso Otero
 
Examen rmn 2019.docx(2)
Examen   rmn 2019.docx(2)Examen   rmn 2019.docx(2)
Examen rmn 2019.docx(2)PaolaSusana1
 
repaso-salud-publica-epidemiologia-bioestadistica-parte-2.pdf
repaso-salud-publica-epidemiologia-bioestadistica-parte-2.pdfrepaso-salud-publica-epidemiologia-bioestadistica-parte-2.pdf
repaso-salud-publica-epidemiologia-bioestadistica-parte-2.pdfJACKDAIVISLAIZAREYES
 
Respuestas mopece 2 y 3 (isaac trejo)
Respuestas mopece 2 y 3 (isaac trejo)Respuestas mopece 2 y 3 (isaac trejo)
Respuestas mopece 2 y 3 (isaac trejo)Isaac Trejo
 

Similar a Examen_Simulacro_Tema_A_y_B_Sabado_4_de.pdf (20)

100615 final-examen-parte-a-estudios myc
100615 final-examen-parte-a-estudios myc100615 final-examen-parte-a-estudios myc
100615 final-examen-parte-a-estudios myc
 
I TALLER CORRELACIÓN CLINICA
I TALLER CORRELACIÓN CLINICAI TALLER CORRELACIÓN CLINICA
I TALLER CORRELACIÓN CLINICA
 
I Taller Correlación Clínica
I Taller Correlación ClínicaI Taller Correlación Clínica
I Taller Correlación Clínica
 
BANCO MEDICINA INTERNA PATOLOGÍA INFECCIOSA - Estudios M y C.pdf
BANCO MEDICINA INTERNA PATOLOGÍA INFECCIOSA - Estudios M y C.pdfBANCO MEDICINA INTERNA PATOLOGÍA INFECCIOSA - Estudios M y C.pdf
BANCO MEDICINA INTERNA PATOLOGÍA INFECCIOSA - Estudios M y C.pdf
 
BANCO MEDICINA INTERNA PATOLOGÍA INFECCIOSA - Estudios M y C.pdf
BANCO MEDICINA INTERNA PATOLOGÍA INFECCIOSA - Estudios M y C.pdfBANCO MEDICINA INTERNA PATOLOGÍA INFECCIOSA - Estudios M y C.pdf
BANCO MEDICINA INTERNA PATOLOGÍA INFECCIOSA - Estudios M y C.pdf
 
Enfermería SES 2008 - Supuesto 1
Enfermería SES 2008 - Supuesto 1Enfermería SES 2008 - Supuesto 1
Enfermería SES 2008 - Supuesto 1
 
Examen RM 2014B
Examen RM 2014BExamen RM 2014B
Examen RM 2014B
 
Examen 1 de_la_salle
Examen 1 de_la_salleExamen 1 de_la_salle
Examen 1 de_la_salle
 
ENAM 2021 - 20 DE MARZO.pdf
ENAM 2021 - 20 DE MARZO.pdfENAM 2021 - 20 DE MARZO.pdf
ENAM 2021 - 20 DE MARZO.pdf
 
30788305 preguntas-medicina-interna
30788305 preguntas-medicina-interna30788305 preguntas-medicina-interna
30788305 preguntas-medicina-interna
 
Caso clinico para entregar
Caso clinico para entregarCaso clinico para entregar
Caso clinico para entregar
 
Examen RM 2014A
Examen RM 2014AExamen RM 2014A
Examen RM 2014A
 
PPT-REUMATOLOGIA-II-PR.pdf
PPT-REUMATOLOGIA-II-PR.pdfPPT-REUMATOLOGIA-II-PR.pdf
PPT-REUMATOLOGIA-II-PR.pdf
 
PPT-INFECTOLOGIA-I.pdf
PPT-INFECTOLOGIA-I.pdfPPT-INFECTOLOGIA-I.pdf
PPT-INFECTOLOGIA-I.pdf
 
Examen rmn 2019.docx(2)
Examen   rmn 2019.docx(2)Examen   rmn 2019.docx(2)
Examen rmn 2019.docx(2)
 
Taller busquedas sistemicas mbe
Taller busquedas sistemicas mbeTaller busquedas sistemicas mbe
Taller busquedas sistemicas mbe
 
2015
20152015
2015
 
repaso-salud-publica-epidemiologia-bioestadistica-parte-2.pdf
repaso-salud-publica-epidemiologia-bioestadistica-parte-2.pdfrepaso-salud-publica-epidemiologia-bioestadistica-parte-2.pdf
repaso-salud-publica-epidemiologia-bioestadistica-parte-2.pdf
 
Simulacro 1.pdf
Simulacro 1.pdfSimulacro 1.pdf
Simulacro 1.pdf
 
Respuestas mopece 2 y 3 (isaac trejo)
Respuestas mopece 2 y 3 (isaac trejo)Respuestas mopece 2 y 3 (isaac trejo)
Respuestas mopece 2 y 3 (isaac trejo)
 

Más de IsusKonIntiAi

Desgloses_CTO_MIR_1995_2005 (1).pdf
Desgloses_CTO_MIR_1995_2005 (1).pdfDesgloses_CTO_MIR_1995_2005 (1).pdf
Desgloses_CTO_MIR_1995_2005 (1).pdfIsusKonIntiAi
 
EXAMEN_COMENTADO_DE_RESIDENCIAS_MEDICAS (1).pdf
EXAMEN_COMENTADO_DE_RESIDENCIAS_MEDICAS (1).pdfEXAMEN_COMENTADO_DE_RESIDENCIAS_MEDICAS (1).pdf
EXAMEN_COMENTADO_DE_RESIDENCIAS_MEDICAS (1).pdfIsusKonIntiAi
 
GUIA_TOTAL_DE_ANATOMIA_1.docx
GUIA_TOTAL_DE_ANATOMIA_1.docxGUIA_TOTAL_DE_ANATOMIA_1.docx
GUIA_TOTAL_DE_ANATOMIA_1.docxIsusKonIntiAi
 
CUESTIONARIO_EXAMEN_TITULACION (1).docx
CUESTIONARIO_EXAMEN_TITULACION (1).docxCUESTIONARIO_EXAMEN_TITULACION (1).docx
CUESTIONARIO_EXAMEN_TITULACION (1).docxIsusKonIntiAi
 
banco-de-preguntas-de-neurologia-72891-downloable-2768928 (1).pdf
banco-de-preguntas-de-neurologia-72891-downloable-2768928 (1).pdfbanco-de-preguntas-de-neurologia-72891-downloable-2768928 (1).pdf
banco-de-preguntas-de-neurologia-72891-downloable-2768928 (1).pdfIsusKonIntiAi
 
banco-de-preguntas-semiologia-grupo-8-284827-downloable-2768928.pdf
banco-de-preguntas-semiologia-grupo-8-284827-downloable-2768928.pdfbanco-de-preguntas-semiologia-grupo-8-284827-downloable-2768928.pdf
banco-de-preguntas-semiologia-grupo-8-284827-downloable-2768928.pdfIsusKonIntiAi
 

Más de IsusKonIntiAi (6)

Desgloses_CTO_MIR_1995_2005 (1).pdf
Desgloses_CTO_MIR_1995_2005 (1).pdfDesgloses_CTO_MIR_1995_2005 (1).pdf
Desgloses_CTO_MIR_1995_2005 (1).pdf
 
EXAMEN_COMENTADO_DE_RESIDENCIAS_MEDICAS (1).pdf
EXAMEN_COMENTADO_DE_RESIDENCIAS_MEDICAS (1).pdfEXAMEN_COMENTADO_DE_RESIDENCIAS_MEDICAS (1).pdf
EXAMEN_COMENTADO_DE_RESIDENCIAS_MEDICAS (1).pdf
 
GUIA_TOTAL_DE_ANATOMIA_1.docx
GUIA_TOTAL_DE_ANATOMIA_1.docxGUIA_TOTAL_DE_ANATOMIA_1.docx
GUIA_TOTAL_DE_ANATOMIA_1.docx
 
CUESTIONARIO_EXAMEN_TITULACION (1).docx
CUESTIONARIO_EXAMEN_TITULACION (1).docxCUESTIONARIO_EXAMEN_TITULACION (1).docx
CUESTIONARIO_EXAMEN_TITULACION (1).docx
 
banco-de-preguntas-de-neurologia-72891-downloable-2768928 (1).pdf
banco-de-preguntas-de-neurologia-72891-downloable-2768928 (1).pdfbanco-de-preguntas-de-neurologia-72891-downloable-2768928 (1).pdf
banco-de-preguntas-de-neurologia-72891-downloable-2768928 (1).pdf
 
banco-de-preguntas-semiologia-grupo-8-284827-downloable-2768928.pdf
banco-de-preguntas-semiologia-grupo-8-284827-downloable-2768928.pdfbanco-de-preguntas-semiologia-grupo-8-284827-downloable-2768928.pdf
banco-de-preguntas-semiologia-grupo-8-284827-downloable-2768928.pdf
 

Último

PPT HIS PROMSA - PANAS-MINSA DEL 2024.pptx
PPT HIS PROMSA - PANAS-MINSA DEL 2024.pptxPPT HIS PROMSA - PANAS-MINSA DEL 2024.pptx
PPT HIS PROMSA - PANAS-MINSA DEL 2024.pptxOrlandoApazagomez1
 
AFERESIS TERAPEUTICA para el personal médico
AFERESIS TERAPEUTICA para el personal médicoAFERESIS TERAPEUTICA para el personal médico
AFERESIS TERAPEUTICA para el personal médicoGabrielMontalvo19
 
Torax normal-Oscar 2024- principios físicos del rx de torax
Torax normal-Oscar 2024- principios físicos del rx de toraxTorax normal-Oscar 2024- principios físicos del rx de torax
Torax normal-Oscar 2024- principios físicos del rx de toraxWillianEduardoMascar
 
Trombocitopenia Inmune primaria , clínica
Trombocitopenia Inmune primaria , clínicaTrombocitopenia Inmune primaria , clínica
Trombocitopenia Inmune primaria , clínicaVillegasValentnJosAl
 
6.METODOLOGIA ATENEA MICHAEL. ZAPATA.pdf
6.METODOLOGIA ATENEA MICHAEL. ZAPATA.pdf6.METODOLOGIA ATENEA MICHAEL. ZAPATA.pdf
6.METODOLOGIA ATENEA MICHAEL. ZAPATA.pdfbibianavillazoo
 
TRANSMISION DE LA INFORMACIÓN GENETICA - Clase 1.pptx
TRANSMISION DE LA INFORMACIÓN GENETICA - Clase 1.pptxTRANSMISION DE LA INFORMACIÓN GENETICA - Clase 1.pptx
TRANSMISION DE LA INFORMACIÓN GENETICA - Clase 1.pptxJoshueXavierE
 
Posiciones anatomicas basicas enfermeria
Posiciones anatomicas basicas enfermeriaPosiciones anatomicas basicas enfermeria
Posiciones anatomicas basicas enfermeriaKarymeScarlettAguila
 
Cuadro-comparativo-Aparato-Reproductor-Masculino-y-Femenino.pptx
Cuadro-comparativo-Aparato-Reproductor-Masculino-y-Femenino.pptxCuadro-comparativo-Aparato-Reproductor-Masculino-y-Femenino.pptx
Cuadro-comparativo-Aparato-Reproductor-Masculino-y-Femenino.pptxguadalupedejesusrios
 
CONTROL DE CALIDAD EN LA INDUSTRIA FARMACEUTICA
CONTROL DE CALIDAD EN LA INDUSTRIA FARMACEUTICACONTROL DE CALIDAD EN LA INDUSTRIA FARMACEUTICA
CONTROL DE CALIDAD EN LA INDUSTRIA FARMACEUTICAmjaicocr
 
WE DO TRANSFORMATIONS DAY presentación .pptx
WE DO TRANSFORMATIONS DAY presentación   .pptxWE DO TRANSFORMATIONS DAY presentación   .pptx
WE DO TRANSFORMATIONS DAY presentación .pptxr7dzcbmq2w
 
Plan de Desparasitacion 27.03.2024 minsa.pptx
Plan de Desparasitacion 27.03.2024 minsa.pptxPlan de Desparasitacion 27.03.2024 minsa.pptx
Plan de Desparasitacion 27.03.2024 minsa.pptxOrlandoApazagomez1
 
Celulas del sistema nervioso clase medicina
Celulas del sistema nervioso clase medicinaCelulas del sistema nervioso clase medicina
Celulas del sistema nervioso clase medicinaSalomeLoor1
 
Clase 15 Artrologia mmii 1 de 3 (Cintura Pelvica y Cadera) 2024.pdf
Clase 15 Artrologia mmii 1 de 3 (Cintura Pelvica y Cadera) 2024.pdfClase 15 Artrologia mmii 1 de 3 (Cintura Pelvica y Cadera) 2024.pdf
Clase 15 Artrologia mmii 1 de 3 (Cintura Pelvica y Cadera) 2024.pdfgarrotamara01
 
2.INJERTO Y COLGAJO CUIDADOS DE ENFERMERIA
2.INJERTO Y COLGAJO CUIDADOS DE ENFERMERIA2.INJERTO Y COLGAJO CUIDADOS DE ENFERMERIA
2.INJERTO Y COLGAJO CUIDADOS DE ENFERMERIADiegoOliveiraEspinoz1
 
Patologías de los eritrocitos-Histologia
Patologías de los eritrocitos-HistologiaPatologías de los eritrocitos-Histologia
Patologías de los eritrocitos-Histologia Estefa RM9
 
(2024-25-04) Epilepsia, manejo el urgencias (ptt).pptx
(2024-25-04) Epilepsia, manejo el urgencias (ptt).pptx(2024-25-04) Epilepsia, manejo el urgencias (ptt).pptx
(2024-25-04) Epilepsia, manejo el urgencias (ptt).pptxUDMAFyC SECTOR ZARAGOZA II
 
CLASE DE VIH/ETS - UNAN 2024 PEDIATRIA I
CLASE DE VIH/ETS - UNAN 2024 PEDIATRIA ICLASE DE VIH/ETS - UNAN 2024 PEDIATRIA I
CLASE DE VIH/ETS - UNAN 2024 PEDIATRIA ILucy López
 
Mapa-conceptual-del-Sistema-Circulatorio-2.pptx
Mapa-conceptual-del-Sistema-Circulatorio-2.pptxMapa-conceptual-del-Sistema-Circulatorio-2.pptx
Mapa-conceptual-del-Sistema-Circulatorio-2.pptxJhonDarwinSnchezVsqu1
 
Sistema Nervioso Periférico (1).pdf
Sistema Nervioso Periférico      (1).pdfSistema Nervioso Periférico      (1).pdf
Sistema Nervioso Periférico (1).pdfNjeraMatas
 

Último (20)

PPT HIS PROMSA - PANAS-MINSA DEL 2024.pptx
PPT HIS PROMSA - PANAS-MINSA DEL 2024.pptxPPT HIS PROMSA - PANAS-MINSA DEL 2024.pptx
PPT HIS PROMSA - PANAS-MINSA DEL 2024.pptx
 
AFERESIS TERAPEUTICA para el personal médico
AFERESIS TERAPEUTICA para el personal médicoAFERESIS TERAPEUTICA para el personal médico
AFERESIS TERAPEUTICA para el personal médico
 
Torax normal-Oscar 2024- principios físicos del rx de torax
Torax normal-Oscar 2024- principios físicos del rx de toraxTorax normal-Oscar 2024- principios físicos del rx de torax
Torax normal-Oscar 2024- principios físicos del rx de torax
 
Trombocitopenia Inmune primaria , clínica
Trombocitopenia Inmune primaria , clínicaTrombocitopenia Inmune primaria , clínica
Trombocitopenia Inmune primaria , clínica
 
6.METODOLOGIA ATENEA MICHAEL. ZAPATA.pdf
6.METODOLOGIA ATENEA MICHAEL. ZAPATA.pdf6.METODOLOGIA ATENEA MICHAEL. ZAPATA.pdf
6.METODOLOGIA ATENEA MICHAEL. ZAPATA.pdf
 
TRANSMISION DE LA INFORMACIÓN GENETICA - Clase 1.pptx
TRANSMISION DE LA INFORMACIÓN GENETICA - Clase 1.pptxTRANSMISION DE LA INFORMACIÓN GENETICA - Clase 1.pptx
TRANSMISION DE LA INFORMACIÓN GENETICA - Clase 1.pptx
 
Posiciones anatomicas basicas enfermeria
Posiciones anatomicas basicas enfermeriaPosiciones anatomicas basicas enfermeria
Posiciones anatomicas basicas enfermeria
 
Cuadro-comparativo-Aparato-Reproductor-Masculino-y-Femenino.pptx
Cuadro-comparativo-Aparato-Reproductor-Masculino-y-Femenino.pptxCuadro-comparativo-Aparato-Reproductor-Masculino-y-Femenino.pptx
Cuadro-comparativo-Aparato-Reproductor-Masculino-y-Femenino.pptx
 
CONTROL DE CALIDAD EN LA INDUSTRIA FARMACEUTICA
CONTROL DE CALIDAD EN LA INDUSTRIA FARMACEUTICACONTROL DE CALIDAD EN LA INDUSTRIA FARMACEUTICA
CONTROL DE CALIDAD EN LA INDUSTRIA FARMACEUTICA
 
WE DO TRANSFORMATIONS DAY presentación .pptx
WE DO TRANSFORMATIONS DAY presentación   .pptxWE DO TRANSFORMATIONS DAY presentación   .pptx
WE DO TRANSFORMATIONS DAY presentación .pptx
 
Plan de Desparasitacion 27.03.2024 minsa.pptx
Plan de Desparasitacion 27.03.2024 minsa.pptxPlan de Desparasitacion 27.03.2024 minsa.pptx
Plan de Desparasitacion 27.03.2024 minsa.pptx
 
Celulas del sistema nervioso clase medicina
Celulas del sistema nervioso clase medicinaCelulas del sistema nervioso clase medicina
Celulas del sistema nervioso clase medicina
 
Clase 15 Artrologia mmii 1 de 3 (Cintura Pelvica y Cadera) 2024.pdf
Clase 15 Artrologia mmii 1 de 3 (Cintura Pelvica y Cadera) 2024.pdfClase 15 Artrologia mmii 1 de 3 (Cintura Pelvica y Cadera) 2024.pdf
Clase 15 Artrologia mmii 1 de 3 (Cintura Pelvica y Cadera) 2024.pdf
 
2.INJERTO Y COLGAJO CUIDADOS DE ENFERMERIA
2.INJERTO Y COLGAJO CUIDADOS DE ENFERMERIA2.INJERTO Y COLGAJO CUIDADOS DE ENFERMERIA
2.INJERTO Y COLGAJO CUIDADOS DE ENFERMERIA
 
Patologías de los eritrocitos-Histologia
Patologías de los eritrocitos-HistologiaPatologías de los eritrocitos-Histologia
Patologías de los eritrocitos-Histologia
 
(2024-25-04) Epilepsia, manejo el urgencias (ptt).pptx
(2024-25-04) Epilepsia, manejo el urgencias (ptt).pptx(2024-25-04) Epilepsia, manejo el urgencias (ptt).pptx
(2024-25-04) Epilepsia, manejo el urgencias (ptt).pptx
 
Transparencia Fiscal HJPII Marzo 2024
Transparencia  Fiscal  HJPII  Marzo 2024Transparencia  Fiscal  HJPII  Marzo 2024
Transparencia Fiscal HJPII Marzo 2024
 
CLASE DE VIH/ETS - UNAN 2024 PEDIATRIA I
CLASE DE VIH/ETS - UNAN 2024 PEDIATRIA ICLASE DE VIH/ETS - UNAN 2024 PEDIATRIA I
CLASE DE VIH/ETS - UNAN 2024 PEDIATRIA I
 
Mapa-conceptual-del-Sistema-Circulatorio-2.pptx
Mapa-conceptual-del-Sistema-Circulatorio-2.pptxMapa-conceptual-del-Sistema-Circulatorio-2.pptx
Mapa-conceptual-del-Sistema-Circulatorio-2.pptx
 
Sistema Nervioso Periférico (1).pdf
Sistema Nervioso Periférico      (1).pdfSistema Nervioso Periférico      (1).pdf
Sistema Nervioso Periférico (1).pdf
 

Examen_Simulacro_Tema_A_y_B_Sabado_4_de.pdf

  • 1. CICLO DE REVISION EN MEDICINA Examen Simulacro Tema A y B Academia de Preparación Médica Estudios M&C SAC. Jr. Manuel Corpancho Nro. 313 Urb. Santa Beatriz, Lima. www.estudiosmyc.com Sábado, 4 de Agosto del 2012 1. Hombre de 48 años que acude a Urgencias por dolor abdominal y vómitos. Los datos analíticos iniciales son: GOT: 80 U/I. GTP 54 U/I. Leucocitos 21.800/mm3. Amilasa 4.500 U/I. La TAC abdominal demuestra colección peripancreática. Se inicia tratamiento con fluidoterapia y analgésicos. Señale cuál de los siguientes fármacos añadiría al tratamiento, en primer lugar, para mejorar el pronóstico de este caso: A) Metilprednisolona. B) Antiinflamatorios no esteroideos. C) Omeprazol. D) Imipenem. E) Inhibidores de la secreción pancreática. 2. ¿Cuál es el principal factor responsable de la no cicatrización de una úlcera péptica?: A) Estrés. B) Infección por Helicobacter pylori. C) Consumo de alcohol. D) No abandono del hábito tabáquico. E) Determinados hábitos dietéticos. 3. ¿Cuál de las siguientes recomendaciones es FALSA en relación con el tratamiento de la peritonitis bacteriana espontánea en un paciente cirrótico?: A) El diagnóstico se basa en la existencia de más de 250 PMN/mm3 en el líquido ascítico. B) El tratamiento de elección lo constituyen las cefalosporinas de tercera generación. C) La administración de albúmina intravenosa previene el desarrollo de insuficiencia renal. D) El tratamiento antibiótico debe mantenerse durante 15 días. E) Una vez resuelta la infección, debe iniciarse tratamiento profiláctico con norfloxacino. 4. Paciente de 72 años, que como único tratamiento toma antidiabéticos orales, presenta anemia ferropénica crónica, con hemorragias ocultas positivas. ¿Cual es el método diagnóstico más indicado para localizar la lesión sangrante?: A) Tránsito gastroduodenal. B) Gammagrafía con hematíes marcados. C) Tránsito intestinal. D) Colonoscopia total. E) Panendoscopia oral. 5. ¿Cuál de las siguientes definiciones corresponde con mayor precisión al concepto de Metaanálisis?: A) Es una revisión narrativa en la que la bibliografía se busca de manera no estructurada. B) Es una revisión narrativa en la que la bibliografía se busca de manera estructurada. C) Es una revisión narrativa en la que se presentan tablas estructuradas de los resultados de los distintos estudios incluidos. D) Es una revisión en la que se combinan estadísticamente los resultados de los estudios incluidos.
  • 2. Examen Simulacro :: Ciclo de Revisión en Medicina :: Sábado 4 de Agosto del 2012 :: www.estudiosmyc.com E) Es una revisión en las que se presentan tablas estructuradas de los resultados de los distintos estudios incluidos. 6. Le encargan el diseño de un ensayo clínico en el que es muy importante que un factor pronóstico se distribuya por igual en los dos grupos de tratamiento. El método de aleatorización que usted utilizaría es: A) Aleatorización simple. B) Aleatorización por bloques. C) Aleatorización estratificada. D) Aleatorización centralizada. E) Aleatorización ciega (ocultación de la secuencia de aleatorización). 7. En una revista biomédica se publica un estudio, en el que los autores notifican el resultado en coste/años de vida ganados. ¿De qué tipo de análisis de evaluación económica se trata?: A) Coste de la enfermedad. B) Coste-efectividad. C) Coste-beneficio. D) Coste-consecuencia. E) Coste-utilidad. 8. La mejor forma de verificar una hipótesis en epidemiología es a través de: A) Un estudio descriptivo. B) Un estudio experimental. C) Un estudio caso-control. D) Un estudio de cohortes. E) Un estudio transversal. 9. Todas estas situaciones, EXCEPTO una, aumentan la probabilidad de detectar un cáncer colorrectal, señálela: A) Enfermedad inflamatoria intestinal de larga evolución. B) Endocarditis por Streptococcus bovis. C) Tabaquismo de más de 35 años de duración. D) Ureterosigmoidostomía hace 20 años, para corregir una malformación vesical. E) Ingesta crónica de aspirina o antiinflamatorios no esteroideos. 10. ¿Cuál de las siguientes características NO es propia de un ensayo clínico controlado?: A) Intervención experimental. B) Tratamiento asignado según el criterio del investigador. C) Participación voluntaria del sujeto. D) Asignación aleatoria a las distintas posibilidades de tratamiento en estudio. E) Enmascaramiento de la medicación. 11. La vacuna antineumocócica está recomendada para todas, MENOS una de las siguientes situaciones clínicas, señálela: A) Alcoholismo crónico. B) Contactos familiares de un paciente con neumonía neumocócica C) Fístula crónica de líquido cefalorraquídeo tras un traumatismo craneal. D) Infección VIH avanzada. E) Insuficiencia cardíaca crónica. 12. ¿Cuál de la siguientes vacunas no se incluye en las recomendaciones actuales de un paciente esplenectomizado?: A) Vacuna neumocócica. B) Vacuna meningocócica. C) Vacuna frente a Haemophilus influenzae tipo B. D) Vacuna frente al virus de la gripe. E) Vacuna frente al virus hepatitis A. 13. ¿En qué patología pensaría en primer lugar en un paciente de 65 años, que presenta disminución lenta, progresiva e indolora de su agudeza visual sin signos de inflamación ocular?: A) Error de refracción. B) Distrofia corneal. C) Papilitis. D) Glaucoma agudo. E) Degeneración macular senil. 14. Un paciente diabético tratado mediante fotocoagulación focal con láser de Argón tres años antes, presenta una pérdida brusca e importante de visión, sin dolor ni alteraciones en la superficie ocular. La causa más probable de esta disminución de agudeza visual es: A) Hemorragia vítrea. B) Edema corneal. C) Glaucoma crónico simple. D) Catarata nuclear. E) Atrofia óptica. 15. Una de los siguientes signos o síntomas, NO es habitual en una uveítis anterior aguda: A) Midriasis. B) Dolor. C) Inyección ciliar. D) Sinequias posteriores. E) Fotofobia. 16. ¿Cuál es el tratamiento de la obstrucción intestinal no quirúrgica en la fase terminal del cáncer?: A) Sonda nasogástrica, aspiración continua, sueroterapia intravenosa. B) Administración de vitamina grupo B, laxante y enema de limpieza. C) Alimentación parenteral total y esteroides i.v. D) Morfina, buscapina y haloperidol por vía subcutánea. E) Dieta absoluta y sueros por vía subcutánea. 17. ¿Cuál de los siguientes opioides NO es adecuado para el tratamiento del dolor crónico de etiología cancerosa?: A) Metadona. B) Meperidina. C) Morfina.
  • 3. Examen Simulacro :: Ciclo de Revisión en Medicina :: Sábado 4 de Agosto del 2012 :: www.estudiosmyc.com D) Fentanilo. E) Tramadol. 18. La localización ideal para hacer una traqueotomía es: A) La membrana crico-tiroidea. B) La membrana mio-tiroidea. C) Primer anillo traqueal. D) Segundo o tercer anillo traqueal. E) Cuarto o quinto anillo traqueal. 19. ¿Cuál de estas afirmaciones es FALSA en relación con la otitis media secretoria?: A) Los niños con paladar hendido son más susceptibles a padecerla. B) En casos unilaterales en adultos, es obligada la inspección del cavum. C) Su período de mayor incidencia en climas templados, es de junio a septiembre, coincidiendo con la época de baños. D) El 66% de los cultivos de las muestras obtenidas por miringocentesis presentan bacterias. E) La otoscopia puede mostrar niveles líquidos o burbujas de aire tras el tímpano íntegro. 20. Una mujer de 68 años, sin aparentes factores de riesgo cardiovascular, ingresa en la Unidad Coronaria del Hospital por un cuadro ¿agudo de cardiopatía isquémica. En la analítica realizada a su llegada se objetiva una anemia (Hb 8gr/dl) previamente no conocida. En este caso, la actitud más adecuada con respecto a la anemia es: A) Actitud expectante, ya que sólo se debe transfundir la anemia sintomática. B) Transfundir hematíes. C) Transfundir sangre total. D) Instaurar tratamiento con eritropoyetina. E) Administrar hierro intravenoso. 21. Los cuerpos de Howell-Jolly, son inclusiones eritrocitarias de fragmentos nucleares y se observan en: A) Asplenia. B) Mielofibrosis. C) Leucemia linfática crónica. D) Déficit de G6PD. E) Intoxicación por plomo. 22. ¿Cuál de los siguientes hallazgos es el más característico de la leucemia mieloide crónica?: A) El cromosoma Filadelfia. B) El esplenomegalia palpable. C) La disminución de la fosfatasa alcalina granulocitaria. D) El aumento del ácido úrico sérico. E) El reordenamiento del gen bcr/abl. 23. Joven de 23 años, con historia de conducta sexual de riesgo, consulta por fiebre, malestar general y dolor en glande. La exploración física muestra múltiples tatuajes, "piercings" y la presencia de tres lesiones vesiculosas en glande y adenopatías inguinales bilaterales. ¿Cuál es el diagnóstico más probable?: A) Herpes genital. B) Secundarismo luético. C) Primoinfección VIH. D) Infección por citomegalovirus. E) Condilomas acuminados. 24. En cuanto a la neumonía por Neumococo en paciente infectados por el virus de la inmunodeficiencia humana (VIH) es FALSO que: A) Es una infección frecuentemente bacteriémica. B) Tiene mayor incidencia que en la población general. C) Junto a Haemophilus influenzae es la causa más común de neumonía en pacientes con SIDA. D) Se recomienda vacuna neumocócica en aquellos pacientes con CD4 < 100 cel/L. E) Esta neumonía puede ser vista en pacientes con sistema inmune relativamente intacto. 25. La tuberculosis asociada a la infección por VIH se caracteriza por: A) Presentación subclínica de la enfermedad. B) Aparición característica en los estadios de inmunodepresión más severa (>50 CD4/mm3). C) Elevada frecuencia de afectación extrapulmonar y diseminada. D) Escaso rendimiento de los métodos microbiológicos de diagnóstico. E) Mala respuesta al tratamiento antituberculoso. 26. NO es propio del shock tóxico estafilocócico: A) Fiebre elevada. B) Lesiones cutáneas. C) Metástasis sépticas. D) Fracaso renal. E) Rabdomiólisis. 27. ¿Cuál de las siguientes permite diferenciar la pielonefritis aguda de la cistitis aguda?: A) Leucocituria. B) Hematuria. C) Bacteriuria. D) Antecedentes de infección urinaria. E) Fiebre de más de 38.5ºC. 28. Un paciente de 22 años, que acude para evaluación de una lesión genital ulcerada, presenta un VDRL positivo a título de 8 diluciones con un FTA-Abs negativo. La interpretación más adecuada de estos resultados es: A) Falso positivo de las pruebas no treponémicas. B) Falso negativo de las pruebas treponémicas. C) Sífilis curada (pendiente la positivización del FTA- Abs). D) Sífilis curada (pendiente la negativización del VDRL). E) Sífilis de larga evolución.
  • 4. Examen Simulacro :: Ciclo de Revisión en Medicina :: Sábado 4 de Agosto del 2012 :: www.estudiosmyc.com 29. Señale cuál de las siguientes asociaciones de helmintos y su clínica característica es INCORRECTA: A) Ascaris lumbricoides - Síndrome de Löeffler. B) Giardia lamblia – Mala absorción intestinal. C) Ancylostoma duodenale - Anemia megaloblástica. D) Strongyloides stercolaris - Síndrome de hiperinfestación en inmunodeprimidos. E) Taenia solium - Convulsiones generalizadas. 30. Paciente de 30 años, seropositivo VIH conocido desde 5 años antes, con antecedentes de neumonía por P. jivovecci, que consulta por cefalea desde 10 días antes. La exploración física muestra como datos más relevantes mínima rigidez de nuca y temperatura de 37,5ºC, fondo de ojo normal, TAC: ligera atrofia cortical. La punción lumbar da salida a líquido claro con 40 células mononucleares, proteínas: 90 mgrs%, glucosa: 30 mg% (glucemia: 90 mg%). Señalar, de entre las siguientes, la causa más probable: A) Herpesvirus tipo 8. B) Listeria. C) Criptococo. D) CMV. E) VIH. 31. En relación con Streptococcus pyogenes y la faringoamigdalitis, ¿cuál de las siguientes afirmaciones NO es correcta?: A) En tratamiento de la faringoamigdalitis estreptocócica se efectúa con una sola inyección i.m. de 1,200,000 UU de penicilina benzatina. B) El tratamiento antibiótico de la faringoamigdalitis estreptocócica se efectúa con 250,000 UU/6h oral de penicilina V durante 10 días. C) El tratamiento antibiótico de la faringoamigdalitis estreptocócica se efectúa con una sola inyección i.m. de 1,200,000 UU de penicilina procaína. D) El tratamiento antibiótico de la faringoamigdalitis estreptocócica se efectúa con amoxicilina oral 500 mg/8h durante 10 días. E) El tratamiento antibiótico recomendado de la faringoamigdalitis estreptocócica en los casos de alergia a la penicilina es un macrólido oral durante 10 días. 32. A lo largo de los últimos 10 años se ha producido un cambio muy importante en la etiología de la Endocarditis Infecciosa del adulto. El microorganismo más frecuente en la actualidad, es: A) Microorganismos del grupo HACEK. B) Staphylococcus aureus. C) Estafilococos coagulasa negativos. D) Bacilos gram negativos. E) Streptococcus viridans. 33. Un hombre de 74 años con un infarto agudo de miocardio es tratado con estreptoquinasa. Seis horas después desarrolla un cuadro de hipotensión arterial severa y obnubilación. ¿Cuál de las siguientes complicaciones es MENOS probable que sea la causa?: A) Infarto de ventrículo derecho. B) Tromboembolismo pulmonar. C) Rotura del músculo papilar. D) Rotura de la pared libre ventricular. E) Hemorragia cerebral. 34. En el tratamiento de la hipertensión arterial, la ventaja de los bloqueadores de los receptores de la Angiotensina II con respecto a los inhibidores del enzima conversor de la Angiotensina es que: A) Son más potentes. B) Producen menos tos. C) No producen hiperpotasemia. D) Se puede dar en embarazadas. E) Se pueden dar en sujetos con estenosis de la arteria renal bilateral. 35. El tratamiento más eficaz para prevenir recurrencias en el aleteo o flutter auricular común es: A) Buen control de la hipertensión arterial que con frecuencia padecen estos enfermos. B) Digoxina asociada a un fármaco que disminuya la conducción en el nodo A-V (anticálcicos o betabloqueantes). C) Ablación con catéter y radiofrecuencia del istmo cavo tricúspide. D) Inserción de un marcapasos con capacidad antitaquicardia. E) Amiodarona. 36. Un paciente obnubilado con una presión arterial de 80/40 mmHg, un gasto cardíaco de 3 l/min, una presión de enclavamiento pulmonar de 14 mmHg y una presión auricular derecha de 14 mmHg, puede estar sufriendo cualquiera de las siguientes situaciones patológicas con la excepción de una: A) Taponamiento cardíaco. B) Deshidratación. C) Infarto de ventrículo derecho. D) Tromboembolismo pulmonar. E) Constricción pericárdica. 37. La campana del fonendoscopio es más adecuada que la membrana para: A) Distinguir un soplo diastólico de uno sistólico. B) Oír mejor el chasquido de apertura en pacientes con estenosis mitral. C) Valorar la presencia de un tercer y/o cuarto tono. D) Detectar el click mesosistólico del prolapso mitral. E) Auscultar a los niños pequeños y bebés, en los que la membrana no capta bien la tonalidad de los ruidos cardíacos. 38. ¿Cuál de las siguientes respuestas sobre el Delirium Tremens es la correcta?: A) Se trata del estado peculiar de intoxicación producido por el consumo de alcohol. B) Se caracteriza por la aparición de conductas impulsivas tras el consumo de una pequeña cantidad de alcohol.
  • 5. Examen Simulacro :: Ciclo de Revisión en Medicina :: Sábado 4 de Agosto del 2012 :: www.estudiosmyc.com C) Es un cuadro clínico con elevado índice de complicaciones psiquiátricas y con pocas complicaciones orgánicas. D) Aparece con mayor frecuencia entre los hombres alcohólicos a partir de los 50 años. E) Su tratamiento de elección son las benzodiacepinas. 39. ¿Cuáles son los trastornos perceptivos, de entre los que se enumeran, más característicos de las esquizofrenias?: A) Ilusiones hipnagógicas. B) Alucinaciones visuales zoomórficas. C) Alucinaciones o pseudoalucinaciones auditivas. D) Paraeidolias. E) Alucinosis auditivas. 40. Una paciente de 24 años acude a la Urgencia llevada por un familiar por vómitos recidivantes. Parece bulimia nerviosa. ¿Cuál de las pruebas de laboratorio es más útil para evaluar la gravedad de los vómitos?: A) Nivel de Hemoglobina. B) Nivel plasmático de Amilasa. C) Nivel sérico de Socio. D) Nivel plasmático de Calcio. E) Nivel plasmático de Creatinina. 41. Acude a urgencias un paciente de 37 años con gran postración, fiebre elevada (38,5°C) e intensa cefalea. Presenta en miembros inferiores, tronco y raíz de miembros superiores un exantema tenue eritemato- violáceo, maculoso, escasamente confluente, con afectación de palmas y plantas. Al interrogar a familiares, reconocen haber estado hace siete días en una excursión. Reexplorado el paciente, en cara posterior de la piera derecha aparece una lesión necrótica-costrosa, de 0,5 cm. de diámetro, rodeada por un halo violáceo, edematoso de 0,3-0,4 cm. El tratamiento más indicado sería: A) Doxiclina v.o. (200 mgr. cada 12 horas un día ó 100 mgr. cada 12 horas por 5 días). B) Cloxacilina 1 gr. i.v. cada 6 horas durante 10 días. C) Ceftriaxona 1 gr. i.v. cada 12 horas durante 10 días. D) TMP-SMX 500 mgr. i.v. cada 8 horas por 10 días. E) Eritromicina 250 mgr. i.v. cada 6 horas por 7 días. 42. Un paciente de 27 años ex-ADVP desde hace tres años, con muguet oral y antecedentes de neumonía por neumocistis carinii, presenta en el dedo índice, en superficie dorsal de la 2.ª falange, una lesión única, úlcero necrótica, con crecimiento serpinginoso de aproximadamente 4-5 cms. de diámetro, bordes geográficos y algunas áreas costrosas, hemorrágicas. Es extremadamente dolorosa. Su 1.ª sospecha clínica es: A) Linfoma B cutáneo. B) Ulcera secundaria al tratamiento. C) Infección por herpes simple. D) Chancro luético. E) Picadura sobreinfectada. 43. Un paciente de 67 años, comienza a presentar lesiones eritematoedematosas en brazos y abdomen. Ocasionalmente se observan ampollas salpicando el área afectada. No hay afectación de mucosas. Conserva el estado general. Se realizó biopsia de piel donde se observó las imagen de una vesícula subepidérmica con eosinófilos. En inmunofluorescencia directa se observó una imagen de depósito lineal en UDE de IgG. Tras usar la técnica de separación con sal, la banda de depósito aparece tanto en el suelo, como en el techo de la ampolla. El diagnóstico más probable será: A) Enfermedad de During-Brocq. B) Fogo Selvagen. C) Enfermedad IgA lineal. D) Epidermólisis ampollosa adquirida. E) Penfigoide ampolloso. 44. Un paciente de 44 presenta lesiones pápulo- erosivas muy pruriginosas en glúteo y rodillas, que comenzaron a salir hace meses, cursando en brotes que curan espontáneamente en una semana. Se ha realizado biopsia de piel observándose ampolla subepidérmica en tinción de hematoxilina-eosina y un depósito de IgA linea en UDE. La biopsia de vellosidades intestinales en normal. Los anticuerpos antigliadina IgA son negativos. Los anticuerpos antigliadina IgG y los anticuerpos antiendomisio son positivos. Presenta además anticuerpos antitiroideos. El tratamiento adecuado será: A) Corticoides orales porque es una enfermedad IgA lineal. B) Corticoides orales porque es una Dermatitis Herpetiforme con biopsia intestinal negativa. C) Sulfonas más dieta sin gluten, porque es una Dermatitis Herpetiforme. D) Sulfonas sin dieta, porque es una Dermtitis Herpetiforme con biopsia intestinal negativa. E) Sulfonas más inmunosupresores. 45. Un varón de 6 años presenta de manera brusca un brote de lesiones eritematoescamosas redondeadas, de pqueño tamaño, distribuídas por todo el tegumento. La semana anterior acudió a Urgencias por un cuadro catarral, con T°= 37,5°C y amigdalitis. En el cultivo faríngeo se aisló un estreptococo. La patología cutánea más probable corresponde a: A) Pitiriasis rosada. B) Eczema numular. C) Psoriasis gutata. D) Herpes circinado. E) Vasculitis séptica. 46. Un joven de 26 años, con antecedentes familiares de psoriasis, presenta un brote de lesiones en placas eritematosas, con centro amarillento y halo periférico descamativo, ovaladas, bien delimitadas, en cara anterior y posterior del tronco. La 1.ª lesión había aparecido hacía 1 semana, era de mayor tamaño y se localizaba en el tercio superior de la espalda. El
  • 6. Examen Simulacro :: Ciclo de Revisión en Medicina :: Sábado 4 de Agosto del 2012 :: www.estudiosmyc.com paciente conservó un buen estado general en todo momento. Probablemente el cuadro corresponde a: A) Psoriasis en pequeñas placas. B) Psoriasis tipo Von Zumbusch. C) Pitiriasis rosada de Gibert. D) Roséola sifilítica. E) Eczema diseminado. 47. Un varón de 16 años comienza a presentar vesículas de contenido claro agrupadas, de localización peribucal, que a lo largo de 1 semana se van rompiendo formándose erosiones y costras. Transcurrido este período, en el dorso de las manos aparecen 2 lesiones ampollosas con vesículas dispuestas periféricamente y alguna pápula eritematosa con centro más oscuro. Se trata de: A) Inicio de una varicela. B) Aparición de lesiones en manos por contagio directo desde la zona peribucal. C) Impétigo estafilocócico. D) Eritema multiforme minor. E) Síndrome de Stevens-Johnson. 48. Una mujer de 65 años viene presentando desde hace 2 años brotes de lesiones nodulares < de 2 cm., localizadas de forma bilateral en MMII, acompañados de livedo reticularis, febrícula y artralgias. La histología corresponde a una vasculitis leucocitoclástica. Probablemente se trata de: A) Síndrome de Sweet. B) Eritema elevatum diutinum. C) Panarteritis nodosa cutánea. D) Vasculitis urticarial. E) Enfermedad de Kawasaki. 49. Un paciente de 63 años de edad presenta una eritrodermia de varios meses de evolución con adenopatías generalizadas y más de 10% de células con núcleo cerebriforme en sangre periférica. Tiene además intenso prurito y edemas pretibiales. Su diagnóstico sería: A) Eczema seborreico. B) Exantema medicamentoso crónico. C) Eritrodermia psoriasica. D) Síndrome de Sezary. E) Parapsoriasis en grandes placas. 50. Un niño sin alteraciones cutáneas al nacimiento comienza a presentar hacia los 3 meses de edad escamas grandes y negruzcas en tronco y extremidades, afectando pliegues axilares y poplíteos. Tiene un hermano mayor igualmente afecto. Es probable que se trate de: A) Una ictiosis ligada a X. B) Una ictiosis laminar. C) Una ictiosis vulgar. D) Enfermedad de Darier. E) Ninguna de las anteriores. 51. Una mujer acude a nuestra consulta poque tras haber estado esa mañana expuesta al sol presenta una quemadura solar exagerada localizada en cara (respetando región peiorbitaria, retroauricular y submentoniana), dorso de manos y en ambas piernas hasta la altura de las rodillas. Estaba en tratamiento con un diurético tiacidico. A) Se trata de una reacción fototóxica. B) Es una reacción fotoalérgica por mecanismo tipo IV. C) Debe eliminarse el agente causante, poner tratamiento sintomático y evitar la radiación lumínica hasta que remita el cuadro. D) Es una erupción polimorfa lumínica. E) Son correctas A y C. 52. Una joven de 16 años de edad presenta varias máculas de color blanco lechoso de varios centímetros de diámetro, de distribución simétrica sobre codos, rodillas, manos y zona peribucal. En la biopsia cutánea hay ausencia de malanocitos. Tras fotoquimioterapia sistémica con psoralenos han pigmentado parcialmente las lesiones. El diagnóstico de esta paciente es: A) Esclerosis tuberosa. B) Lepra. C) Hipomelanosis guttata idiopática. D) Vitíligo. E) Hipomelanosis de Ito. 53. Niño de 10 años de edad que acude a la consulta por amigdalitis pultácea con adenopatias cervicales. Se pone tratamiento con penicilina a dosis correctas. A las 72 horas acude de nuevo al no experimentar mejoria. Debemos pensar en: A) Posible Mononucleosis infecciosa. B) Posible amigdalitis viral de etiología diferente al Estreptococo beta hemolítico del grupo A. C) Linfoma. D) V. Parainfluenzae. E) Rubéola. 54. La presencia de vómitos en la infancia es uno de los motivos más frecuentes de consulta. Pueden ser debidos a enfermedades digestivas y extradigestivas. ¿Cuál de las siguientes patologías produce menos frecuentemente vómitos en la etapa de la lactancia? A) Reflujo gastroesofágico. B) Estenosis hipertrófica de píloro. C) Apendicitis. D) Invaginación intestinal. E) Gastroenteritis. 55. Ante un niño que de forma brusca presenta hipotensión, vómitos y colapso cardiovascular que no responde a la administración de drogas vasoactivas o catecolaminas y en la analítica realizada presenta hiponatremia e hiperpotasemia, se debe considerar como probable diagnóstico: A) Insuficiencia cardíaca. B) Insuficiencia suprarrenal. C) Diabetes juvenil. D) Diabetes insípida. E) Intoxicación por monóxido de carbono.
  • 7. Examen Simulacro :: Ciclo de Revisión en Medicina :: Sábado 4 de Agosto del 2012 :: www.estudiosmyc.com 56. Un varón de 17 años se encuentra mareado, con vómitos y en el transcurso de unos minutos se halla tumbado en el suelo en coma, con una exploración neurológica normal. La causa más probable sería: A) Ingestión de barbitúricos. B) Hemorragia subaracnoidea. C) Coma etílico. D) Status convulsivo. E) Tumor cerebral. 57. Un varón de 4 años tiene lesiones purpúricas palpables, simétricas, de 3 días de evolución en las extremidades inferiores. Los estudios hematológicos revelan: Hemoglobina: 10 g/dl; recuento leucocitario 16.500/mm3; recuento plaquetario 240.000/mm3 y VSG de 45 mm/hora. La etiología más probable es: A) Maltrato infantil. B) Púrpura de Schonlein Henoch. C) Enfermedad de Kawasaki. D) Meningococemia. E) Enfermedad de Von Willebrand. 58. Durante la evaluación previa al ingreso en el colegio de un niño de 5 años, se detecta retraso en el habla. Como antecedentes se recogen episodios reiterados de cuadros catarrales sin control médico. ¿Cuál de las siguientes es la casusa más probabnle de esta situación? A) Trastorno de déficit de atención con hiperactividad. B) Hipoacusia de conducción. C) Disfunción de la Trompa de Eustaquio. D) Retraso mental. E) Hipoacusia neurosensorial. 59. Niña de 7 años sin antecedentes de interés que acude a urgencias por dolor abdominal generalizado y vómitos desde doce horas antes. Deposición normal. No antecedentes quirúrgicos previos. A la exploración presenta abdomen muy distendido y dolor en zona periumbilical con aumento de ruidos intestinales, Blumberg (-). En la Radiografía de abdomen en bipedestación se observa obstrucción a nivel de intestino delgado. El diagnóstico más probable sería: A) Invaginación intestinal. B) Brida intestinal. C) Malrotación intestinal. D) Divertículo de Meckel. E) Estenosis ileal congénita. 60. Niño de 5 meses que llevan a la consulta por tos y secreción nasal desde hace 24 horas. Toma mal los biberones por presentar fatiga. Se observa febrícula y retraccion intercostal, con zonas de hipoventilación y estertores en la auscultacion respiratoria. Existen otros familiares con cuadro catarral. Se le pone tratamiento sintomático y se aconseja la revisión a las 24 horas. En la nueva visita el niño ha empeorado, con importante insuficiencia respiratoria, sibilancias, tos continua y fiebre de 38°C. ¿Cuál de los siguientes cuadros cree que presenta el niño?: A) Catarro habitual descendente con evolución a asma. B) Aspiración de cuerpo extraño. C) Bronquiolitis. D) Epiglotitis. E) Laringitis. 61. Un niño nacido a término de 2.100 gr. de peso presenta irritabilidad y temblores amplios a las 36 horas de vida. Se alimenta mal y tiene diarrea y obstrucción nasal. ¿Cuál es el diagnostico más probable?: A) Hipocalcemia. B) Hipomagnesemia. C) Déficit de piridoxina. D) Sindrome de abstinencia por adición materna a heroína. E) Hipoglucemia. 62. Lactante de dos meses que presenta llanto agudo, en crisis, desde hace 20 días, síntomas motores y heces normales para su edad. ¿El diagnóstico más probable será?: A) Gastroenteritis aguda. B) Intolerancia a la lactosa. C) Cólico del lactante. D) Otitis media aguda. E) Hernia inguinal. 63. Lactante de tres meses sin antecedentes previos de interés, que en el examen de salud se le detecta un deterioro en las adquisiciones sicomotoras, el resto de la exploración por aparatos es normal. En la anamnesis presenta sacudidas musculares breves de cabeza y extremidades superiores. ¿Cuál sería el diagnóstico más probable?: A) Cólico del lactante. B) Hemorragia cerebral. C) Síndrome de West. D) Síndrome de Lenaux-Gastaut. E) Fenilcetonuria. 64. Ante un lactante de 1,5 meses de edad que presenta ictericia debemos pensar en todos los siguientes cuadros, excepto en: A) Infección urinaria. B) Lactancia materna. C) Atresia congénita de vías biliares. D) Ictericia fisiológica. E) Hipotiroidismo congénito. 65. Un paciente acude por cuadro de dolor intenso epigástrico, de aparición brusca, acompañado de vómitos en los que sólo consigue arrojar saliva, gran distensión abdominal alta e imposibilidad para pasar más allá del esófago distal con una sonda nasogástrica, ¿cuál sería la principal sospecha diagnóstica?: A) Perforación gástrica. B) Estenosis pilórica aguda. C) Vólvulo gástrico agudo.
  • 8. Examen Simulacro :: Ciclo de Revisión en Medicina :: Sábado 4 de Agosto del 2012 :: www.estudiosmyc.com D) Síndrome de Boerhaave. E) Tricobezoar. 66. Un paciente de 60 años refiere dolor en epigastrio desde hace unas tres semanas y en menor medida desde meses antes, que se alivia con los alimentos y reaparece 2 horas después de las comidas, con irradiación a hipocondrio derecho. En la gastroscopia muestra una úlcera de 1 cm. en curvadura menor, con bordes netos bien definidos, ausencia de islotes de tejido dentro del nicho ulceroso, con mucosa de aspecto normal. Ante lo cual Ud.: A) Decide no practicar biopsia de la úlcera por tener características de benignidad. B) Sólo practicará biopsia del fondo del nicho. C) Practicaría biopsias múltiples independientemente de las características del nicho ulceroso. D) Algunos de los signos descritos son sugestivos de malignidad, por lo que practicaría biopsias múltiples. E) Por no tener características de malignidad claras, no practicaría biopsias. 67. Paciente de 84 años que presenta cuadro de diarrea mucosa con decaimiento generalizado y pérdida de peso. Analíticamente el paciente presenta anemia con hipopotasemia, hiponatremia e hipocloremia; al tacto rectal se palpa una masa homogénea de consistencia blanda, recubierta de moco, no dolorosa. Este paciente presentará con gran probabilidad: A) Hemorroides. B) Enteropatía pierde proteínas. C) Leiomioma de recto. D) Adenoma velloso de recto. E) Hamartoma rectal. 68. Mujer de 42 años que acude al hospital con historia de 10 años de disfagia, primero para líquidos y posteriormente para sólidos, y que en la actualidad presenta regurgitaciones de carácter principalmente nocturno. ¿Cuál sería la prueba diagnóstica esencial?: A) Rx de tórax. B) Tránsito digestivo. C) Endoscopia alta. D) Manometría esofágica. E) TAC torácica. 69. Un varón de 60 años sin antecedentes personales ni familiares de interés presenta sangre roja mezclada con las heces. El médico realiza una inspección perianal y un tacto rectal encontrando hemorroides internas grado III. La actitud más correcta sería: A) Medidas higiénicas más pomada anti hemorroidal. B) Solicitar endoscopia alta. C) Solicitar un enema opaco. D) Solicitar rectoscopia y enema opaco. E) Solicitar colonoscopia total. 70. Paciente de 43 años en tratamiento por colitis ulcerosa con corticoides y sulfasalazina; acude a urgencias por malestar general, fiebre, distensión abdominal, náuseas y vómitos. Presenta gran distensión abdominal, intenso dolor a la palpación y signos de irritación peritoneal. En la analítica presenta marcada leucocitosis con desviación izquierda. En la RX simple de abdomen se observa gran dilatación del colon. El paso siguiente es: A) Intervención quirúrgica. B) Rectocolonoscopia. C) Inmunosupresores. D) Sonda nasogástrica, sueros y corticoides iv. E) TAC abdominal. 71. Una mujer de 52 años diagnosticada de úlcera gástrica de 2 cm. con biopsia negativa por malignidad, a las 8 semanas de tratamiento con ranitidina 150 mg cada 12 horas, se somete a control endoscópico encontrándose una úlcera de 0,5 cm, con nueva biopsia negativa y totalmente asintomática. ¿Cuál es la conducta preferida?: A) Cambiar el medicamento a fenotidina. B) Añadir curalfato a la ranitidina. C) Remitir por cirugía. D) Continuar con ranitidina 8 semanas más. E) Suspender toda medicación. 72. Una mujer de 76 años presenta vómito en posos de café. En la endoscopia se encontró un pólipo en cuerpo gástrico. No se identificó ningún otro origen de la hemorragia. El hematócrito es del 28%. ¿Cuál es la mejor opción terapéutica?: A) Biopsia con pinzas por estudio histológico, si es benigna ninún tratamiento adicional. B) Biopsia con pinzas, si es benigna, antagonistas H2. C) Polipectomía endoscópica con asas. D) Resección quirúrgica. E) Actitud expectante por si se repitiese la hemorragia. 73. Un paciente de 45 años con anemia ferropénica y colonoscopia normal se sometió a endoscopia alta observándose un duodeno con mucosa testoreada sin lesiones sugerentes de hemorragia. La biopsia mostró atrofia total de vellosidades, lo cual sugiere: A) Glardiasis. B) Mucosa normal. C) Celíaca. D) Enfermedad de Crohn. E) Linfoma intestinal. 74. Una mujer de 65 años con artritis reumatoide deformante grave se presenta en el hospital por dolor periumbilical de inicio nocturno, con aumento rápido de intensidad. La exploración abdominal era casi normal con hemocultivo positivo, recuento de leucocitos de 20.000 mm3 con desviación a la izquierda y VSG > 100 mm/h. ¿Cuál es el diagnóstico más probable?: A) Colitis ulcerosa. B) Enfermedad de Crohn. C) Isquemia intestinal. D) Parasitosis intestinal. E) Angiodisplasia de colon.
  • 9. Examen Simulacro :: Ciclo de Revisión en Medicina :: Sábado 4 de Agosto del 2012 :: www.estudiosmyc.com 75. Varón de 78 años con debilidad, pérdida de peso, diarrea, artritis y fiebre, durante el último año. En el examen físico: pérdida de masa muscular, linfadenopatía y tumefacción en rodilla izquierda. En pruebas de laboratorio: anemia ferropénica y hemault positivo. Con Rx abdomen y enema opaco banales. El diagnóstico diferencial debe incluir todos los siguientes, excepto: A) SIDA. B) Enfermedad de Crohn. C) Vasculitis reumatoide. D) Enfermedad de Whipple. E) Singellosis. 76. Se hace colonoscopia en un enfermo, donde aparece un pólipo de 8 mm en colon sigmoide que se extirpa sin observar otras lesiones. ¿Cuál de los siguientes considera el intervalo de vigilancia en este paciente?: A) Seis meses. B) Un año. C) Dos años. D) Tres años. E) Cinco años. 77. Niña de 6 años remitida a consulta por hemorragia vginal, que presenta un desarrollo mamario, en etapa III de Tanner, estatura alta, con una edad ósea de 9 años (Rx. de mano y muñeca izda.), valores basales de gonadotropinas y estradiol elevados para la prepubertad con ovarios aumentados para la edad y con múltiples quistes de diámetro igual o mayor de 14 mm. Examen neurológico clínico-radiológico normal, sin pigmentación cutánea en mancha de café con leche y estudio hormonal tiroideo y suprarrenal normal. El tratamiento de elección es: A) Laparocopia diagnóstico-terapéutica ya que su etiología es un tumor ovárico. B) Agonistas de GnRH, ya que estamos ante una PPV, y los agonistas de GnRH son los únicos que retrasan el desarrollo sexual y la maduración esquelética. C) Danazol. D) Acetato de medroxiprogesterona ya que producen un retraso en el desarrollo sexual y muy buenos resultados en el control del crecimiento. E) No precisa tratamiento, aunque sí una vigilancia anual. 78. Una paciente de 30 años, que consulta por amenorrea secundaria, presenta concentraciones plasmáticas basales de FSH 2 MUI/ml., LH 1,5, MUI/ml., prolactina 9 ngr./ml. Tras la administración de gestágenos 10 mgr./día durante 5 días no se observa sangrado vaginal. En cambio, tras la administración de estrógenos conjugados durante 21 días y en los 5 últimos días gestágenos aparece una menstruación. De las siguientes causas de amenorrea ¿cuál es la que corresponde con el cuadro clínico?: A) Síndrome de ovario poliquístico. B) Fallo ovárico autoinmune. C) Prolactinoma hipofisario. D) Síndrome de Asherman. E) Tumor cerebral. 79. Una mujer de 54 años solicita tratamiento hormonal sustitutivo por síntomas neurovegetativos y manifestaciones genitourinarias importantes. Refiere que la >FUR fue hace un año y medio, pero que hace 2 meses ha empezado a sangar muy abundantemente. Presenta exploración física y mamografía normal. ¿Cuál es la actitud más apropiada en esta paciente?: A) Tratamiento con gestágenos. B) Tratamiento con estrógenos. C) Tratamiento con terapia combinada. D) Ecografía transvaginal para valorar línea media y si existe alguna duda de patología endometrial realizar histología endometrial. E) Citología (triple toma). 80. Una mujer de 45 años con el antecedente de un proceso gripal hace dos semanas por el que fue tratado con amoxicilina presenta ahora un cuadro de prurito vulvar y leucorrea. En la exploración se aprecia enrojecimiento y edemas de la vulva y del introito y secreción vaginal blanca grumosa de aspecto caseoso. En la mucosa vaginal aparecen unas placas blanquecinas irregulares que se desprenden con facilidad y dejan ulceraciones rojas superficiales. ¿La etiología probable es?: A) Candidiasis o moniliasis. B) Herpes genital. C) Tricomaniasis. D) Gardnerella vaginalis. E) Clamidias. 81. Mujer de 40 años que consulta por cuadro de poliartritis simétrica de grandes y pequeñas articulaciones de 15 días de evolución. En la analítica destaca un factor reumatoide positivo siendo el resto del estudio inmunológico negativo. ¿Qué diagnóstico realizaría?: A) Artritis reumatoide. B) Artritis paraneoplásica. C) Artritis no filiada. D) Enfermedad de Still del adulto. E) Poliartritis vírica. 82. Mujer de 65 años que ingresa por cuadro de cefalea, fiebre y dolor con limitación de ambos hombros y ambas caderas de 2 meses de evolución. El resto de anamnesis y exploración física no aporta datos relevantes. La analítica pone de manifiesto una gran elevación de los reactantes de fase aguda (VSG y PCR). La conducta a seguir sería: A) Diagnosticar a la paciente de polimialgia reumática e iniciar tratamiento. B) Realizar biopsia de la arteria temporal para descartar la existencia de arteritis pues el tratamiento difiere.
  • 10. Examen Simulacro :: Ciclo de Revisión en Medicina :: Sábado 4 de Agosto del 2012 :: www.estudiosmyc.com C) Iniciar tratamiento con prednisona 1 mg./kg./día para “curarse en salud” ante la posible existencia de una arteritis. D) Realizar artrocentesis de un hombro o una cadera para descartar primero la existencia de una artritis séptica. E) Iniciar tratamiento con 20 mg./día de prednisona y si no mejorara en 15 días subir a 1 mg./kg./día. 83. Varón de 25 años afecto clínica y radiológicamente de una sacroileítis unilateral de 2 meses de evolución. La conducta a seguir es la siguiente: A) Solicitar analítica con HLA y dar tratamiento con AINE ante la sospecha diagnóstica de espondiloartropatía. B) Además de lo anterior añadir salazopirina al tratamiento pues probablemente padecerá una espondilitis anquilosante. C) Además de lo anterior realizar una buena anamnesis y exploración física para descartar la existencia de psoriasis. D) Realizar anamnesis y exploración física, solicitar analítica con HLA así como ppD y serologías a brucella. E) Lo primero es realizar una artrocentesis de la articulación afecta para descartar proceso infeccioso crónico. 84. Mujer de 45 años que acude a la consulta por “dolor óseo generalizado” desde hace varios años. La conducta a seguir es: A) Diagnosticarla de fibromialgia y tratarla con analgésicos y antidepresivos. B) Sospechar la posible existencia de proceso metastásico y realizar un completo estudio de búsqueda del tumor primario. C) Realizar una correcta anamnesis, exploración física y solicitar analítica completa con hormonas tiroideas y CPK. D) Además de lo explicado en el apartado 3., solicitar una gammagrafía ósea para asegurarse de la existencia de proceso inflamatorio articular. E) Realizar una anamnesis y exploración física correctas. Solicitar un estudio analítico con VSG y PCR. Cuando exista sospecha clínica solicitar determinación de hormonas tiroideas, anticuerpos anti-ANA y creatin- P-quinasas. 85. Varón de 45 años que acude a la consulta por presentar dolor, tumefacción y edema a nivel de la mano izquierda. Entre sus antecedentes patológicos únicamente destaca la luxación del hombro izquierdo 1 mes antes. La conducta a seguir es la siguiente: A) Realizar artrocentesis de la muñeca para descartar artritis séptica. B) Solicitar analítica y radiografías ante la sospecha de artritis de muñeca. C) Solicitar analítica, radiografías y gammagrafía ósea con tecnecio ante la sospecha de DSR. D) Solicitar analítica con inmunología y radiografías ante la sospecha de inicio de proceso inflamatorio crónico. E) Solicitar analítica, radiografías y gammagrafía con tecnecio y galio ante la sospecha de DSR. 86. Mujer de 20 años que acude por dolor y tumefacción de ambos tobillos junto lesiones cutáneas eritematovioláceas dolorosas en ambas EEII de 10 días de evolución. El diagnóstico más probable es: A) Artritis reactiva. B) Vasculitis. C) Sarcoidosis. D) Enfermedad inflamatoria intestinal. E) Artropatía no filiada. 87. Una paciente de 55 años sin antecedentes patológicos de interés consulta por dorsalgia de inicio brusco y ritmo mecánico de dolor. Aporta Rx simple de columna en la que se aprecian aplastamientos vertebrales múltiples. La exploración física es normal salvo por la presencia de debilidad muscular proximal. Ante la sospecha clínica de osteomalacia, cuál de las siguientes exploraciones es más rentable: A) VSG y hemograma. B) Determinación de calcio, fosforo y fosfatasas alcalinas. C) Densitometría ósea. D) Gammagrafía ósea. E) Resonancia magnética. 88. Una paciente de 23 años con antecedentes de ulcus duodenal presenta una poliartritis simétrica con afección predominante de manos. En la exploración física, aparte de la poliartritis, presenta aftas orales. Se practica analítica general que muestra como únicas alteraciones VSG: 40 mm/1.ª hora, leucocitos: 3.000/mm3 (1.200 linfocitos) y ANA + 1/320 patrón homogéneo. ¿Cuál sería su diagnóstico?: A) Artritis reumatoide. B) Lupus eritematoso sistémico. C) Gota poliarticular. D) Condrocalcinosis. E) Artritis reactiva. 89. ¿Cuál es el tratamiento indicado en la paciente de la pregunta anterior?: A) Antiinflamatorios no esteroideos. B) Antipalúdicos. C) Glucocorticoides orales. D) Pulsos de metilprednisolona. E) Inmunosupresores. 90. Un paciente de 45 años consulta porque en una analítica de rutina se ha detectado una uricemia de 9 mg./dl. No existen antecedentes de artritis ni cálculos urinarios. ¿Cuál es la actitud correcta?: A) Iniciar tratamiento con uricosúricos. B) Iniciar tratamiento con uricosúricos y colchicina. C) Iniciar tratamiento con alopurinol. D) Iniciar tratamiento con alopurinol y colchicina. E) No precisa tratamiento.
  • 11. Examen Simulacro :: Ciclo de Revisión en Medicina :: Sábado 4 de Agosto del 2012 :: www.estudiosmyc.com 91. Una paciente de 15 años es remitida por sospecha de fiebre reumática: tras un cuadro gripal con artromialgias generalizadas se determinaron las antiestreptolisinas (ASLO), que son positivas. ¿Cuál de las siguientes afirmaciones es verdadera?: A) Ante el cuadro clínico de la paciente y las ASLO +, el diagnóstico de fiebre reumática es seguro. B) Con las ASLO +, bastaría para diagnosticar la fiebre reumática. C) La fiebre reumática es un tipo de artritis séptica. D) Con estos datos, no se puede realizar el diagnóstico de fiebre reumática. E) Debemos iniciar rápidamente tratamiento con penicilina. 92. Una paciente de 40 años presenta una gonartosis con afección predominante del compartimento interno de la rodilla e impotencia funcional severa. Señale la respuesta verdadera: A) El tratamiento es reposo absoluto y esperar a que sea más mayor para colocarle una prótesis. B) Una osteotomía podría estar indicada. C) Nunca se debe tratar con antiinflamatorios. D) No debe realizar fisioterapia ya que podría dañarse más la articulación. E) El uso de un bastón está contraindicado. 93. Un paciente presenta latencia del sueño de menos de 10 minutos con dificultades para despertarse y episodios de sueño de 18-20 horas, asociado al despertarse con hiperfagia, hipersexualidad, aumento de peso, irritabilidad, depresión, comportamiento impulsivo, disfunción vegetativa y alteraciones neurológicas. Las siestas diurnas duran varias horas. Estos episodios duran varias semanas intercalándose varios meses sin somnolencia. En uno de los episodios de somnolencia el paciente tuvo un grave accidente. El diagnóstico más probable es: A) Epilepsia. B) Narcolepsia. C) Simulación o trastorno conversivo-histeria. D) Síndrome de Klein-Levin. E) Apnea del sueño. 94. Una chica de 17 años acude al servicio de urgencias traída por su madre por un desmayo. Cuando la vemos está irritable y dice querer irse de alta porque no le pasa nada. Va con ropas holgadas y algo más abrigada que el resto de pacientes, está delgada y parece más joven de lo que le corresponde por su edad. Dice comer normal y niega usar laxantes o diuréticos, su madre dice que no come con ellos porque es muy activa y está todo el día fuera. Rompió con el novio hace un año. Dice que está harta de tener a su madre siempre pendiente de ella. El dato que más nos ayudaría a diferenciar una anorexia nerviosa de otros trastornos sería: A) Presencia de amenorrea. B) Peso inferior al 15%. C) Que la paciente diga que tiene miedo a ganar peso o que diga que se ve gorda. D) Que diga que tiene frío. E) Uñas y pelo frágil. 95. Una adolescente de apariencia física normal acude por irritabilidad y tristeza. Comenta que decidió comer menos en las comidas porque estaba algo gorda. Haciendo esto consigue adelgazar a temporadas pero luego se vuelve a engordar. Ahora está en período de transición, sigue restringiendo las comidas pero se pasa todo el día picoteando y en ocasiones come más de lo que quisiera de forma descontrolada. En esos descontroles efectivamente la paciente come una gran cantidad de alimentos especialmente chocolate, galletas y dulces, aunque en otras ocasiones son salados. Tras estos atracones se siente muy culpable e irritada y vomita para evitar engordarse. El diagnóstico más probable es: A) Síndrome de Klein-Levin. B) Depresión bipolar. C) Bulimia. D) Esquizofrenia. E) Anorexia nerviosa tipo compulsivo/purgativo. 96. Una paciente de 20 años se presenta en la guardia quejándose al internista de que tiene ataques de corazón y sensación de ahogo, sudoración y sensación de mareo que se inicia bruscamente mientras estaba leyendo relajadamente en su casa, le duró unos minutos, creyó morir y sentía que lo que le ocurría no era real, temiendo perder el control o estar volviéndose loca. Se le realizan pruebas ECG y auscultación, detectándose frecuencia cardíaca alta y signos sugerentes de prolapso mitral. Aun así dada la gran ansiedad de la paciente se llama al psiquiatra ya que insiste en que se le hagan más pruebas y en que no se quiere quedar sola en casa nunca más, ¿cuál es el posible diagnóstico?: A) Prolapso mitral. B) Agorafobia. C) Angor. D) Trastorno de angustia. E) A y D. 97. Una paciente de 55 años acude a consultas por cansancio especialmente por la mañana, falta de concentración desde hace 1 mes. Dice estar por las mañanas despierta antes de hora sin poder dormir. Ha perdido apetito si bien come como siempre pero forzándose, por lo que no ha perdido peso. Pierde el hilo de las conversaciones y está irritable. Ha dejado de hacer las cosas que le gustaba hacer y no sale de casa más que lo justo. Viene acompañada por su hermana y nos ruega que no le digamos nada a su marido ya que desde que no hace las cosas de la casa tan bien como antes no quiere darle más disgustos. ¿Cuál es el diagnóstico más probable?: A) Ansiedad y depresión. B) Demencia. C) Depresión mayor. D) Trastorno de personalidad.
  • 12. Examen Simulacro :: Ciclo de Revisión en Medicina :: Sábado 4 de Agosto del 2012 :: www.estudiosmyc.com E) Neurosis. 98. El paciente, que es traído por un familiar, dice no saber porqué está aquí y parece irritado y con agresividad contenida. El familiar nos hace gestos con los ojos haciendo ver que el paciente no está muy bien. Entrevistando a ambos por separado el paciente se muestra contenido y dice estar bien, mejor que nunca. El familiar dice que últimamente ha comprado cosas innecesarias y ha hecho algunos regalos. Dice que quiere iniciar la carrera de psicología, se ha apuntado a un gimnasio y quiere proponer a su jefe algunos cambios en la empresa si bien no tiene responsabilidades directivas. Está muy hablador y agudo en las conversaciones pero irritable, dominante y con explosiones de mal genio. Debe dormir 4 horas pero no está cansado al día siguiente, no hay problemas de apetito. No ha habido problemas en su trabajo, pero pasa mucho tiempo en la calle. ¿Cuál es el diagnóstico más correcto?: A) Esquizofrenia. B) Manía. C) Depresión ansiosa. D) Trastorno esquizoafectivo. E) Trastorno de personalidad. 99. Paciente traído por la policía por haberlo encontrado andando descalzo por la carretera. El paciente tiene 28 años, es de otra ciudad y por la documentación que lleva parece que pueda ser estudiante. Parece deshidratado, completamente desaseado y con olor a orín. Pupilas normales. La piel está tostada sin quemaduras en las zonas de exposición al sol. Está ausente, como absorto de manera que al hablarle más fuerte se sobresalta y nos atiende momentáneamente sonriéndonos de forma inapropiada hasta que ladea la cabeza y la gira sin motivo. Murmura algo y mira al techo, permaneciendo con los brazos extendidos. ¿Cuál es el diagnóstico más probable?: A) Intoxicación por LSD. B) Esquizofrenia. C) Demencia. D) Cuadro maníaco. E) Histeria. 100. Acude la madre de un presunto paciente de 18 años porque su hijo no sale casi nada de casa, parece evitarles. Se hace su propia comida a partir de conservas generalmente y come aparte. Nunca usa conservas o alimentos ya empezados, ni acaba los que no usa por completo. Alguna vez lo ha visto en el pasillo haciendo posturas extrañas o dando golpes de kárate. Fue aficionado a las lecturas de ovnis y ciencia ficción. Fue regular estudiante. No trabaja ni estudia. Cierra todas las persianas y se queja de que los vecinos están demasiado pendientes de ellos. Por las noches a veces no duerme. Dice que puede oír al vecino del tercero (ellos viven en el primero) hablar mal de ellos y que siempre está comentando lo que él va haciendo. ¿Cuál es el diagnóstico más probable?: A) Esquizofrenia. B) Depresión psicótica. C) Manía. D) Demencia. E) Klein-Levin. TEMA B 1. Paciente de 64 años ingresado para cirugía diferida que al día siguiente del ingreso presenta cuadro de hiperactividad vegetativa, temblor, sobresaltos, hiperactividad e hiperalerta pero con confusión, alucinaciones y convulsiones. ¿Cuál es la causa más frecuente?: A) Abstinencia a alcohol. B) Sobredosis de neurolépticos. C) Sobredosis de benzodiacepinas. D) Sobredosis de anticolinérgicos. E) Ingestión de barbitúricos. 2. Mujer de 25 años que acude por intento autolítico con benzodiacepinas. Es el quinto intento en 7 años, en esta ocasión tras una pelea familiar. Su madre dice que es muy voluble y caprichosa que siempre consigue salirse con la suya mediante grandes crisis de nervios y peleas. La madre dice estar harta ya que siempre está regañándola mientras que su marido la perdona todo y no la castiga lo suficiente. No tiene novio fijo ya que rompe con ellos en seguida, es mala estudiante y no es capaz de dedicarse a nada fijo ya que se aburre en seguida, dice sentirse en ocasiones muy vacía y en otras muy inspirada en hacer el bien pero nada le dura mucho, quisiera poder tener una relación con un hombre romántico. Sale mucho por la noche y en ocasiones no llega hasta el día siguiente por la tarde, llegando ebria en ocasiones. Ha perdido a sus amigas, que se quejaban entre otras cosas de que las estaba llamando continuamente. Le han diagnosticado previamente de hepatitis. El diagnóstico de esta paciente es: A) Esquizofrenia. B) Manía. C) Trastorno de personalidad límite o inestable. D) Trastorno de personalidad por dependencia. E) Trastorno esquizoide. 3. El paciente acude a nuestra consulta porque quisiera dejar de hacer algunas de las cosas que actualmente hace ya que le quitan mucho tiempo. El paciente tarda mucho tiempo en vestirse ya que ha de pensar la ropa que le dará suerte para esa mañana. Teme que si no coge la adecuada algo podría salir mal. No puede evitar sumar los números que ve en las matrículas de los coches, así consigue neutralizar los pensamientos que le vienen sobre la posibilidad de que el coche tenga un accidente. El sabe que éstos son tonterías suyas pero son cosas que no puede quitarse de la cabeza. Es muy ordenado en algunos aspectos pero se le acumula el
  • 13. Examen Simulacro :: Ciclo de Revisión en Medicina :: Sábado 4 de Agosto del 2012 :: www.estudiosmyc.com trabajo y no consigue hacer las cosas que tiene preparadas en una lista para ese día. Le gusta corregir sus escritos hasta que estén impecables, lo mismo le ocurre con su trabajo, es muy perfeccionista, exigente consigo mismo y con los demás, es incapaz de delegar trabajo. El diagnóstico del paciente es: A) Esquizofrenia. B) Trastorno de personalidad dependiente. C) Trastorno obsesivo-compulsivo. D) Fobia. E) Depresión. 4. Paciente que acude por no poder salir de casa sola. Dice que le da miedo salir por sitios solitarios por si le ocurre algo y que en los sitios con mucha gente se agobia y se pone muy nerviosa teniendo crisis de ansiedad con palpitaciones, se imagina así misma necesitada de ayuda y sin poder salir. Si se encuentra acompañada se encuentra mejor ya que esa persona podría ayudarla si ocurriera algo. Desde hace unos meses tiene unas crisis parecidas pero más cortas estando relajada, incluso acompañada, sin aviso, la primera estando en la calle distraída viendo un escaparate. Cuando le dan en casa es capaz de salir a la calle para coger aire o bien abre las ventanas. Desde la primera crisis cogió algo de miedo a estar sola hasta llegar a la situación actual. Se ha comprado un teléfono móvil para poder pedir ayuda si fuera necesario. Si entra en un cine siempre está pendiente de sentarse cerca de alguna salida. El cuadro clínico de la paciente es: A) Agorafobia. B) Hipocondría. C) Trastorno facticio. D) Trastorno obsesivo. E) Agorafobia con crisis de pánico. 5. Ante un paciente con dolor epigástrico irradiado en hemicinturón, náuseas, distensión abdominal, descenso de los ruidos hidroaéreos y ascenso del ST en el electrocardiograma, ¿cuál sería su actitud?: A) Llamaría de inmediato a la Unidad de Cuidados Intensivos para tratar el infarto agudo de miocardio. B) Repetiría el electrocardiograma a las 8 horas para confirmar el diagnóstico. C) Lo diagnosticaría de pancreatitis aguda. D) Su diagnóstico sería aneurisma disecante de aorta. E) El paciente tiene una pericarditis aguda. 6. Ingresa en el Servicio de Urgencias un paciente con dolor epigástrico irradiado en hemicinturón y amilasa elevada, que pierde de forma brusca visión ¿qué pensaría que ha ocurrido?: A) Retinopatía de Purtscher. B) Accidente cerebro vascular agudo de la región occipital posterior. C) Desprendimiento de retina. D) Simulación. E) Glaucoma agudo. 7. Ante una paciente que ingresa en el Servicio de Urgencias con dolor abdominal, elevación de la amilasa y la glucosa en suero y con un pH de 7,1 ¿en qué pensaría primero?: A) Pancreatitis aguda. B) Rotura de embarazo ectópico. C) Quiste de ovario. D) Cetoacidosis diabética. E) Ulcera de estómago. 8. Un paciente de 18 años consulta por ictericia sin fiebre y coluria sin prurito. En las pruebas complementarias presenta: Ac. anti VHA IgG positivos, Ac. antiHBs positivos, Ac. anti HBc IgG positivos, ecografía hepática normal, hay predominio de la bilirrubina directa con monoconjugados, la colecistografía oral es normal y al dar al paciente fenobarbital se observa disminución de las cifras de bilirrubina total. ¿Cuál sería su diagnóstico de sospecha?: A) Coledocolitiasis. B) Síndrome de Gilbert. C) Síndrome de Dubin-Johnson. D) Hepatitis aguda por VHA, con hepatitis aguda por VHB curada. E) Síndrome de Rotor. 9. Ante un paciente con cirrosis alcohólica en estadío C- 10 de Child-Pugh que ingresa por descompensación hidrópica, y que se encuentra hipotenso, taquicárdico, oligúrico, con un sodio sérico de 125 mEq/l. y de 5 mEq/l. en orina, con sedimento normal, creatinina sérica 3 mg./dl. y un aclaramiento de creatinina de 40 ml., ¿en qué pensaría usted?: A) Deficiente tratamiento diurético. B) Insuficiencia renal prerrenal secundaria a tercer espacio. C) Síndrome hepatorrenal. D) Glomerulonefritis mesangial IgA. E) Insuficiencia renal prerrenal secundaria a deshidratación. 10. Paciente varón de 28 años, VIH(+), en tratamiento con DDI (Dideoxinosina), que acude al hospital por dolor intenso en epigastrio irradiado hacia la espalda, que mejora al flexionar el tronco, acompañado de náuseas y vómitos. ¿Cuál de los siguientes diagnósticos es el más probable?: A) Pericarditis. B) Obstrucción intestinal. C) Apendicitis. D) Pancreatitis aguda. E) Endocarditis. 11. Tras una semana de ingreso por una pancreatitis aguda, a pesar del tratamiento médico sintomático persiste la fiebre, leucocitosis e hiperamilasemia. Se aprecia a la palpación una masa abdominal localizada en hipocondrio derecho. ¿Cuál es el diagnóstico más probable?:
  • 14. Examen Simulacro :: Ciclo de Revisión en Medicina :: Sábado 4 de Agosto del 2012 :: www.estudiosmyc.com A) Carcinoma de páncreas. B) Flemón pancreático. C) Pancreatitis crónica. D) Plastón secundario a perforación duodenal. E) Quiste hidatídico. 12. El principal diagnóstico de presunción ante una enferma que acude por un cuadro de diarrea acuosa, dos úlceras duodenales resistentes al tratamiento médico e hipercalcemia es: A) Insulinoma. B) Gastrinoma. C) Adenocarcinoma de páncreas. D) Somatostatinoma. E) Vipoma. 13. Varón de 60 años que acude por pérdida de peso de 10 kilos, dolor sordo en piso abdominal superior de 3 meses de evolución, acompañado de ictericia mucocutánea y deposición de color blanco desde hace 5 días. A la exploración destaca masa palpable en hipocondrio derecho. ¿Qué patología se sospecharía en primer lugar?: A) Adenocarcinoma pancreático. B) Tumor gástrico. C) Coledocolitiasis. D) Colecistitis. E) Hepatitis aguda. 14. Varón de 45 años, etilismo crónico, con dolor abdominal en el hipocondrio izquierdo, de 3 meses de evolución, que empeora con la ingesta y se acompaña de deposiciones diarreicas pastosas muy mal olientes. En las exploraciones complementarias destaca: glucemia de 280, amilasemia en los límites de la normalidad y en la placa de abdomen múltiples calcificaciones a nivel de L2: A) Pancreatitis aguda. B) Cólico biliar. C) Pancreatitis crónica. D) Hepatitis aguda. E) Ulcus gástrico 15. Paciente de 60 años con fibrilación auricular en tratamiento con amiodarona desde hace meses, colelitiasis diagnosticada ecográficamente e insuficiencia renal crónica moderada. Consulta por cuadro de anorexia, astenia, náuseas, vómitos e ictericia mucocutánea. En los datos de laboratorio destaca un leve aumento de las transaminasas y una creatinina de 2,2. Se realiza una biopsia hepática, observándose al microscopio electrónico cuerpos lisosómicos lamelares cargados de fosfolípidos. Qué proceso patológico le sugieren estos datos: A) Hepatitis viral aguda. B) Cólico biliar. C) Pancreatitis aguda. D) Hepatitis tóxica de origen medicamentoso. E) Hepatitis viral crónica. 16. Un varón de 13 años que había presentado un cuadro febril de vías respiratorias altas de una semana de evolución acude a urgencias por vómitos súbitos e incoercibles. La semana previa había consumido ácido acetilsalicílico para la sintomatología respiratoria y tres semanas antes estuvo en contacto con un paciente con hepatitis aguda por virus B. Dos días después del ingreso el paciente se encuentra estuporoso con convulsiones sin signos neurológicos de focalidad y dolor en hipocondrio derecho con hepatomegalia. ¿Cuál sería su diagnóstico?: A) Cuadro convulsivo en relación con la fiebre. B) Hepatitis fulminante vírica. C) Síndrome de Reye. D) Reagudización del cuadro gripal. E) Intoxicación por ácido acetilsalicílico. 17. Un hombre de 21 años recibió un golpe en el escroto dos horas antes de ser examinado en urgencias. Su escroto está tenso, hinhado, y equimótico. No se pueden palpar los testículos. El paso siguiente es: A) Hacer uretrografía retrógrada. B) Hacerle un Doppler de flujo color. C) Realizar una ecografía del escroto. D) Tratarle con hielo, reposo y suspensorio. E) Hacer exploración quirúrgica del escroto. 18. Un hombre de 64 años presenta una hinchazón indolora del testículo derecho de tres meses de duración. Los resultados de los análisis de orina son normales y la ecografía testicular muestra un aumento del tamaño de dicho teste. El diagnóstico más probable es: A) Linfoma testicular. B) Leucemia linfática crónica. C) Seminoma espermatocítico. D) Teratocarcinoma. E) Carcinoma de células embrionarias. 19. Una mujer sana presenta de forma aguda polaquiuria y disuria. En el sedimento urinario se observan más de 5 leucocitos por campo y el urinocultivo revela 1.000 colonias de E. coli por ml. El diagnóstico más probable es: A) Uretritis por clamydia. B) Síndrome uretral. C) Bacteriuria por E. coli. D) Cistitis intersticial. E) Cistitis quística. 20. Un hombre de 74 años con cáncer de próstata metastásico conocido presenta dolor agudo de cadera derecha. Hace dos años se le practicó una orquiectomía pero no ha recibido ningún otro tratamiento. Su estado general es bueno. Una gammagrafía ósea muestra metástasis difusas y una Rx simple revela una osteólisis en el acetábulo derecho. La siguiente medida a aplicar es: A) Flutamida.
  • 15. Examen Simulacro :: Ciclo de Revisión en Medicina :: Sábado 4 de Agosto del 2012 :: www.estudiosmyc.com B) Análogos de la LH-RH. C) Radioterapia localizada. D) Fosfato de estramustina. E) Ketoconazol. 21. Un recién nacido presenta hematuria, proteinuria y creatinina elevada. Las presiones de la arteria umbilical están significativamente altas y el paciente desarrolla una insuficiencia cardíaca congestiva. Una gammagrafía renal revela una ausencia de función en el riñón izquierdo. El diagnóstico más probable es: A) Trombosis de la vena renal. B) Necrosis cortical renal. C) Hemorragia adrenal. D) Rotura de un nefroma mesoblástico. E) Trombosis de la arteria renal. 22. Un hombre de 35 años presenta un cólico renal izquierdo. En la Rx de abdomen se observa un cálculo de 3 mm. de diámetro alojado en uréter medio. La necesidad de tratamiento quirúrgico depende de: A) La duración de los síntomas del paciente. B) Del número de episodios de cólicos previos. C) Del número de intervenciones quirúrgicas previas. D) De la presencia de fiebre e infección urinaria. E) De una anormalidad metabólica subyacente. 23. A un paciente se le somete a una intervención de bypass en el intestino delgado y presenta una litiasis urinaria. ¿Cuál será la composición más probable de la litiasis?: A) Acido úrico. B) Urato amónico. C) Fosfato cálcico. D) Oxalato cálcico. E) Estruvita. 24. El factor pronóstico más importante en los niños que presentan un tumor de Wilms intracava es: A) La histología. B) El volumen del tumor. C) La extensión atrial del tumor. D) La afectación de ganglios linfáticos. E) La diseminación del tumor durante la intervención quirúrgica. 25. Un muchacho de 7 años presenta de forma súbita dolor escrotal derecho de 4 horas de duración. Se sospecha de una torsión testicular. ¿Cuál de las siguientes observaciones es la más probable?: A) La ausencia del reflejo cremastérico. B) El aumento de la captación por parte del teste derecho después de la exploración radioisotópica. C) La presencia del reflejo cremastérico. D) Una transiluminación correcta del compartimento escrotal derecho. E) La estetoscopia con Doppler será simétrica en ambos compartimentos escrotales. 26. En un paciente asmático, ¿cuál de los siguientes fármacos está contraindicado en el tratamiento de la incontinencia?: A) La efedrina. B) Sudafed. C) El propranolol. D) La fenilefrina. E) Las anfetaminas. 27. Ante un paciente de 60 años con un cáncer de próstata localizado, mal diferenciado y sin metástasis, qué tratamiento le recomendaría para intentar aumentar su supervivencia: A) Prostatectomía radical. B) Análogos de la LH-RH. C) Flutamida. D) Estramustina. E) Orquiectomía. 28. Ante un paciente de 55 años que presenta una tumoración vesical que infiltra la muscular y que ocupa la mitad de la vejiga, ¿cuál sería el tratamiento que emplearía con intención curativa?: A) Cistectomía radical y derivación urinaria. B) Resección transuretral vesical. C) Quimioterapia intravesical. D) Quimioterapia sistémica. E) Cistectomía parcial. 29. Paciente mujer de 75 años de edad, que es traída a la consulta por cambio progresivo de conducta en los últimos meses. Previamente habían observado fallo de memoria reciente sin poder precisar el momento de inicio. Destacaba a la exploración lenguaje pobre y fallos de juicio. ¿Cuál es el diagnóstico más probable en esta paciente?: A) Depresión. B) Síndrome confusional agudo. C) Demencia tipo Alzheimer. D) Demencia multiinfarto. E) Trastorno de la personalidad. 30. Varón de 87 años, con amaurosis bilateral, que ingresa por neumonía basal derecha en la Unidad de Agudos del hospital. La noche del ingreso presenta cuadro de agitación psicomotriz, con alteración del nivel de conciencia y desorientación temporoespacial. ¿Cuál es el diagnóstico más probable?: A) Demencia. B) Depresión delirante. C) ACVA. D) Síndrome confusional agudo. E) Ninguno. 31. Paciente de 85 años de edad con antecedentes de insuficiencia cardíaca en tratamiento con diuréticos, e insomnio que trataba con lorazepam. Presenta nicturia 2-3 veces. Sufre caída al levantarse bruscamente durante la noche para ir al retrete. ¿Qué factores pueden haber contribuido e la caída?:
  • 16. Examen Simulacro :: Ciclo de Revisión en Medicina :: Sábado 4 de Agosto del 2012 :: www.estudiosmyc.com A) Fármacos. B) Hipotensión postural. C) Factores ambientales. D) Ninguno. E) Todos ellos. 32. Paciente de 75 años de edad, varón, que vive solo. Es encontrado la mañana del día 15 de enero caído en el suelo. A su llegada al hospital se evidencia afasia motora y hemiplejía derecha diagnosticándose de ACVA. ¿Cuál de las siguientes patologías debe ser descartada en la valoración inicial?: A) Rabdomiólisis. B) Hipertiroidismo. C) Depresión. D) Hipotermia. E) A y D. 33. Paciente mujer de 85 años que ingresa por fractura pertrocantérea de cadera derecha colocándose tracción. Se evidencia neumonía basal derecha que obliga a retrasar la intervención. La paciente sufre de incontinencia urinaria que se maneja con catéter. A los cinco días se observa enrojecimiento en región sacra que no palidece con la presión, diagnosticándose de úlcera por presión grado I. ¿Cuál es el principal factor de riesgo para esta complicación?: A) Incontinencia urinaria. B) Hipoxemia. C) Edad avanzada. D) Inmovilidad. E) Todos. 34. Paciente de 70 años que sufre caída al suelo golpeándose en la cabeza. Al cabo de unas semanas sufre trastornos de conducta, pérdida de memoria y posteriormente alteración del nivel de conciencia. ¿Qué diagnóstico debe descartarse en primer lugar?: A) Hipotiroidismo. B) Demencia tipo Alzheimer. C) Pseudodemencia. D) Hematoma subdural. E) Tumor cerebral. 35. Mujer de 75 años que consulta por incontinencia urinaria, en la que predomina la urgencia-miccional y que no sigue ningún tratamiento farmacológico habitual. A través de la exploración física no se objetiva patología orgánica, siendo el residuo postmiccional normal. El estudio analítico es normal. El tratamiento médico de elección sería: A) Sondaje vesical intermitente. B) Calcioantagonistas. C) Cirugía. D) Antocolinérgicos. E) Colector externo. 36. Durante los últimos 3 meses un varón de 80 años presenta una rápida progresión de una demencia acompañada de signos extrapiramidales y mioclonías. El diagnóstico de sospecha inicial sería: A) Demencia senil tipo Alzheimer. B) Enfermedad de Creutzfeldt-Jakob. C) Enfermedad de Huntington. D) Hidrocefalia a presión normal. E) Enfermedad de Parkinson. 37. ¿Cuál de las lesiones cutáneas siguientes se asocia más frecuentemente con neoplasia oculta en un paciente anciano?: A) Penfigoide bulloso. B) Dermatomiositis. C) Eritema multiforme. D) Herpes zoster. E) Pénfigo vulgar. 38. Anciano de 70 años con un melanoma maligno de reciente diagnóstico y sin otra patología médica. Su pronóstico vital estará más estrechamente relacionado con una de las siguientes características: A) Ausencia de regresión. B) Tipo histológico de la lesión. C) Grado de invasión. D) Presencia de ulceración. E) Lugar de la lesión. 39. Una mujer de 75 años con historia de diabetes mellitus no insulindependiente y epilepsía secundaria a enfermedad cerebro-vascular padece una inflamación con retracción gingival. Se encuentra en tratamiento con glipizida y fenobarbital. La causa más probable de su proceso gingival será: A) Caries dental. B) Edentulismo. C) Tratamiento con fenobarbital. D) Déficit de cinc. E) Pobre higiene oral. 40. Un varón de 76 años con historia de diabetes mellitus de larga evolución y con datos clínicos de polineuropatía periférica, empieza a tomar amitriptilina por prescripción médica para las parestesias en miembros inferiores. De forma rogresiva nota disminución del volumen de diuresis y ocasionalmente escapes involuntarios de orina. En la revisión médica siguiente se objetiva deterioro del estado general con insuficiencia renal. ¿Cuál es la causa más lógica de su deterioro clínico?: A) Nefrotoxicidad por amitriptilina. B) Pielonefritis aguda. C) Infección urinaria de vías bajas. D) Retención urinaria con fracaso renal secundario. E) Glomeruloesclerosis diabética.
  • 17. Examen Simulacro :: Ciclo de Revisión en Medicina :: Sábado 4 de Agosto del 2012 :: www.estudiosmyc.com 41. Varón de 45 años con otalgia derecha y sensación de taponamiento auditivo, sin otorrea. A los dos días presenta aumento del dolor, que se hace retroauricular, y fiebre en agujas. El Rinne es negativo en oído derecho y el Weber lo lateraliza a la derecha. El diagnóstico más probable es: A) Colesteatoma antral invasivo. B) Petrositis. C) Tromboflebitis del seno lateral. D) Otitis externa maligna. E) Carcinoma de oído derecho. 42. Mujer de 22 años que presenta parálisis facial periférica derecha. A la exploración presenta otoscopia normal y lengua geográfica fisurada. La paciente comenta haber tenido otro episodio anteriormente, aquella vez asociado a edema de labio inferior. El diagnóstico más probable es: A) Síndrome de Guillain-Barré. B) Síndrome de Heerfordt-Waaldenström. C) Déficit de C1-inhibidor. D) Síndrome de Melkerson-Rosenthal. E) Parálisis facial de Bell. 43. Paciente que presenta hipoacusia neurosensorial izquierda de larga evolución con caída en agudo, y acúfeno persistente. La actitud correcta en este caso sería: A) No hacer nada, pues se trata de un traumatismo acústico crónico. B) Potenciales evocados para descartar neurinoma del VIII. C) Instaurar terapia vasodilatadora endovenosa, pues se trata de una hipoacusia súbita. D) Intervenir el posible colesteatoma. E) Administrar sedantes vestibulares ante la posibilidad de vértigo de Ménière. 44. Paciente de 65 años con rinorrea unilateral purulenta, dolor hemifacial y epistaxis ocasionales. En Rx se observan lesiones osteolíticas en maxilar. Probablemente se trate de: A) Cuerpo extraño nasal. B) Ocena. C) Granuloma sangrante de tabique. D) Carcinoma de fosa nasal. E) Coriza. 45. Un varón de 60 años consulta por tumoración indolora en raíz nasal, que desplaza la órbita produciéndole diplopia. El cuadro se debe probablemente a: A) Mucocele etmoidal. B) Quiste de retención en seno frontal. C) Enfermedad de Woakes. D) Papiloma invertido en techo de fosa nasal. E) Plasmocitoma solitario en seno frontal. 46. Una mujer de 45 años acude a la urgencia con un síndrome de disnea y estridor importantes, con tiraje supraclavicular. A la exploración presenta taquicardia y leve cianosis, un tiroides agrandado y, en la laringoscopia, parálisis de ambas cuerdas vocales en posición paramediana. La actitud que debe seguirse es: A) Bolo de corticoides endovenosos, ante la posibilidad de carcinoma subglótico. B) Tiroidectomía de urgencia, pues probablemente el tiroides agrandado comprime la tráquea. C) Intubación y observación. D) Traqueostomía de urgencia, pues se trata de una parálisis recurrencial bilateral secundaria a patología tiroidea. E) Administración endovenosa de espasmolíticos. 47. Un niño de 3 años es traído a urgencias con un cuadro de fiebre y tos irritativa "perruna", a lo que se asocia disfonía y cierto grado de disnea. El cuadro es compatible con todas menos: A) Cuerpo extraño en vías aéreas. B) Laringotraqueítis aguda. C) Epiglotitis aguda. D) Adenoamigdalitis aguda obstructiva. E) Edema alérgico. 48. Paciente mujer de 25 años que consulta por pérdida de audición de comienzo insidioso. Su madre era sorda y ganó audición tras operarse del oído. Otoscopia normal. Rinne negativo oído izdo., positivo oído dcho. Weber a la izda. Audiometría: hipoacusia transmisiva izquierda. Reflejo estapedial abolido en oído izdo. y presente en derecho. Timpanometría normal. El diagnóstico probable es: A) Otosclerosis oído izquierdo. B) Colesteatoma izquierdo. C) Fijación de cadena osicular derecha. D) Timpanosclerosis izquierda. E) Luxación de cadena osicular izquierda. 49. Un varón de 30 años acude a urgencias por vértigo periférico intenso con nistagmus a la derecha y acúfeno en oído izquierdo. Recuerda haber tenido déficit auditivo izquierdo previo, que le desapareció. Audiometría: hipoacusia neurosensorial izda. leve. Reflejo estapedial: derecho en 70 dB; izquierdo en 50 dB. Diagnóstico probable: A) Neuronitis vestibular. B) Neurinoma del VIII par con reclutamiento positivo. C) Tumor de tronco cerebral. D) Otitis media secretora con fístula perilinfática. E) Síndrome de Ménière. 50. Un niños de 2 años presenta rinorrea purulenta de larga evolución por fosa nasal derecha, con mala ventilación nasal. Son diagnósticos posibles todos menos: A) Tuberculosis nasal. B) Cuerpo extraño intranasal. C) Sinusitis maxilar. D) Coriza común. E) Rinitis alérgica sobreinfectada.
  • 18. Examen Simulacro :: Ciclo de Revisión en Medicina :: Sábado 4 de Agosto del 2012 :: www.estudiosmyc.com 51. Paciente de 30 años que acude a urgencias presentando una ulceración amigdalar unilateral. Son diagnósticos probables todos menos: A) Amigdalitis tifoidea de Duguet. B) Angina de Ludwig. C) Carcinoma escamoso de amígdala. D) Angina de Plaut-Vincent. E) Agranulocitosis. 52. Un niños de 12 años acude a urgencias por obstrucción nasal bilateral crónica y otitis seromucosa bilateral. Recientemente ha tenido epistaxis importante por ambas fosas nasales. El diagnóstico más probable es: A) Hipertrofia adenoidea. B) Ototubaritis asociada a rinitis alérgica. C) Poliposis nasal bilateral. D) Adenocarcinoma de etmoides. E) Angiofibroma nasofaríngeo. 53. Mujer de 75 años con historia de cefalea presenta pérdida súbita de visión unilateral con edema de papila ipsilateral. ¿Qué medida tomaríamos en primer lugar?: A) TAC. B) VSG. C) Ingreso hospitalario paratratamiento antibiótico iv. D) Observación. E) Radiografía centrada en el agujero óptico. 54. Mujer de 20 años con ojo rojo bilateral, acompañado de quemosis, folículos conjuntivales tarsales, adenopatía preauricular, sin pérdida de visión. La etiología más frecuente será: A) Adenovirus. B) S. aureus. C) H. influenzae. D) Queratitis herpética. E) Parainfluenzae virus. 55. Mujer de 23 años que presenta pérdida brusca de agudeza visual indolora en ojo derecho, en la exploración se observa defecto pupilar aferente relativo en ojo derecho, segmento anterior normal y F.O. normal. El diagnóstico más probable es: A) Neuritis óptica retrobulbar. B) Histeria. C) Compresión quiasmática por tumor hipofisario. D) Compresión del globo ocular por tumor orbitario. E) Glaucoma agudo de ángulo cerrado. 56. Varón de 50 años con ojo rojo y doloroso, midriasis arreactiva y cámara anterior estrecha. ¿Cuál de las siguientes respuestas es falsa?: A) No sería raro que fuera hipermétrope. B) Puede presentar cefalea con náuseas y vómitos. C) Evitar tomar la PIO por posible etiología infecciosa. D) El tratamiento definitivo es láser o cirugía. E) Puede presentar visión en halos de colores. 57. Ante un varón de 32 años con síndrome febril y pérdida visual unilateral que presenta en fondo de ojo lesiones en queso y tomate. ¿Cuál de las siguientes afirmaciones es falsa?: A) Sería aconsejable realizar serología HIV. B) El pronóstico vital del enfermo es malo. C) No necesita tratamiento por ser con frecuencia una alteración transitoria. D) Con bastante probabilidad se deberá a CMV. E) La afectación suele ser bilateral. 58. Paciente de 70 años que presenta metamorfosias, micropsia y disminución de la agudeza visual de varias semanas de evolución, el diagnóstico más probable es: A) Catarata nuclear. B) Catarata subcapsular posterior. C) Degeneración macular senil. D) Pars planitis. E) Hemorragia vítrea. 59. Ante un niño de 5 años con endotropía que ha seguido tratamiento con corrección óptica y colusiones y cuya agudeza visual aún no es normal. La pauta a seguir será: A) Corrección quirúrgica de la endotropía. B) Continuar las oclusiones sobre ojo con mejor visión. C) Continuar las oclusiones sobre ojo con peor visión. D) No ocluir más y pasar a otro tratamiento. E) Observación. 60. Varón de 70 años que presenta pérdida visual progresiva unilateral, que precisa cambios sucesivos de corrección óptica miópica. La causa más probable es: A) Degeneración macular senil. B) Glaucoma crónico simple. C) Vitritis senil. D) Coriorretinosis senil. E) Catarata nuclear. 61. Varón de 51 años que sufre súbita pérdida visual total e indolora en ojo derecho, apreciándose en el F.O. una retina de color blanquecino con mácula rojo-cereza. El diagnóstico más probable es: A) Desprendimiento de retina con afectación macular. B) Enfermedad de Tay-Sacks. C) Enfermedad de Newman-Pick. D) Obstrucción de arteria central de la retina. E) Obstrucción de vena central de la retina. 62. Varón de 10 años de edad que presenta estrabismo, disminución de agudeza visual y leucocoria en ojo derecho, sin malformaciones oculares. ¿Qué patología debemos descartar como más probable?: A) Retinoblastoma. B) Catarata congénita. C) Vítreo primario hiperplásico persistente. D) Enfermedad de Coats. E) Fibroplasia retrolental.
  • 19. Examen Simulacro :: Ciclo de Revisión en Medicina :: Sábado 4 de Agosto del 2012 :: www.estudiosmyc.com 63. Paciente de 65 años con Diabetes mellitus tipo II de 10 años de evolución con mal control metabólico presenta disminución de visión brusca unilateral. La causa más probable será: A) Isquemia macular. B) Desprendimiento de retina traccional. C) Hemorragia vítrea. D) Edema macular. E) Obstrucción de arteria central de la retina. 64. Varón de 35 años sano de carácter nervioso comienza con metamorfopsias y escotoma central unilateral, disminución de visión moderada y patrón angiográfico en chimenea. El diagnóstico más probable será: A) Distrofia viteliforme de Best. B) Retinosis pigmentaria. C) Degeneración macular ligada a la edad. D) Enfermedad de Coats. E) Coriorretinopatía central serosa. 65. Una mujer de 35 años, no fumadora, atleta de fondo y sin antecedentes personales de interés refiere en los últimos 3 meses una menor respuesta al ejercicio habitual, con disnea de moderados esfuerzos. Tos no productiva pero niega fiebre. En una ocasión reciente presentó la emisión de varios esputos hemoptoicos, que no se han vuelto a repetir. La Rx de tórax muestra un patrón reticular fino difuso bilateral y un mínimo derrame pleural derecho. En las pruebas funcionales llama la atención el incremento de los volúmenes pulmonares. De los siguientes, ¿cuál le parece el diagnóstico más probable?: A) Lupus eritematoso sistémico. B) Tuberculosis. C) Sarcoidosis. D) Linfangioleiomatosis. E) Enfermedad de Hamman-Rich. 66. Varón de 50 años, no fumador, que refiere disnea de moderados esfuerzos de unos 4 meses de evolución con tos no productiva. Ha recibido tratamiento con diuréticos de asa (furosemida) en el último mes tras realizarse una Rx de tórax (que no aporta). Acude a urgencias por incremento de la disnea y expectoración de esputos claros. Refiere febrícula de predominio vespertino y pérdida de unos 6 Kg de peso. Presenta hipoxemia con hipocapnia (insuficiencia respiratoria parcial) y la Rx de tórax muestra densidades difusas bilaterales, confluentes, mal definidas de predominio parahiliar con un índice cardiotorácico en el límite de la normalidad. FVC: 65%, FEV1: 70%, FEV1/FVC: 75%. DLCO: 60%. A) Insuficiencia cardiaca. Edema agudo de pulmón. B) Enfermedad de Hamman-Rich. C) Hemorragia pulmonar. D) Neumonía por CMV. E) Carcinoma bronquioalveolar. 67. Varón de 64 años, veterinario de profesión, bebedor esporádico que presenta fiebre, cefalea y artromialgias desde hace 7 días. Durante las últimas 48 horas refiere tos productiva, dolor pleurítico en el costado derecho y disnea progresiva. Ha sido tratado con eritromicina, pese a lo que se encuentra febril y desorientado. Se evidencian estertores crepitantes en la base derecha y una hepatomegalia a 4 cm del reborde costal. En las pruebas complementarias efectuadas, destaca Hb de 14.8 mg/dL, leucocitosis (17.000/μL) con desviación izquierda, GOT (AST): 106, GPT (ALT): 82. En la Rx de tórax se aprecia un infiltrado intersticial en lóbulo inferior derecho. Su diagnóstico de presunción debe ser: A) Neumonía neumocócica. B) Neumonitis por hipersensibilidad. C) Fiebre Q. D) Legionella. E) Tuberculosis. 68. Varón de 37 años que viene presentando durante los tres últimos meses astenia, esputos hemoptoicos y disnea progresiva hasta hacerse de mínimos esfuerzos, con intolerancia al ejercicio. Salvo una TA de 90/60, los datos exploratorios son anodinos. Sin embargo, los estudios complementarios nos sorprenden: pH: 7.37, PaO2: 62, PaCO2: 37, HCO3: 27. Hb: 9.2 y creatinina de 2.3. La Rx de tórax muestra infiltrados difusos parahiliares bilaterales. Ante los hallazgos reseñados, se añade la petición de un sedimento de orina, que muestra microhematuria y proteinuria. De las siguientes, ¿qué prueba diagnóstica le parece prioritaria en la evaluación del enfermo?: A) Test de difusión (DLCO). B) Examen citológico (esputo o lavado broncoalveolar). C) Anticuerpos anti membrana basal glomerular, c- ANCA. D) Biopsia pulmonar. E) Biopsia renal. 69. En el caso anterior, la capacidad de difusión del CO está aumentada y el título de anticuerpos antimembrana basal glomerular es de 1:128. ¿Cuál le parece el diagnóstico más probable?: A) Granulomatosis de Wegener. B) Síndrome de Goodpasture. C) Tuberculosis. D) Tromboembolismo pulmonar. E) Granulomatosis de Churg-Strauss. 70. Mujer de 78 años que en el curso de un postoperatorio por fractura de cadera comienza con un cuadro brusco de disnea y febrícula. Exploración: taquipnea a 30 r.p.m., taquicardia a 130 l.p.m., refuerzo del segundo tono, abolición del murmullo vesicular en base de pulmón derecho y extremidades sin edemas, no dolorosas, sin signos flogóticos. Complementarios: GAB: pH: 7.52, PaO2: 56, PaCO2: 30, HCO3: 25. 13.000 leucocitos con desviación izquierda. Rx de tórax: pinzamiento del seno costodiafragmático derecho. ECG: Taquicardia sinusal con bloqueo
  • 20. Examen Simulacro :: Ciclo de Revisión en Medicina :: Sábado 4 de Agosto del 2012 :: www.estudiosmyc.com incompleto de rama derecha. Señale la actitud más adecuada: A) Diuréticos. B) Toracocentesis. C) Antibióticos. D) Corticoides. E) Heparina. 71. Mujer de 37 años, fumadora, que consulta por presentar durante el último mes fiebre, malestar general, artralgias y una erupción cutánea dolorosa en ambas piernas. Complementarios. Rx de tórax: adenopatías hiliares bilaterales, sin afectación del parénquima pulmonar. Mantoux negativo. Se realizó una FBC con lavado broncoalveolar (LBA). Líquido del LBA: 22% de linfocitos con cociente CD4/CD8 de 5.2. El diagnóstico más probable es: A) Tuberculosis. B) Sarcoidosis. C) Linfoma. D) Cáncer de pulmón . E) Asbestosis. 72. Varón de 47 años que acude a urgencias por fiebre, tos, artralgias y rinorrea purulenta con ulceraciones de la mucosa nasal de dos semanas de evolución. Inició tratamiento antibiótico 7 días antes, al ser diagnosticado por su médico de cabecera de sinisitis (opacificación de ambos senos maxilares), sin obtener una mejoría clínica. La Rx de tórax presenta múltiples nódulos pulmonares bilaterales, algunos de ellos cavitados. En los análisis efectuados destaca un sedimento de orina con 8 hematíes por campo con algún cilindro eritrocitario. La biopsia de la mucosa nasal mostró inflamación granulomatosa con necrosis. El diagnóstico más probable es: A) Granulomatosis de Wegener. B) Granulomatosis de Churg-Strauss. C) Cáncer de cavum con metástasis pulmonares. D) Granulomatosis linfomatoide. E) Tuberculosis. 73. Un grave problema de las unidades de cuidados intensivos son las infecciones (neumonías y sepsis) por gérmenes gramnegativos multirresistentes (pseudomonas, serratias, citrobacter, morganella, acinetobacter, etc.). Ya se han identificado previamente cepas multirresistentes en nuestra UCI. A falta de un antibiograma, ¿cuál sería el tratamiento empírico de elección? A) Ceftazidima, amicamicina y vancomicina. B) Ceftriaxona y tobramicina. C) Imipenem o ciprofloxacino. D) Imipenem y amikamicina. E) Esperar hasta los resultados del antibiograma. 74. Un varón de 30 años, fumador de 20 cigarrillos/día desde los 20 a los 25 años y ex fumador desde entonces, presenta, en un reconocimiento laboral, un nódulo pulmonar solitario (NPS) de unos 2 cm. de diametro en la periferia del LSD. La Rx de tórax muestra un mediastino normal y no permite identificar calcificaciones en el NPS. El paciente se encuentra asintomático y niega la posibilidad de recuperar radiografías anteriores antes de 6 meses (por cambio de domicilio). ¿Qué actitud mantendría ante este enfermo? A) Informar de la baja probabilidad de malignidad y Rx de tórax en 3 meses. B) Fibrobroncoscopia. C) Realización de una TAC torácica. D) Realización preferente de una PAAF con control de TAC. E) Insistir en la recuperación de las Rx previas y nueva cita en la consulta entonces (6 meses). 75. En el caso anterior, la TAC no muestra nuevos datos (confirma la ausencia de calcificaciones, no adenopatías ni afectación mediastínica y no existe afectación pleural). La familia del enfermo ha localizado las Rx de tórax previas (14 meses antes) en la que se identifica el mismo NPS con un diámetro de 1,4 cm. ¿Cuál sería la actitud más adecuada? A) Actitud expectante y repetir pruebas de imagen en 2 meses. B) Fibrobroncoscopia con citología en las muestras obtenidas. C) Fibrobroncoscopia y biopsia transbronquial (BTB). D) PAAF con control de TAC. E) Toracotomía. 76. En caso de que la PAAF obtenga material suficiente y el resultado sea de malignidad, ¿qué tipo histológico le parece el más probable? A) Ca. epidermoide. B) Adenocarcinoma. C) Ca células pequeñas (CCP). D) Ca. bronquioalveolar. E) Carcinoide. 77. Paciente de 36 años con amenorrea de 10 semanas. Tiene un antecedente de infertilidad por factor tubárico. Refiere episodios de dolor cólico hipogástrico desde hace aproximadamente un mes. El test de embarazo en orina es positivo. La prueba diagnóstica que solicita a continuación es: A) Amniocentesis precoz. B) Ecografía. C) Laparoscopia. D) Triple screening. E) Biopsia de vellosidades coriónicas.
  • 21. Examen Simulacro :: Ciclo de Revisión en Medicina :: Sábado 4 de Agosto del 2012 :: www.estudiosmyc.com 78. Gestante de 16 semanas sin antecedentes de interés que presenta los siguientes resultados en la analítica de triple screening: alfafetoproteína 0,3 MM (disminuido), betahcg 1,7 MM (normal), riesgo estimado de T21 1/43. A continuación se le realiza: A) Ecografía. B) Amniocentesis. C) Biopsia de vellosidades coriónicas. D) Funiculocentesis. E) Fetoscopia. 79. Gestante de 26 semanas que consulta por fiebre de 39° C y dolor lumbar unilateral. La analítica de sangre presenta 16.000 leucocitos y desviación izquierda. El tratamiento indicado es: A) Abundante ingesta de líquidos. B) Analgésicos orales y abundante ingesta de líquidos. C) Analgésicos endovenosos y forzar diuresis. D) Antibióticos orales y reposo domiciliario. E) Antibióticos endovenosos intrahospitalarios. 80. Gestante de 30 semanas con aumento excesivo de peso (ganancia de 18 kg. hasta la actualidad) a la que se realiza un test de O’Sullivan que resulta patológico. La actitud médica ha de ser a continuación: A) Vigilancia fetal estricta, con registros semanales de la frecuencia cardíaca fetal. B) Controles de glucemia capilar (BMtest) en desayuno, comida y cena. C) Ecografías seriadas para diagnosticar a la mayor brevedad posible un hidramnios o macrosomía fetal. D) Confirmar el diagnóstico mediante una prueba de tolerancia oral a la glucosa. E) Tratar con insulina rápida según los resultados del test de O’Sullivan. 81. Gestante de 9 semanas que consulta por metrorragia menor que una regla y dolor abdominal. Todavía no ha acudido a ninguna visita de control por su tocólogo. En la exploración se obseva un útero de aproximadamente 8 semanas de gestación, abdomen blando y depresible y cérvix permeable a un dedo. Poco después la paciente empieza a sangrar abundantemente, mucho más que una regla. El tratamiento indicado es: A) Ingreso y observación. B) Laparoscopia. C) Legrado. D) Laparotomía. E) Venoclisis de oxitocina. 82. Primigesta de 27 años, sin antecedentes médicos de interés, que consulta por metrorragia insidiosa y recurrente en semana 36 de embarazo. No presenta dolor abdominal, el útero está relajado y el latido fetal se escucha vigoroso. El diagnóstico más probable será: A) Vasa praevia. B) Desprendimiento prematuro de placenta. C) Expulsión del tapón mucoso. D) Placenta previa. E) Pérdida de líquido amniótico hemático. 83. Tercigesta isoinmunizada que presenta test de Coombs indirecto de 1/10 en semana 30 de embarazo. Se realiza un amniocentesis en que se determina la madurez fetal (se confirma la presencia de fosfatidil glicerol en líquido amniótico) y el nomograma de Liley, que se encuentra en la zona II. La actitud indicada es: A) Nueva amniocentesis en una semana. B) Seguimiento ecográfico con la paciente hospitalizada. C) Administración de corticoides para inducir la madurez fetal. D) Extracción fetal. E) Administración endovenosa materna de IgG contra Ac anti D. 84. Gestante de 33 semanas que consulta por dinámica uterina, disminución de movimientos fetales y febrícula. Al ingreso presenta una analítica de sangre con leucocitosis moderada y el resto de los parámetros normales. En la ecografìa se observa un oligoamnios y un perfil biofísico fetal de 9. El tratamiento consiste en: A) Antibióticos endovenosos. B) Antibióticos intracavitarios. C) Inducción del parto. D) Cultivo del líquido amniótico y tratamiento según antibiograma. E) Administración de corticoides para favorecer la madurez fetal e inducción del parto en semana 36. 85. Gestante de embarazo gemelar que presenta dinámica espontánea en semana 36. Ambos gemelos se encuentran en presentación cefálica. El parto del primero se produce sin complicaciones, pero el segundo se encuentra en posición occipito iliaca derecha transversa al cabo de 25 minutos desde el nacimiento del primero, sin progresar desde un III plano de Hodge. Para finalizar el parto está indicada la realización de: A) Vacuum. B) Fórceps. C) Cesarea. D) Maniobra de Kristeller. E) Cualquiera de las anteriores según el estado fetal. 86. Primigesta de 36 años que presenta un aumento de la tensión diastólica de 30 mmHg respecto a tomas iniciales, y albuminuria con edemas generalizados. Actualmente se encuentra en la 34 semana de embarazo. Súbitamente inicia un cuadro de cefalea y transtornos visuales. Vd. le inicia tratamiento médico con: A) Nifedipina. B) Dihidralacina. C) Alfametildopa. D) Sulfato de magnesio. E) Labetalol.
  • 22. Examen Simulacro :: Ciclo de Revisión en Medicina :: Sábado 4 de Agosto del 2012 :: www.estudiosmyc.com 87. En la paciente del caso anterior el estudio fetal descubre un feto afecto de crecimiento intrauterino retardado, en el que la relación entre el área cefálica y el área abdominal es mayor de uno. El feto se encuentra en situación cefálica. La conducta obstétrica adecuada es: A) Expectante, con parto vaginal como via de elección. B) Controles de bienestar fetal (perfil biofísico) semanales hasta la semana 40. C) Valoración de la madurez fetal e inducción del parto, si el feto es maduro. D) Cesárea inmediata. E) Evaluación de la funcionalidad placentaria mediante ecografía Doppler. 88. Puérpera que acaba de parir mediante parto eutócico un varón de 3,450 kg. a los 30 minutos no ha alumbrado, por lo que se realiza una maniobra de Credé para extraer la placenta. Transcurridos unos minutos presenta un cuadro de disnea y hemorragia profusa. El útero está bien contraído, pero el sangrado no cesa. El diagnóstico más probable es: A) Ruptura uterina. B) Desgarro de cérvix. C) Hipotonía uterina. D) Retención de restos placentarios. E) Coagulación intravascular diseminada. 89. Un paciente de 26 años de edad, adicto a drogas por vía parenteral, consulta por malestar general, fiebre de 39°con tiritona, escalofríos y dolor y tumefacción en rodilla derecha. En la exploración llama la atención un soplo cardíco panfocal que previamente no estaba en la historia del enfermo y artritis de rodilla derecha. No se pudo realizar artrocentesis diagnóstica. La cobertura empírica antibiótica más segura sería: A) Vancomicina. B) Vancomicina y Gentamicina. C) Ciprofloxacina. D) Eritromicina y cefuloxima. E) Ceftriaxona. 90. Un joven de 18 años sin antecedentes de interés, consulta por tos, fiebre y otalgia. En la Rx de tórax presenta infiltrado intersticial derecho. La exploración ORL demuestra miringitis ampollosa. Iniciaríamos tratamiento con: A) Ceftriaxona. B) Vancomicina. C) Norfloxacina. D) Isoniazida, rifampicina y pirazinamida. E) Eritromicina. 91. Un paciente al que le detecta una infección urinaria por Pseudomona inicia tratamiento con ceftacidima. Tras 2 días de tratamiento el paciente comienza a encontrarse peor, la fiebre aumenta hasta los 39° y desarrolla hipotensión. En el hemograma destaca leucopenia e importante trombopenia que previamente no estaban. Bioquímicamente presenta hiperglucemia de 198 g./dl. Probablemente nos encontramos ante: A) Efecto secundario de la ceftacidima. B) Evolución natural del proceso. C) Situación de shock séptico. D) Probable asociación de una hemopatía. E) Cetoacidosis diabética. 92. Un varón de 27 años consulta por presenta en la región balanoprepucial una úlcera de 1 cm. de tamaño de borde indurado, no doloroso y que secreta serosidad. Presenta adicionalmente adenopatías inguinales bilaterales y fiebre. Estaríamos obligados a solicitarle en el estudio: A) Aglutinaciones a salmonella. B) Aglutinaciones a brucella. C) Serología de VIH. D) Serología luética. E) Serología de micoplasma. 93. Una paciente de 19 años ha sido diagnosticada de mononucleosis infecciosa, confirmada mediante Paul- Bunnel. Ha sido tratada con paracetamol a dosis de 2,5 g./día. A los cinco días del diagnóstico comienza de forma brusca con un cuadro de abdomen agudo y shock. El proceso más probable es: A) Rotura espontánea del bazo. B) Hepatitis por virus de Epstein-Barr. C) Complicación infecciosa intraabdominal. D) Complicación del tratamiento. E) Otra cualquier causa de abdomen agudo: p. ej. apendicitis aguda. 94. Un lactante de 7 meses de edad, con Tetralogía de Fallot intervenida, desarrolla un cuadro de dificultad respiratoria, sibilancias y tos. No se termometra fiebre. Además de las medidas de soporte, el tratamiento de elección sería: A) Reintervención quirúrgica de su cardiopatía. B) Esteroides a dosis plenas. C) Ribavirina en aerosol. D) Eritromicina intravenosa. E) Ceftriaxona intravenosa. 95. Una paciente de 49 años consulta en un Servicio de Urgencias por dolor, enrojecimiento y tumefacción de la parte distal de su miembro inferior derecho. Por sospecha de trombosis venosa profunda se le realiza flebografía que resulta ser negativa. La actitud terapéutica a seguir es: A) Tratamiento antiinflamatorio. B) Heparina de bajo peso molecular. C) Cobertura empírica antibiótica con Oxacilina. D) Reposo del miembro sin más. E) Repetir flebografía pasadas 48 horas.
  • 23. Examen Simulacro :: Ciclo de Revisión en Medicina :: Sábado 4 de Agosto del 2012 :: www.estudiosmyc.com 96. Un adicto a drogas por vía parenteral consulta por tumoración fluctuante y con signos inflamatorios en la flexura del codo donde ha realizado inyecciones intravenosas. Se debe realizar: A) Tratamiento quirúrgico de drenaje. B) Cobertura con ceftriaxona. C) Cobertura con vancomicina. D) Cobertura con Oxacilina. E) Medidas antiinflamatorias. 97. Una pareja de turistas que han regresado a Lima, luego de estar en la Selva Central del Perú, comienzan a presentar deposiciones en cantidad abundante de características líquidas, similares al agua, en número de 20-25 al día. No se acompaña de dolor abdominal ni fiebre. El cuadro obliga a una rehidratación y ésta se consigue con dificultad dada la gran pérdida de agua y electrólitos. El cuadro clínico más probable es: A) Gastroenteritis por Salmonella. B) Disentería bacilar. C) Disentería amebiana. D) Cólera. E) Gastroenteritis viral. 98. Un enfermo leucémico, muy inmunodeprimido, desarrolla tras un tratamiento antibacteriano de amplio espectro, cuadro importante de insuficiencia respiratoria y fiebre. En la Rx. de tórax se aprecia una masa densa, cubierta por un menisco delgado de aire en el interior de una cavidad. El diagnóstico más probable es: A) Tuberculosis. B) Neumonía bacteriana. C) Masa tumoral sobreinfectada. D) Neumonía por cándidas. E) Aspergiloma. 99. Un niño de 3 años desarrolla un cuadro de infección respiratoria de vías altas con fiebre elevada. Posteriormente aparece tos paroxística con gallo inspiratorio. El tratamiento de elección sería: A) Ceftriaxona. B) Cefonicid. C) Cefalotina. D) Eritromicina. E) Amoxicilina-clavulánico. 100. Un paciente de 56 años acude a su médico de cabecera por presentar en el último mes fiebre diaria, con una distribución de dos picos, matutino y vespertino. A la exploración llama la atención una hepatoesplenomegalia muy importante. No se objetivan adenopatías a ningún nivel. El hemograma muestra pancitopenia. La prueba diagnóstica que se debería realizar sería: A) Estudio de médula ósea. B) Ecografía abdominal. C) Marcadores tumorales. D) Biopsia hepática. E) Marcadores de hepatitis.